SlideShare a Scribd company logo
ĐẠI HỌC QUỐC GIA HÀ NỘI
TRƯỜNG ĐẠI HỌC KHOA HỌC TỰ NHIÊN
Nguyễn Thành Giáp
MỘT SỐ BÀI TOÁN VỀ DÃY SỐ
LUẬN VĂN THẠC SĨ KHOA HỌC
Hà Nội - 2011
ĐẠI HỌC QUỐC GIA HÀ NỘI
TRƯỜNG ĐẠI HỌC KHOA HỌC TỰ NHIÊN
Nguyễn Thành Giáp
MỘT SỐ BÀI TOÁN VỀ DÃY SỐ
Chuyên ngành: Phương pháp toán sơ cấp
Mã số: 60.46.40
LUẬN VĂN THẠC SĨ KHOA HỌC
Người hướng dẫn khoa học:
TS. Phạm Văn Quốc
Hà Nội - 2011
Mục lục
Lời nói đầu 3
Chương 1. Một số kiến thức chuẩn bị 5
1.1. Dãy số . . . . . . . . . . . . . . . . . . . . . . . . . . . . . . . . . . . . . . . . . . . . . . . . . . . . . . . . . . . . 5
1.1.1. Định nghĩa . . . . . . . . . . . . . . . . . . . . . . . . . . . . . . . . . . . . . . . . . . . . . . . . . . 5
1.1.2. Cách cho một dãy số . . . . . . . . . . . . . . . . . . . . . . . . . . . . . . . . . . . . . . . . 6
1.1.3. Một vài dãy số đặc biệt . . . . . . . . . . . . . . . . . . . . . . . . . . . . . . . . . . . . . . 6
1.1.4. Giới hạn của dãy số . . . . . . . . . . . . . . . . . . . . . . . . . . . . . . . . . . . . . . . . . 8
1.2. Sơ lược về phương pháp sai phân . . . . . . . . . . . . . . . . . . . . . . . . . . . . . . . . . 11
1.3. Số học . . . . . . . . . . . . . . . . . . . . . . . . . . . . . . . . . . . . . . . . . . . . . . . . . . . . . . . . . . . 14
1.3.1. Đồng dư thức . . . . . . . . . . . . . . . . . . . . . . . . . . . . . . . . . . . . . . . . . . . . . . 14
1.3.2. Một số định lý cơ bản của số học . . . . . . . . . . . . . . . . . . . . . . . . . . . 15
Chương 2. Tính chất số học của dãy số 17
2.1. Tính chia hết . . . . . . . . . . . . . . . . . . . . . . . . . . . . . . . . . . . . . . . . . . . . . . . . . . . . . 17
2.2. Tính chất số nguyên . . . . . . . . . . . . . . . . . . . . . . . . . . . . . . . . . . . . . . . . . . . . . . 36
1
2.3. Tính chính phương . . . . . . . . . . . . . . . . . . . . . . . . . . . . . . . . . . . . . . . . . . . . . . . . 46
2.4. Bài tập . . . . . . . . . . . . . . . . . . . . . . . . . . . . . . . . . . . . . . . . . . . . . . . . . . . . . . . . . . . 57
Chương 3. Giới hạn của dãy số 60
3.1. Giới hạn của tổng . . . . . . . . . . . . . . . . . . . . . . . . . . . . . . . . . . . . . . . . . . . . . . . . . 60
3.2. Dãy con và sự hội tụ của dãy số . . . . . . . . . . . . . . . . . . . . . . . . . . . . . . . . . . . 65
3.3. Dãy số xác định bởi phương trình . . . . . . . . . . . . . . . . . . . . . . . . . . . . . . . . . 73
3.4. Bài tập . . . . . . . . . . . . . . . . . . . . . . . . . . . . . . . . . . . . . . . . . . . . . . . . . . . . . . . . . . . 81
Kết luận 84
Tài liệu tham khảo 85
2
Lời nói đầu
Dãy số là một lĩnh vực khó và rất rộng, trong các đề thi học sinh giỏi quốc
gia, quốc tế cũng thường xuất hiện các bài toán về dãy số. Để giải được các bài
toán về dãy số đòi hỏi người làm toán phải có kiến thức tổng hợp về số học,
đại số, giải tích. Các vấn đề liên quan đến dãy số cũng rất đa dạng và cũng có
nhiều tài liệu viết về vấn đề này, các tài liệu này cũng thường viết khá rộng về
các vấn đề của dãy số, các vấn đề được quan tâm nhiều hơn là các tính chất số
hoc và tính chất giải tích của dãy số.
Tính chất số học của dãy số thể hiện như tính chia hết, tính nguyên, tính
chính phương. . . , tính chất giải tích có nhiều dạng nhưng quan trọng là các bài
toán tìm giới hạn dãy số. Các bài toán về dãy số thường là các bài toán hay và
khó, tác giả luận văn đã sưu tầm, chọn lọc và phân loại theo từng chủ đề.
Luận văn với đề tài “Một số bài toán về dãy số” có mục đích trình bày
một cách hệ thống, chi tiết tính chất số học của dãy số, giới hạn dãy số. Luận
văn được trình bày với 3 chương.
Chương 1. Một số kiến thức chuẩn bị. Chương này hệ thống lại kiến thức
cơ bản nhất về dãy số, số học, phương pháp sai phân sẽ được dùng để giải quyết
các bài toán trong các chương sau.
Chương 2. Tính chất số học của dãy số. Chương này trình bày một số vấn
đề về tính chất số học của dãy số như tính chia hết, tính nguyên, tính chính
phương. . . và nêu ra các phương pháp giải toán, phân tích các bài toán cụ thể.
3
Chương 3. Giới hạn của dãy số. Chương này đề cập đến một số bài toán
về giới hạn dãy số như: Giới hạn của tổng, dãy con và sự hội tụ của dãy số, dãy
số xác định bởi phương trình cùng với phương pháp giải cụ thể cho từng dạng
toán.
Luận văn được hoàn thành với sự quan tâm giúp đỡ, hướng dẫn khoa học
của TS. Phạm Văn Quốc, thày đã tận tình chỉ bảo cách tập nghiên cứu khoa
học, cách làm và trình bày bản luận văn này đồng thời thày cũng có nhiều ý
kiến quý báu để hoàn thành luận văn. Tác giả xin bày tỏ lòng cảm ơn sâu sắc
nhất tới thày.
Nhân dịp này tác giả cũng xin cảm ơn khoa Toán – Cơ – Tin học, phòng
Sau đại học, phòng Công tác chính trị sinh viên trường Đại học Khoa học Tự
nhiên – Đại học Quốc gia Hà nội đã tạo điều kiện giúp đỡ tác giả trong suốt
hai năm học cũng như trong quá trình làm luận văn, cảm ơn Ban giám hiệu,
các bạn đồng nghiệp trường THPT Nguyễn Trung Ngạn đã giúp đỡ cho tác giả
trong công tác và trong học tập thời gian qua, tác giả cũng xin cảm ơn gia đình,
bạn bè đã cổ vũ, động viên tác giả vượt qua mọi khó khăn để hoàn thành bản
luận văn này.
Hà Nội, ngày 25 tháng 11 năm 2011
Học viên
Nguyễn Thành Giáp
4
Chương 1
Một số kiến thức chuẩn bị
1.1. Dãy số
1.1.1. Định nghĩa
Mỗi hàm số u xác định trên tập các số nguyên dương N∗
được gọi là
một dãy số vô hạn (gọi tắt là dãy số). Kí hiệu:
u :N∗
−→ R
n −→ u(n)
Dãy số thường được viết dưới dạng khai triển: u1, u2, ..., un, .... Trong đó
un = u(n) và gọi u1 là số hạng đầu, un là số hạng thứ n và là số hạng tổng quát
của dãy số.
Mỗi hàm số u xác định trên tập M = 1, 2, 3, . . . , m với m ∈ N∗
được gọi là
một dãy số hữu hạn. Dạng khai triển của nó là u1, u2, ..., um trong đó u1 là số
hạng đầu, um là số hạng cuối.
Dãy số (un) được gọi là:
• Dãy đơn điệu tăng nếu un+1 > un với mọi n = 1, 2, ...
5
• Dãy đơn điệu không giảm nếu un+1 ≥ un với mọi n = 1, 2, ...
• Dãy đơn điệu giảm nếu un+1 < un với mọi n = 1, 2, ...
• Dãy đơn điệu không tăng nếu un+1 ≤ un với mọi n = 1, 2, ...
Dãy (un) được gọi là:
• Dãy số bị chặn trên nếu tồn tại số M sao cho un < M, với mọi N = 1, 2, ...
• Dãy số bị chặn dưới nếu tồn tại số m sao cho un > m, với mọi N = 1, 2, ...
• Dãy số bị chặn nếu nó vừa bị chặn trên vừa bị chặn dưới.
Dãy số (un) được gọi là tuần hoàn với chu kỳ k nếu un+k = un, với mọi n ∈ N.
Dãy số (un) được gọi là dãy dừng nếu tồn tại một số N0 sao cho un = C với
mọi n ≥ N0. (C là hằng số, gọi là hằng số dừng)
1.1.2. Cách cho một dãy số
Dãy số cho bằng công thức của số hạng tổng quát: Ví dụ xét dãy số
(un) được cho bởi
un =
1
√
5
1 +
√
5
2
n
−
1
√
5
1 −
√
5
2
n
.
Dãy số cho bằng phương pháp truy hồi: Dãy số (un) được xác định bởi



u1 = 1, u2 = 50
un+1 = 4un + 5un−1 − 1975, n = 2, 3, 4, ...
Dãy số cho bằng phương pháp mô tả: Ví dụ xét dãy số (un) được cho bởi:
a1 = 19, a2 = 98. Với mỗi số nguyên n ≥ 1, xác định an+2 bằng số dư của phép
chia an + an+1 cho 100.
6
1.1.3. Một vài dãy số đặc biệt
1.1.3.1. Cấp số cộng
Định nghĩa 1.1.1. Dãy số u1, u2, u3, ... được gọi là một cấp số cộng với công
sai d (d khác 0) nếu un = un−1 + d với mọi n = 2, 3, ...
Tính chất:
1. un = u1 + (n − 1)d.
2. uk =
uk−1 + uk+1
2
với mọi k = 2, 3, ...
3. Nếu cấp số cộng có hữu hạn phần tử u1, u2, ..., un thì u1 +un = uk +un−k
với mọi k = 2, 3, ..., n − 1 và
Sn = u1 + u2 + · · · + un =
n
2
(u1 + un) =
n
2
[2u1 + (n − 1)d].
1.1.3.2. Cấp số nhân
Định nghĩa 1.1.2. Dãy số u1, u2, u3, ... được gọi là một cấp số nhân với công
bội q (q khác 0 và khác 1) nếu un = un−1q với mọi n = 2, 3, ...
Tính chất:
1. un = u1qn−1
với mọi n = 2, 3, ....
2. u2
k = uk−1 · uk+1 với mọi k = 2, 3, ...
3. Sn = u1 + u2 + · · · + un =
u1(qn
− 1)
q − 1
.
1.1.3.3. Dãy Fibonacci
Định nghĩa 1.1.3. Dãy u1, u2, ... được xác định như sau:



u1 = 1, u2 = 1
un = un−1 + un−2, với mọi n = 2, 3, ...
7
được gọi là dãy Fibonacci.
Bằng phương pháp sai phân có thể tìm được công thức tổng quát của dãy
là:
un =
1
√
5
1 +
√
5
2
n
−
1
√
5
1 −
√
5
2
n
.
1.1.4. Giới hạn của dãy số
Định nghĩa 1.1.4. Ta nói rằng dãy số (un) có giới hạn là hằng số thực a hữu
hạn nếu với mọi số dương ε (có thể bé tùy ý), luôn tồn tại chỉ số n0 ∈ N (n0
có thể phụ thuộc vào ε và vào dãy số (un) đang xét), sao cho với mọi chỉ số
n ∈ N, n ≥ n0 ta luôn có |un − a| < ε. Khi đó kí hiệu lim
n→+∞
un = a hoặc
lim un = a và còn nói rằng dãy số (un) hội tụ về a. Dãy số không hội tụ gọi là
dãy phân kì.
Định lý 1.1.5. Nếu một dãy số hội tụ thì giới hạn của nó là duy nhất.
Định lý 1.1.6 (Tiêu chuẩn hội tụ Weierstrass).
a) Một dãy số đơn điệu và bị chặn thì hội tụ.
b) Một dãy số tăng và bị chặn trên thì hội tụ.
c) Một dãy số giảm và bị chặn dưới thì hội tụ.
Định lý 1.1.7. Nếu (un) → a và (vn) ⊂ (un), (vn) = C thì (vn) → a.
Định lý 1.1.8 (định lý kẹp giữa về giới hạn). Nếu với mọi n ≥ n0 ta đều
có un ≤ xn ≤ vn và lim un = lim vn = a thì lim xn = a.
Định lý 1.1.9 (định lý Lagrange). Nếu hàm số f(x) liên tục trên [a; b] và
có đạo hàm trong khoảng (a; b) thì tồn tại c ∈ (a; b) thỏa mãn:
f(b) − f(a) = f (c)(b − a).
Định lý 1.1.10 (định lý trung bình Cesaro). Nếu dãy số (un) có giới hạn
hữu hạn là a thì dãy các trung bình cộng
u1 + u2 + · · · + un
n
cũng có giới hạn
8
là a.
Định lý này có thể phát biểu dưới dạng tương đương sau:
Định lý 1.1.11 (định lý Stolz). Nếu lim
n→+∞
(un+1 − un) = a thì
lim
n→+∞
un
n
= a.
Chứng minh. Ta chỉ cần chứng minh cho trường hợp a = 0. Vì
lim
n→+∞
(un+1 − un) = a nên với mọi ε > 0 luôn tồn tại N0 sao cho với mọi
n ≥ N0, ta có |un+1 − un| < ε. Khi đó, với mọi n > N0 ta có
un
n
≤
1
n
|uN0 | + |uN0+1 − uN0 | + · · · + |un − un−1| < |uN0 | ·
1
n
+ (n − N0) ·
ε
n
Giữ N0 cố định, ta có thể tìm được N1 > N0 sao cho
1
N1|uN0 |
< ε. Khi đó với
mọi n > N1, ta sẽ có
un
n
< 2ε. Vậy nên lim
n→+∞
un
n
= 0.
Định lý 1.1.12. Cho f : D → D là hàm liên tục. Khi đó
1) Phương trình f(x) = x có nghiệm ⇔ phường trình fn(x) = x có nghiệm.
2) Gọi α, β là các mút trái, mút phải của D. Biết lim
x→α+
[f(x) − x] và
lim
x→β−
[f(x)−x] cùng dương hoặc cùng âm. Khi đó, phương trình f(x) = x
có nghiệm duy nhất ⇔ phương trình fn(x) = x có nghiệm duy nhất. Trong
đó fn(x) = f(............f
(n−1) lần f
(x)).
Chứng minh. 1) a) nếu x0 là nghiệm của phương trình f(x) = x thì x0 cũng là
nghiệm của phương trình fn(x) = x.
b) Nếu phương trình f(x) = x vô nghiệm thì f(x)−x > 0 hoặc f(x)−x < 0
với mọi x ∈ D. Do đó fn(x) − x > 0 hoặc fn(x) − x < 0 với mọi x ∈ D nên
phương trình fn(x) = x cũng vô nghiệm.
2) a) Giả sử phương trình f(x) = x có nghiệm duy nhất là x0 thì đây cũng
9
là một nghiệm của phương trình fn(x) = x. Đặt F(x) = f(x) − x, do F(x) liên
tục trên (x0; β) và (α; x0) nên F(x) giữ nguyên một dấu.
Nếu lim
x→α+
[f(x)−x] và lim
x→β−
[f(x)−x] cùng dương thì F(x) > 0 trong khoảng
(x0; β) và (α; x0), suy ra f(x) > x với mọi x ∈ D  {x0}.
Xét x1 ∈ D {x0} suy ra f(x1) > x1 ⇒ f(f(x1)) > f(x1) > x1. Từ đó, ta có
fn(x1) > x1 nên x1 không là nghiệm của phương trình fn(x) = x. Vậy phương
trình fn(x) = x có nghiệm duy nhất x = x0.
Trường hợp lim
x→α+
[f(x) − x] và lim
x→β−
[f(x) − x] cùng âm được chứng minh
tương tự.
b) Ta thấy mọi nghiệm của phương trình f(x) = x đều là nghiệm của phương
trình fn(x) = x, do đó nếu phương trình fn(x) = x có nghiệm duy nhất thì
phương trình f(x) = x có nghiệm duy nhất.
Định lý 1.1.13. Cho hàm f : D → D là hàm đồng biến, dãy (xn) thỏa mãn
f(xn) = xn+1 với mọi n ∈ N∗
. Khi đó:
a) Nếu x1 < x2 thì dãy (xn) tăng.
b) Nếu x1 > x2 thì dãy (xn) giảm.
Chứng minh. a) Ta chứng minh bằng phương pháp quy nạp. Với n = 1 ta
có x1 < x2, mệnh đề đúng. Giả sử mệnh đề đúng với n = k (k ≥ 1) tức là
xk < xk+1. Do f(·) là hàm đồng biến nên f(xk) < f(xk+1), suy ra xk+1 < xk+2.
b) Chứng minh tương tự.
Định lý 1.1.14. Cho hàm f : D → D là hàm nghịch biến, dãy (xn) thỏa mãn
xn+1 = f(xn) với mọi n ∈ N∗
. Khi đó:
a) Các dãy (x2n+1) và (x2n) đơn điệu, trong đó có một dãy tăng và một dãy
giảm.
b) Nếu dãy (xn) bị chặn thì tồn tại α = lim x2n và β = lim x2n+1.
10
c) Nếu f(x) liên tục thì α và β là các nghiệm của phương trình
f(f(x)) = x. (1.1)
Vì vậy nếu (1.1) có nghiệm duy nhất thì α = β và lim xn = α = β.
Chứng minh. a) Vì f(x) là hàm nghịch biến nên f(f(x)) là hàm đồng biến. Áp
dụng định lý 1.1.6, ta có điều phải chứng minh.
b) Suy ra từ a).
c) Ta có f(f(x2n)) = f(x2n+1) = x2n+2 và
lim f(f(x2n)) = lim x2n+2 = α, lim x2n = α
do f(x) liên tục nên f(f(α)) = α. Chứng minh tương tự ta có f(f(β)) = β.
Vậy α, β là nghiệm của phương trình f(f(x)) = x.
1.2. Sơ lược về phương pháp sai phân
Định nghĩa 1.2.1. Cho hàm số y = f(x) xác định trên R. Đặt xk = x0 + kh
(k ∈ N∗
) với x0 ∈ R, h ∈ R bất kỳ, cho trước. Gọi yk = f(xk), khi đó hiệu số
∆yk := yk+1 − yk, k ∈ N∗
được gọi là sai phân cấp một của hàm số f(x).
Hiệu số ∆2
yk := ∆yk+1 − ∆yk = ∆(∆yk) (k ∈ N∗
) được gọi là sai phân cấp
hai của hàm số f(x). Tổng quát
∆i
yk := ∆i−1
yk+1 − ∆i−1
yk = ∆(∆i−1
yk), k ∈ N∗
được gọi là sai phân cấp i của hàm số f(x) (i = 1, 2, ..., n, ...)
Mệnh đề 1.2.2. Sai phân mọi cấp đều có thể biểu diễn theo các giá trị của
hàm số: y0, y1, y2, ..., yn, ...
11
Định nghĩa 1.2.3. Phương trình sai phân (cấp k) là một hệ thức tuyến tính
chứa sai phân cấp k
f(yn, ∆yn, ∆2
yn, ..., ∆k
yn) = 0 (1.2)
Vì sai phân các cấp có thể biểu diễn theo các giá trị của hàm số nên ta có
thể viết phương trình dạng
a0yn+k + a1yn+k−1 + · · · + akyn = f(n) (1.3)
trong đó a0, a1, ..., ak, f(n) là các giá trị đã biết, còn yn, yn+1, ..., yn+k là các
giá trị chưa biết.
Hàm số yn thỏa mãn (1.2) gọi là nghiệm của phương trình sai phân tuyến
tính (1.2).
Phương trình a0yn+k + a1yn+k−1 + · · · + akyn = f(n) được gọi là phương
trình sai phân tuyến tính cấp k. Để giải phương trình này, chúng ta làm như
sau:
Bước 1. Giải phương trình sai phân tuyến tính thuần nhất tương ứng
a0yn+k + a1yn+k−1 + · · · + akyn = 0. (1.4)
Để giải phương trình trên, ta xét phương trình đặc trưng
a0λk
+ a1λk−1
+ · · · + ak−1λ + ak = 0. (*)
Khi đó nếu (∗) có k nghiệm thực khác nhau λ1, λ2, ..., λk thì nghiệm tổng
quát của (1.4) là
ˆyn = c1λn
1 + c2λn
2 + · · · + ckλn
k (1.5)
trong đó c1, c2, .., ck là các hằng số tùy ý.
Nếu (∗) có nghiệm thực λj bội s thì nghiệm tổng quát của (1.4) là
ˆyn =
s−1
i=1
cj+ini
λn
j +
k
i=1,i=j
ciλn
i .
12
Nếu phương trình (∗) có nghiệm phức đơn λj = r(cos θ + i sin θ) thì cũng
có nghiệm λj = r(cos θ − i sin θ). Đặt λj+1 = λj. Để thu được công thức tổng
quát, trong công thức (1.5) ta thay bộ phận cjλn
j +cj+1λn
j+1 bởi bộ phận tương
ứng cjrn
cos nθ + cj+1rn
sin nθ.
Nếu phương trình (∗) có nghiệm phức bội s:
λj = λj+s+1 = ... = λj+2s−1 = r(cos θ − i sin θ).
Trong trường hợp này để thu được công thức nghiệm tổng quát, trong công
thức (1.5) ta thay bộ phận
cjλn
j + cj+1λn
j+1 + · · · + cj+2s−1λn
j+2s−1
bởi bộ phận tương ứng
s−1
i=0
cj+ini
rn
cos nθ+
s−1
i=0
cj+s+ini
rn
sin nθ
Bước 2. Tìm nghiệm tổng quát của phương trình sai phân tuyến tính cấp
k. Nghiệm tổng quát có dạng
yn = ˆyn + y∗
n
trong đó yn là nghiệm của phương trình sai phân tuyến tính cấp k, ˆyn là nghiệm
của phương trình sai phân tuyến tính thuần nhất tương ứng và y∗
n là một nghiệm
riêng của phương trình không thuần nhất.
13
1.3. Số học
1.3.1. Đồng dư thức
1.3.1.1. Định nghĩa
Nếu hai số nguyên a và b khi chia cho m (m = 0) mà có cùng số dư thì
ta nói a đồng dư với b theo modun m, kí hiệu là a ≡ b(mod m). Như vậy
a ≡ b(mod m) ⇔ a − b chia hết cho m.
Hệ thức dạng: a ≡ b(mod m) gọi là một đồng dư thức, a gọi là vế trái của
đồng dư thức, b gọi là vế phải, còn m gọi là môđun.
1.3.1.2. Một số tính chất
Kí hiệu a, b, c, d, m, ... là các số nguyên dương (Z+
), ta luôn có:
Tính chất 1. • a ≡ a(mod m).
• a ≡ b(mod m) ⇔ b ≡ a(mod m).
• a ≡ b(mod m) và b ≡ c(mod m) thì a ≡ c(mod m)
Tính chất 2. Nếu a ≡ b(mod m) và c ≡ d(mod m) thì:
• a ± c ≡ b ± d(mod m),
• ac ≡ bd(mod m),
• Nếu d là một ước chung của a, b, m thì
a
d
≡
b
d
mod
m
d
.
Tính chất 3. Nếu a ≡ b(mod m) và c ∈ Z+
thì ac ≡ bc(mod mc).
1.3.1.3. Một số kiến thức liên quan
• Với mọi a, b ∈ Z+
(a = b) và n là số tự nhiên: (an
− bn
)
...(a − b);
14
• Trong n số nguyên liên tiếp (n ≥ 1) có một và chỉ một số chia hết cho n;
• Lấy n + 1 số nguyên bất kỳ (n ≥ 1) đem chia cho n thì phải có hai số
khi chia cho n có cùng số dư (theo nguyên lý Dirichlet).
• Tìm m chữ số tận cùng của số A là tìm số dư khi chia A cho 10m
.
1.3.2. Một số định lý cơ bản của số học
1.3.2.1. Định lý Euler
Hàm số Euler-µ(m): Cho hàm số µ(m) được xác định như sau
• m = 1, ta có µ(m) = 1,
• m > 1 thì µ(m) là các số tự nhiên không vượt quá m − 1 và nguyên tố
với m.
Công thức tính µ(m): Bước 1. Nếu m = pα
(p là số nguyên tố, α là số
tự nhiên khác 0). Ta có
µ(m) = µ(pα
) = pα
1 −
1
p
.
Bước 2. Nếu m = pα1
1 pα2
2 · · · pαn
n (pi là các số nguyên tố, αi là số tự nhiên
khác 0). Ta có
µ(m) = m 1 −
1
p1
1 −
1
p2
· · · 1 −
1
pn
.
Định lý 1.3.1 (định lý Euler). Cho m là số tự nhiên khác 0 và a là số nguyên
tố với m. Khi ấy, ta có
aµ(m)
≡ 1(modm).
15
1.3.2.2. Định lý Fermat
Định lý 1.3.2 (định lý Fermat). Cho p là một số nguyên tố và a là một số
nguyên không chia hết cho p. Khi ấy, ta có
ap−1
≡ 1(modp).
Đối với số nguyên a bất kỳ ta có ap
≡ a(modp).
16
Chương 2
Tính chất số học của dãy số
Dãy số nguyên là phần quan trọng trong lý thuyết dãy số. Ngoài các
vấn đề chung như tìm số hạng tổng quát của dãy số, tìm công thức tính tổng n
số hạng đầu tiên. . . các bài toán về dãy số thường quan tâm đến tính chất số
học của dãy số như tính chia hết, đồng dư, nguyên tố, chính phương, nguyên
tố cùng nhau. . . Các bài toán về dãy số nguyên rất đa dạng, trong nhiều trường
hợp dãy số chỉ là cái bề ngoài còn bản chất bài toán là bài toán số học.
2.1. Tính chia hết
Một số bài toán về sự chia hết của các số hạng của dãy số thường được
giải bằng cách xác định số hạng tổng quát của dãy số sau đó dựa vào các định
lý về đồng dư để chứng minh sự chia hết. Việc xác định số hạng tổng quát của
dãy số thường được tìm bằng phương pháp sai phân hoặc thông qua dãy số phụ
để đưa về phương trình sai phân thuần nhất.
17
Bài tập 2.1.1. Dãy số (un) được xác định như sau:



u1 = 1, u2 = 50
un+1 = 4un + 5un−1 − 1975, n = 2, 3, 4, ...
Chứng minh rằng u1996 chia hết cho 1997.
Lời giải. Ta tìm số hạng tổng quát của dãy số bằng phương pháp sai phân.
Công thức truy hồi của dãy là tuyến tính nhưng không thuần nhất, do vậy ta
đặt un = vn + c để có công thức truy hồi vn+1 = 4vn + 5vn−1. Ta có
vn+1 + c = 4(vn + c) + 5(vn−1 + c) − 1975 ⇔ vn+1 = 4vn + 5vn−1 + 8c − 1975.
Do đó c =
1975
8
. Khi đó dãy (vn) được xác định như sau:



v1 = −
1919
8
, v2 = −
1575
8
vn+1 = 4vn + 5vn−1, n = 2, 3, 4, ...
Phương trình đặc trưng: x2
− 4x − 5 = 0 có hai nghiệm x1 = 5 và x2 = −1
nên vn = c15n
+c2(−1)n
. Cho n = 1, n = 2 ta tìm được c1 = −
1747
120
, c2 =
2005
12
.
Do đó
vn = −
1747
120
· 5n
+
2005
12
· (−1)n
⇒ un = −
1747
120
· 5n
+
2005
12
· (−1)n
+
1975
8
.
Với n = 1996, ta có
u1996 = −
1747
120
· 51996
+
9935
24
=
−1747 · 51996
+ 49675
120
.
Do 1997 là số nguyên tố nên 51996
≡ 1(mod1997). Nên
−1747 · 51996
+ 49675 ≡ 0(mod1997).
Lại có 51996
= 25998
≡ 1(mod3) nên
−1747 · 51996
+ 49675 ≡ −1747 + 49675 ≡ 0(mod3).
18
Và 51996
= 25998
≡ 1(mod8) nên
−1747 · 51996
+ 49675 ≡ −1747 + 49675 ≡ 0(mod8).
Hơn nữa, rõ ràng −1747 · 51996
+ 49675 ≡ 0(mod5) mà (3, 5, 8) = 1 nên suy ra
−1747 · 51996
+ 49675
...120.
Mặt khác (120, 1997) = 1 nên
−1747 · 51996
+ 49675
120
chia hết cho 1997, hay
là u1996 chia hết cho 1997.
Bài tập 2.1.2 (HSG QG 2011). Cho dãy số nguyên (an) được xác định bởi:



a0 = 1, a1 = −1
an = 6an−1 + 5an−2 với mọi n ≥ 2.
Chứng minh rằng a2012 − 2010 chia hết cho 2011.
Lời giải. Cách 1. Xét dãy số nguyên (bn) được xác định bởi:



b0 = 1, b1 = −1
bn = 6bn−1 + 2016bn−2 với mọi n ≥ 2.
Dễ thấy rằng với mọi n ≥ 0, ta có an ≡ bn(mod2011). Phương trình đặc
trưng của dãy (bn):
x2
− 6x − 2016 = 0 ⇔ (x − 48)(x + 42) = 0 ⇒ x2 = 48, x1 = −42.
Suy ra số hạng tổng quát của dãy (bn) có dạng:
bn = C1 · (−42)n
+ C2 · (48)n
.
Từ các điều kiện ban đầu của dãy (bn), ta có hệ:



C1 + C2 = 1
42C1 − 48C2 = 1
⇒



C1 =
49
90
C2 =
41
90
.
19
Vì vậy bn =
49 · (−42)n
+ 41 · 48n
90
với mọi n ≥ 0. Vì 2011 là số nguyên tố
nên theo định lý Fermat nhỏ, ta có
(−42)2010
≡ 482010
≡ 1(mod2011).
Do đó
90b2012 = 49 · (−42)2012
+ 41 · 482012
≡ 49 · (−42)2
+ 41 · 482
≡ 90b2(mod2011).
Suy ra b2012 ≡ b2(mod2011) (vì (90, 2011) = 1). Mà b2 = 6b1 + 2016b0 nên
b2012 ≡ 2010(mod2011).
Vì thế a2012 ≡ 2010(mod2011).
Cách 2. Số hạng tổng quát của dãy (an):
an =
1
2
−
2
√
14
(3 +
√
14)n
+
1
2
+
2
√
14
(3 −
√
14)n
. (1)
Đặt p = 2011, ta có
ap+1 =
1
2
−
2
√
14
(3 +
√
14)p+1
+
1
2
+
2
√
14
(3 −
√
14)p+1
.
Do
(3 +
√
14)p+1
= Ap+1 + Bp+1 ·
√
14 và (3 −
√
14)p+1
= Ap+1 − Bp+1 ·
√
14
trong đó
Ap+1 =
p+1
2
i=0
C2i
p+1 · 32i
· 14
p+1
2 − i (2)
và
Bp+1 =
p+1
2
i=1
C2i−1
p+1 · 32i−1
· 14
p+1
2 − i (3)
nên
ap+1 = Ap+1 − 4Bp+1. (4)
20
Do p nguyên tố nên Ck
p ≡ 0(modp) với mọi k = 1, p − 1. Do đó từ
Ck
p+1 = Ck
p + Ck−1
p , suy ra Ck
p+1 ≡ 0(modp) với mọi k = 2, p − 1.
Vì vậy từ (2) và (3) ta được Ap+1 ≡ 14
p+1
2 + 3p+1
(modp). Và
Bp+1 ≡ 3(p + 1) 14
p−1
2 + 3p−1
≡ 3 14
p−1
2 + 3p−1
(modp). (5)
Do đó từ (4) suy ra ap+1 ≡ − 3p
+ 2 · 14
p−1
2 (modp). Mặt khác, ta có
452
≡ 14(modp)
và (45, 14) = 1, theo định lý Fermat nhỏ ta có: 3p
≡ 3(modp) và
14
p−1
2 ≡ 45p−1
≡ 1(modp).
Do đó từ (5) ta được
a2012 = ap+1 ≡ −3 + 2 = −1 ≡ 2010(mod2011).
Việc tìm số hạng tổng quát của dãy số cũng có thể thông qua biến đổi liên
tiếp các số hạng phụ thuộc nhau và biểu diễn qua một vài số hạng đầu và có
thể áp dụng phương pháp quy nạp để chứng minh.
Bài tập 2.1.3. Cho dãy số (un) xác định như sau:



u1 = 3, u2 = 14
nun+2 − (5n + 1)un+1 + 4(n + 1)un = 1 (n ∈ N∗
)
Đặt S = 3
2011
n=1
un − 4(24022
− 1). Chứng minh rằng S chia hết cho 2011.
Lời giải. Ta có
nun+2 − (5n + 1)un+1 + 4(n + 1)un = 1
⇒ n[un+2 − 4un+1 − (n + 1) + 1] = (n + 1)(un+1 − 4un − n + 1)
21
Do đó
un+2 − 4un+1 − (n + 1) + 1 =
n + 1
n
(un+1 − 4un − n + 1)
=
n + 1
n
·
n
n − 1
(un − 4un−1 − (n − 1) + 1)
=
n + 1
n
·
n
n − 1
· · ·
2
1
(u2 − 4u1 − 1 + 1)
= 2(n + 1)
Suy ra
un+2 − 4un+1 = 3n + 2
⇒ un+2 + (n + 2) = 4[un+1 + (n + 1)] = 42
(un + n) = ... = 4n
(u2 + 2)
= 4n+2
nên un = 4n
− n đúng với mọi n ∈ N∗
. Từ đó
S = 3
2011
n=1
un − 4(24022
− 1) = 3
2011
n=1
(4n
− n) − 4(24022
− 1)
= 3
2011
n=1
4n
− 3
2011
n=1
n − 4(42011
− 1)
= 4(42011
− 1) − 3 · 2011 · 1006 − 4(42011
− 1)
= −3 · 2011 · 1006
... 2011
Một số dạng toán liên quan đến tính chia hết có thể cho ở dạng tìm số dư
trong phép chia các số hạng của dãy số cho một số nào đó, ta thường sử dụng
các kiến thức về đồng dư để giải toán.
Bài tập 2.1.4. Cho a1 = 19, a2 = 98. Với mỗi số nguyên n ≥ 1, ta xác định
an+2 bằng số dư của phép chia an + an+1 cho 100. Tìm số dư trong phép chia
a2
1 + a2
2 + · · · + a2
1998 cho 8.
Lời giải. Gọi rn là số dư của phép chia an cho 4. Theo giả thiết, ta có
an + an+1 ≡ an+2(mod100)
22
nên an + an+1 ≡ an+2(mod4). Mặt khác rn ∈ {0, 1, 2, 3}, tức là dãy {rn} bị
chặn và do đó dãy {rn} tuần hoàn. Ta tính được r1 = 3, r2 = 2, r3 = 1, r4 = 3,
r5 = 0, r6 = 3, r7 = 3, r8 = 2, r9 = 1. Dễ kiểm tra {rn} có chu kỳ 6, nghĩa là
rn = rn+6 với mọi n ≥ 1. Lại có a2
n − r2
n = (an − rn)(an + rn), do (an − rn)
... 4
nên an, rn đồng tính chẵn lẻ. Từ đó suy ra (an + rn)
... 2, suy ra (a2
n − r2
n)
... 8
hay a2
n ≡ r2
n(mod8). Vậy
a2
1 + a2
2 + · · · + a2
1998 ≡ r2
1 + r2
2 + · · · + r2
1998(mod8).
Mà
r2
1 + r2
2 + · · · + r2
1998 = 333(r2
1 + r2
2 + r2
3 + r2
4 + r2
5 + r2
6)
= 333(9 + 4 + 1 + 9 + 0 + 9)
... 8.
Do đó a2
1 + a2
2 + · · · + a2
1998 chia hết cho 8 hay số dư bằng 0.
Bài tập 2.1.5. Cho λ là nghiệm dương của phương trình
t2
− 1998t − 1 = 0
và dãy (xn) được xác định như sau:



x0 = 1
xn+1 = [λxn], với mọi n ≥ 0.
Trong đó [x] là số nguyên lớn nhất không vượt quá x. Tìm số dư của phép chia
x1998 cho 1998.
Lời giải. Ta có 1998 < λ = 999+
√
9992 + 1 < 1999 nên x1 = 1998, x2
2 = 19982
.
Vì λ là nghiệm dương của phương trình t2
− 1998t − 1 = 0 nên λ là một số vô
tỷ và
λ2
= 1998λ + 1 ⇒ λ = 1998 +
1
λ
⇒ λxn = 1998xn +
xn
λ
.
Dễ thấy xn là số nguyên dương, do đó
[λxn] = 1998xn +
xn
λ
= 1998xn+
xn
λ
.
23
Vì xn = [λxn−1] nên xn < λxn−1 < xn +1 (do λ là số vô tỷ nên λxn−1 /∈ Z),
suy ra
xn
λ
< xn−1 <
xn
λ
+
1
λ
.
Do λ > 1998 nên
xn
λ
= xn−1 − 1 nên
xn+1 = [λxn] = 1998xn +
xn
λ
= 1998xn + xn−1 − 1.
Suy ra
xn+1 ≡ xn−1 − 1(mod1998) ⇒ x1998 ≡ x0 − 999 ≡ 1000(mod1998).
Bài toán trên có thể tổng quát hóa thành bài toán sau: Cho m, p là
hai số nguyên dương sao cho m2
+ 4p không là số chính phương và m ≥ p. Gọi
λ là nghiệm dương của phương trình x2
− mx − p = 0, dãy (xn) được xác định
như sau:



x0 = a
xn+1 = [λxn], với mọi n ≥ 0, với a nguyên dương.
Tìm số dư của xn khi chia cho m.
Lời giải. Vì m2
+4p không là số chính phương nên λ là số vô tỷ. Ta có λ = m +
p
λ
nên λxn = mxn +
pxn
λ
. Dễ thấy dãy đã cho là dãy số nguyên nên
xn+1 = [λxn] = mxn +
pxn
λ
= mxn+
pxn
λ
.
Vì xn = [λxn−1] nên xn < λxn−1 < xn + 1 ⇒
xn
λ
< xn−1 <
xn
λ
+
1
λ
⇒ xn−1 −
1
λ
<
xn
λ
< xn−1 ⇒ pxn−1 −
p
λ
<
pxn
λ
< pxn−1.
Dễ thấy phương trình x2
− mx − p = 0 luôn có hai nghiệm trái dấu và tổng
của chúng bằng m nên nghiệm dương λ > m suy ra λ > p hay
p
λ
< 1. Nên
pxn−1 − 1 < pxn−1 −
p
λ
⇒ pxn−1 − 1 <
pxn
λ
< pxn−1. Khi đó
pxn
λ
= pxn−1 − 1.
24
Vậy
xn+1 = [λxn] = mxn +
pxn
λ
= mxn+
pxn
λ
= mxn + pxn−1 − 1.
Do đó
xn+1 ≡ pxn−1 − 1(modm) ≡ p(pxn−3 − 1) − 1 = p2
xn−3 − (p + 1)(modm)
.........................
≡ pk+1
xn−(2k+1) − (pk
+ · · · + p + 1)(modm).
Nếu n lẻ: n = 2k + 1 ta có
xn+1 = x2k+2 ≡ pk+1
· x0 − (pk
+ · · · + p + 1) = pk+1
· x0 −
pk+1
− 1
p − 1
(modm).
Nếu n chẵn: n = 2k + 2 ta có
x2k+3 ≡ pk+1
· x1 − (pk
+ · · · + p + 1) = pk+1
· x1 −
pk+1
− 1
p − 1
(modm)
trong đó x1 = [λx0].
Nhận xét. Bài toán này có thể đặc biệt hóa với các giá trị cụ thể của m, p
khác nhau thỏa mãn m2
+4p không là số chính phương và m ≥ p, từ đó ta được
nhiều bài toán cụ thể tương tự bài 2.1.5. Chẳng hạn chọn m = 1, p = 1 hoặc
m = 1000, p=3.
Bài tập 2.1.6. Cho dãy (an) xác định bởi:



a0 = 9
an+1 = 27a28
n + 28a27
n , với mọi n = 0, 1, 2, ...
Chứng minh rằng số a11 có tận cùng nhiều hơn 2000 chữ số 9 (viết trong hệ
thập phân).
Lời giải. Ta có
an+1 + 1 = 27a28
n + 28a27
n + 1 = 27a27
n (an + 1) + a27
n + 1
= (an + 1)(27a27
n + a26
n − a25
n + a24
n − ... + a2
n − an + 1)
= (an + 1)[27(a27
n + 1) + (a26
n − 1) − (a25
n − 1) + · · · + (a2
n − 1) − (an + 1)].
25
Do (a27
n + 1)
... (an + 1); (a26
n − 1)
... (an + 1); (a25
n + 1)
... (an + 1) ... nên
(an+1 + 1)
... (an + 1)2
. (1)
Với a0 = 9 và (1) cho n lấy giá trị từ 1 đến 10, ta suy ra (a11 + 1)
... 10211
hay (a11 + 1)
... 102048
, từ đó suy ra a11 có tận cùng nhiều hơn 2000 chữ số 9.
Một số bài toán về dãy số liên quan đến việc tìm điều kiện cho các chỉ số để
số hạng nào đó của dãy số chia hết cho một số cho trước, với dạng toán này ta
thường phải tìm số hạng tổng quát.
Bài tập 2.1.7. Dãy số (un) được xác định như sau:



u1 = 1, u2 = 2, u3 = 40
un =
10u2
n−1 · un−3 − 24un−1 · u2
n−2
un−2 · un−3
, n = 4, 5, ...
Tìm số n nhỏ nhất để un
... 2048.
Lời giải. Nhận xét: Công thức truy hồi của dãy số rất phức tạp, tuy nhiên nếu
đặt dãy số phụ ta sẽ đưa được về dạng tuyến tính cấp hai. Từ công thức truy
hồi của dãy, ta có
un
un−1
=
10un−1un−3 − 24u2
n−2
un−2un−3
=
10un−1
un−2
−
24un−2
un−3
.
Do vậy, ta đặt vn =
un
un−1
thì dãy (vn) được xác định như sau:



v2 = 2, v3 = 20
vn = 10vn−1 − 24vn−2, n = 4, 5, ...
Phương trình đặc trưng x2
− 10x + 24 = 0 có hai nghiệm là x1 = 6, x2 = 4
nên
vn = C1 · 6n
+ C2 · 4n
.
26
Cho n = 2, n = 3 ta được C1 =
1
6
, C2 = −
1
4
. Do đó vn = 6n−1
− 4n−1
. Vậy
un = vn · vn−1 · · · v2
= (6n−1
− 4n−1
)(6n−2
− 4n−2
) · · · (6 − 4)
= 2n−1
· 2n−2
· · · 2 · (3n−1
− 2n−1
)(3n−2
− 2n−2
) · · · (3 − 2)
= 2
(n−1)n
2 · (3n−1
− 2n−1
)(3n−2
− 2n−2
) · · · (3 − 2).
Do (3n−1
− 2n−1
)(3n−2
− 2n−2
) · · · (3 − 2) là số lể nên để un
... 2048 thì
2
(n−1)n
2
... 2048 hay 2
(n−1)n
2
... 211
. Do đó
(n − 1)n
2
≥ 11 ⇒ n ≥ 6.
Vậy n = 6 là giá trị cần tìm.
Bài tập 2.1.8. Cho dãy số nguyên (an) thỏa mãn:
(n − 1)an+1 = (n + 1)an − 2(n − 1) với mọi n ∈ N∗
.
Biết a1999
... 2000. Tìm số n nhỏ nhất sao cho an (n ≥ 2) chia hết cho 2000.
Lời giải. Chia hai vế của đẳng thức cho (n − 1)(n + 1)n ta có
an−1
n(n + 1)
=
an
(n − 1)n
−
2
n(n + 1)
. (1)
Đặt
an
(n − 1)n
= bn thì (1) có dạng
bn+1 = bn −
2
n(n + 1)
⇔ bn+1 −
2
n + 1
= bn −
2
n
với mọi n ∈ N∗
.
Do đó bn −
2
n
= b2 −
2
2
= b2 − 1 hay bn = b2 − 1 +
2
n
nên
an =
2
n
+ b2 − 1 n(n − 1) = 2(n − 1)+
a2
2
− 1 n(n − 1).
Theo giả thiết 2000 là ước của a1999 = 1998 2 + 1999
a2
2
− 1 . Tức là 1000
là ước của 2 + 1999
a2
2
− 1 do đó
a2
2
∈ Z. Hay
1000 là ước của 2 + 2000
a2
2
− 1 −
a2
2
− 1
⇔ 1000 là ước của 2−
a2
2
− 1 .
27
Vì thế
a2
2
− 1 = 1000m + 2, m ∈ Z. Suy ra
an = 2(n − 1) + (1000m + 2)n(n − 1).
Vì n(n − 1) là số chẵn nên an chia hết cho 2000 khi và chỉ khi
(n − 1) + n(n − 1) = M · 1000 ⇔ (n − 1)(n + 1) = 1000M ⇒ n = 2k + 1.
Vậy k(k + 1) = 250M. Nên số k nhỏ nhất là 124, do đó n nhỏ nhất là 249.
Bài tập 2.1.9 (HSG QG 1998). Cho dãy số nguyên dương (an), n = 0, 1, 2, ...
được xác định bởi:



a0 = 20, a1 = 100
an+2 = 4an+1 + 5an + 20 (1) với n = 0, 1, 2, ...
Tìm số nguyên dương h nhỏ nhất có tính chất an+h − an chia hết cho 1998 với
mọi n ∈ N.
Lời giải. Trước hết ta tìm số hạng tổng quát của dãy. Đặt an = bn + t với mọi
n. Thay vào (1) ta được bn+2 = 4bn+1 + 5bn + 8t + 20. Ta chọn t để
8t + 20 = 0 ⇔ t = −
5
2
.
Từ đó an = bn −
5
2
và bn+2 − 4bn+1 − 5bn = 0. Phương trình đặc trưng
x2
− 4x − 5 = 0 có hai nghiệm là 5 và −1, từ đó có công thức tổng quát
bn = c1 · 5n
+ c2 · (−1)n
.
Cho n = 0, n = 1 ta tính được c1 =
125
6
, c2 =
5
3
. Vậy
an =
125
6
· 5n
+
5
3
· (−1)n
−
5
2
. (2)
Giả sử h là số nguyên dương thỏa mãn
an+h ≡ an(mod1998) (với mọi n ∈ N). (3)
28
Từ a0 = 20, a1 = 100 suy ra h ≥ 2. Ta sẽ chứng minh h thỏa mãn (3) khi
và chỉ khi h chẵn, h ≥ 2 và
ah−1 ≡ 0(mod1998) (4)
Điều kiện cần.
ah = a0+h ≡ a0 = 20(mod1998). (5)
Và ah+1 ≡ a1 = 100(mod1998). Từ 5ah−1 = ah+1 − 4ah − 20 ≡ 0(mod
1998) suy ra ah−1 ≡ 0(mod1998) vì (5, 1998) = 1. Nếu h lẻ thì h − 1 chẵn, từ
(2) ta có
ah = 5ah−1 ≡ 0(mod1998)
mẫu thuẫn với ah ≡ 20(mod1998) theo (5). Vậy h phải chẵn, h ≥ 2 và
ah−1 ≡ 0(mod1998).
Điều kiện đủ. Giả sử h ≥ 2 và ah−1 ≡ 0(mod1998). Khi đó theo (2) có
5ah−2 = ah−1 ≡ 0(mod1998). Từ (1) có
ah = 4ah−1 + 5ah−2 + 20 ≡ 20a0(mod1998)
ah+1 = 4ah + 5ah−1 + 20 ≡ a1(mod1998).
Bằng chứng minh quy nạp dễ dàng suy ra an+h = an(mod1998). Từ (3) ⇔ (4)
có h chẵn, h ≥ 2 và ah−1 ≡ 0(mod1998). Vì h − 1 lẻ theo (2) ta có
ah−1 =
52
6
(5h
− 1) ≡ 0(mod1998)
suy ra (5h
− 1) ≡ 0(mod1998) vì (5, 1998) = 0
(4) ⇔



5h
≡ 1(mod6 · 1998)
h chẵn
1998 = 2 · 33
· 37 ⇒ 6 · 1998 = 22
· 34
· 37 nên áp dụng định lý Euler có
5h
≡ 1(mod22
) thỏa mãn với mọi h
5h
≡ 1(mod34
) ⇔ h
... 2 · 33
= 54
5h
≡ 1(mod37) ⇔ h
... 36
29
(4) ⇔ h chia hết cho [54, 36] = 108. Vậy giá trị nhỏ nhất của h thỏa mãn là
108.
Các bài toán chứng minh dãy số có vô hạn số hạng chia hết cho một số cho
trước thường được chứng minh số dư trong phép chia là hữu hạn và do đó tuần
hoàn dẫn đến có vô hạn số hạng chia hết cho số đã cho.
Bài tập 2.1.10. Dãy số (xn) (n = 0, 1, 2, ...) được xác định như sau:



x0 = a, x1 = b
xn+1 = 5x2
n − 3xn−1 với mọi n ≥ 2.
Chứng minh rằng với mọi cách chọn các số nguyên a, b tùy ý thì dãy trên hoặc
không có số hạng nào chia hết cho 1997, hoặc có vô hạn số hạng chia hết cho
1997.
Lời giải. Với mỗi i ∈ N gọi ri là số dư phép chia xi cho 1997 (0 ≤ ri < 1997). Xét
dãy các cặp số (r0, r1), (r1, r2), ...Vì tập giá trị của dãy này hữu hạn nên tồn tại
cặp số (i, j) với i, j ∈ N∗
, sao cho ri = rj, ri+1 = rj+1 hay xi ≡ xj(mod1997),
xi+1 ≡ xj+1(mod1997). Từ cách xác định xn suy ra với i, j như thế thì
xi+k ≡ xj+k(mod1997) với mọi k ∈ N. (1)
Mặt khác cũng từ công thức truy hồi suy ra 3xi−1 ≡ 3xj−1(mod1997) suy
ra xi−1 ≡ xj−1(mod1997). Tiếp tục như vậy ta được
xi−k ≡ xj−k(mod1997) với mọi k mà 0 ≤ k ≤ i. (2)
Từ (1) và (2) và xj+k = xi+k+j−i với mọi k ≥ 0 suy ra
xn ≡ xn+T (mod1997) với mọi n ∈ N, T = j − i. (3)
• Nếu tồn tại 0 ≤ r ≤ T − 1 để xr ≡ 0(mod1997) thì xn ≡ 0(mod1997)
với mọi n = mT + r (m ∈ N).
• Nếu trong tập {x0, x1, ..., xT −1} không có số nào chia hết cho 1997 thì
mọi n = mT + r với 0 ≤ r ≤ T − 1 ta có xn không chia hết cho 1997.
30
Bài toán này có thể tổng quát: Cho trước A ∈ N∗
và dãy (xn) (n = 0, 1, 2, ...)
được xác định bởi x0 = a0, ..., xm = am và thỏa mãn quan hệ
b0xn+1 = b1xk1
n + b2xk2
n−1 + · · · + bmxkm
n−m+1 + bm+1xn−m
trong đó ai ∈ Z, bi ∈ Z, ki ∈ N mà (bi, A) = 1 với i ∈ {0, 1, ..., m} và n ≥ m.
Chứng minh rằng khi đó hoặc trong dãy không có số nào chia hết cho A, hoặc
có vô hạn số chia hết cho A.
Bài tập 2.1.11. Cho dãy số (an) xác định bởi:
a1 = 824, a2 = 67
an+1 = a2
n − 3an−1, với mọi n ≥ 2.
Chứng minh rằng tồn tại vô hạn số hạng của dãy chia hết cho 2017.
Lời giải. Ta chứng minh dãy số dư của dãy (an) theo mod2017 là dãy tuần
hoàn. Thật vậy, ta nói bộ (x, y) ≡ (a, b)(mod2017) khi và chỉ khi:



x ≡ a(mod2017)
y ≡ b(mod2017).
Rõ ràng một bộ (x, y) bất kỳ chỉ có thể đồng dư với một bộ trong tập
{(i, j) : 0 ≤ i, j ≤ 2016}.
Ta xét 20172
+ 1 bộ: (a1, a2), (a2, a3), ..., (a20172+1, a20172+2). Theo nguyên lý
Dirichlet, tồn tại hai trong số những bộ này đồng dư với nhau, nghĩa là có i
khác j trong N∗
sao cho: (ai, ai+1) ≡ (aj, aj+1). Ta gọi tập hợp những bộ số
(i, j) như vậy là tập A. Ta tìm bộ số thuộc A có chỉ số i nhỏ nhất là (i0, j0), ta
chứng minh i0 = 1. Thật vậy, giả sử i0 ≥ 2, ta có
3ai0−1 = a2
i0
− ai0+1 ≡ a2
j0
− aj0+1 = 3aj0−1.
Vậy (ai0−1, ai0 ) ≡ (aj0−1, aj0 ) nên suy ra bộ (i0 − 1, j0 − 1) cũng thuộc A.
31
Điều này mâu thuẫn cách chọn bộ (i0, j0). Vậy tồn tại T ∈ N∗
sao cho



a1 ≡ a1+T (mod2011)
a2 ≡ a2+T (mod2011).
Suy ra
a3 = a2
2 − 3a1 ≡ a2
2+T − 3a1+T = a3+T
a4 = a2
3 − 3a2 ≡ a2
3+T − 3a2+T = a4+T
......................
an = a2
n−1 − 3an−2 ≡ a2
n−1+T − 3an−2+T = an+T , ∀ n ≥ 3.
Vậy ta có an ≡ an+T với mọi n ∈ N∗
. Mặt khác, ta lại có
a3 = 672
− 3 · 824 = 2017 ≡ 0(mod2017)
nên suy ra a3+kT ≡ a3 ≡ 0(mod2017). Vậy trong dãy đã cho có vô số phần tử
chia hết cho 2017.
Bài tập 2.1.12 (HSG QG 1989). Xét dãy số Fibonaxi 1, 1, 2, 3, 5, 8, ...
Đặt
f(n) = 1985n2
+ 1956n + 1960.
a) Chứng minh rằng tồn tại vô hạn số F của dãy trên sao cho f(F) chia
hết cho 1989.
b) Tồn tại hay không một số G của dãy trên sao cho f(G) + 2 chia hết cho
1989?
Lời giải. a) Đặt g(n) = 4n2
+ 33n + 29. Ta có
g(n) = 1989(n2
+ n + 1) − f(n).
Do đó f(n)
... 1989 ⇔ g(n)
... 1989. Xét dãy số sau đây:
−1, 1, 0, 1, 1, 2, ...
32
F0 = −1, F1 = 1, ..., Fn+1 = Fn + Fn−1 với mọi n ≥ 1 (thêm số −1, 1, 0 vào
trước dãy Fibonaxi).
Gọi ri là số dư của Fi khi chia cho 1989 (0 ≤ ri ≤ 1988). Xét dãy các cặp
số sau: (r0, r1), (r1, r2), (r2, r3), ...Theo nguyên lý Dirichlet trong 19892
+ 1 cặp
đầu tiên, ít nhất phải có hai cặp trùng nhau.
Giả sử (rp, rp+1) = (rp+α, rp+α+1) (p, α ∈ N). Tức là ta có rp = rp+α,
rp+1 = rp+α+1. Từ công thức Fn−1 = Fn+1 − Fn, ta suy ra rp−1 = rp+α−1,
rp−2 = rp+α−2, ..., r2 = rα+2, r1 = rα+1, r0 = rα. Từ r0 = rα, r1 = rα+1 và
Fn+1 = Fn + Fn−1, ta suy ra ri = rα với mọi i = 0, 1, 2... Do đó ta có
r0 = rα = r2α = r3α = ... = rkα, ∀ k ≥ 1.
Từ đây ta có g(Fkα) = 1989A = g(−1) = 1989A. Mặt khác Fkα với
k = 1, 2, 3 đều là các số Fibonaxi, nên ta suy ra có vô hạn số Fibonaxi F thỏa
mãn f(F) chia hết cho 1989.
b) Bây giờ ta chứng minh với mọi n nguyên thì số 4n2
+ 7n + 1 không chia
hết cho 13. Thật vậy, ta có:
16(4n2
+ 7n + 1) = (8n + 7)2
− 7 − 13 · 2.
Đặt 8n + 7 = 13t ± r (0 ≤ r ≤ 6), t và r là các số nguyên, ta có:
(8n + 7)2
= 13(13t2
± 2tr) + r2
do đó 16(4n2
+ 7n + 1) = r2
− 7 + 13m (m là số nguyên nào đó). Thử trực tiếp
các giá trị của r = 0, 1, 2, 3, 4, 5, 6 ta đều thấy r2
− 7 không chia hết cho 13. Vậy
4n2
+ 7n + 1 không chia hết cho 13 với mọi n nguyên. Mặt khác
f(n) + 2 = 1989(n2
+ n + 1) − 26(n + 1) − (4n2
+ 7n + 1).
Do 1989
... 13 nên f(n) + 2 không chia hết cho 13 với mọi n. Vì vậy không
tồn tại số Fibonaxi F nào để f(F) + 2 chia hết cho 1989.
33
Bài tập 2.1.13. Cho dãy số (bn) (n = 1, 2, ...) xác định bởi:



b1 = 0, b2 = 14, b3 = −18
bn+1 = 7bn−1 − 6bn−2, với mọi n ≥ 3.
Chứng minh rằng với mọi số nguyên tố p, ta luôn có bp chia hết cho p.
Lời giải. Ta có phương trình đặc trưng x3
− 7x + 6 = 0 có nghiệm x = 1, x = 2
và x = −3. Từ đó suy ra
bn = a · 1n
+ b · 2n
+ c · (−3)n
(1)
với mọi n ≥ 1. Lần lượt cho n = 1, 2, 3 theo giả thiết ta có



a + 2b − 3c = 0
a + 4b + 9c = 14
a + 8b − 27c = −18
⇔ a = b = c = 1.
Vậy
bn = 1 + 2n
+ (−3)n
. (2)
Với p nguyên tố, theo định lý nhỏ Fermat, ta có 2p
≡ 2(modp) và
(−3)p
≡ −3(modp). Do đó từ (2) ta có bp ≡ 0(modp) với mọi số nguyên tố p
hay bp
... p với mọi số nguyên tố p.
Bài tập 2.1.14. Cho dãy số (an) xác định bởi:



a1 = a2 = 1
an+2 = an+1 + an, n ≥ 1.
Tìm tất cả các số nguyên dương a, b với a < b thỏa mãn điều kiện an − 2nan
chia hết cho b với mọi n ≥ 1.
34
Lời giải. Ta có a3 = 2. Do an ≡ 2nan
(modb) nên



a1 ≡ 2a(modb)
a3 ≡ 6a3
(modb)
⇒



2a ≡ 1(modb)
6a3
≡ 2(modb)
⇒



2a ≡ 1(modb)
3(2a)3
≡ 8(modb)
⇒



2a ≡ 1(modb)
5 ≡ 0(modb)
⇒



2a ≡ 1(modb)
b = 5
⇒



a = 3
b = 5.
Bây giờ ta chứng minh cặp (a; b) = (3; 5) thỏa mãn điều kiện đề bài. Đặt
bn = an − 2n3n
.
Do an+2 = an+1 + an nên
bn+2 = bn+1 + bn − 10 · 3n
· (n + 3)
với mọi n ≥ 1, suy ra bn+2 = bn+1 + bn(mod5). Mà b1 = −5 ≡ 0(mod5) với
mọi n ≥ 1, b2 = −35 ≡ 0(mod5). Nên từ hệ thức trên suy ra bn ≡ 0(mod5) với
mọi n ≥ 1. Hay an − 2nan
chia hết cho 5 với mọi n ≥ 1.
Bài tập 2.1.15. Cho dãy số (an) (n = 0, 1, 2, ...) được xác định bởi:



a0 = 29, a1 = 105, a2 = 381
an+3 = 3an+2 + 2an+1 + an với mọi n = 0, 1, 2, ...
Chứng minh rằng với mỗi số nguyên dương m luôn tồn tại số tự nhiên n sao
cho các số an, an+1 − 1, an+2 − 2 đều chia hết cho m.
Lời giải. Từ công thức truy hồi ta tìm được a−1 = 8, a−2 = 2, a−3 = 1, a−4 = 0.
Giả sử an ≡ rn(modm) trong đó 0 ≤ rn < m. Xét các bộ ba (rn, rn+1, rn+2)
(n ∈ Z). Vì các bộ ba là vô hạn mà tập giá trị này là hữu hạn (không vượt quá
m3
) nên tồn tại p, q mà p < q ∈ Z sao cho (rp, rp+1, rp+2) = (rq, rq+1, rq+2).
Tức là 


rp = rq
rp+1 = rq+1
rp+2 = rq+2
⇔



ap ≡ aq(modm)
ap+1 ≡ aq+1(modm)
ap+2 ≡ aq+2(modm).
35
Từ công thức truy hồi và hệ đồng dư trên ta dễ dàng suy ra
aq+k ≡ ap+k(modm) ∀k ∈ Z ⇔ ak ≡ ap−q+k(modm) ∀k ∈ Z
⇔ ak ≡ at+k(modm), t = q − p ∈ N∗
.
Từ đó ak ≡ aet+k(modm) với mọi e ∈ N∗
. Nói riêng



alt−4 ≡ a−4 ≡ 0(modm)
alt−3 ≡ a−3 ≡ 1(modm)
alt−2 ≡ a−2 ≡ 2(modm).
Cho n đủ lớn ta có lt − 4 ∈ N. Do đó tồn tại n = lt − 4 ∈ N để an ≡ 0(modm),
an+1 ≡ 1(modm) và an+2 ≡ 2(modm).
2.2. Tính chất số nguyên
Các bài toán chứng minh dãy số gồm toàn các số nguyên được đưa về
công thức truy hồi tuyến tính sau đó chứng minh bằng phương pháp quy nạp
với một vài số hạng đầu là số nguyên.
Bài tập 2.2.1. Cho dãy số (an) được xác định bởi:



a1 = 2
an+1 = 3an + 4 + 8a2
n
Chứng minh rằng dãy số gồm toàn các số nguyên.
Lời giải. Từ giả thiết, ta có a2 = 12, an+1 > 3an và
(an+1 − 3an)2
= 4 + 8a2
n, an > 0 với mọi n.
Suy ra
a2
n+1 − 6an+1an + a2
n = 4. (1)
36
Thay n bởi n + 1 ta được
a2
n+2 − 6an+2an+1 + a2
n+1 = 4. (2)
Trừ hai vế của (1) và (2) ta được:
a2
n+2 − a2
n − 6an+1(an+2 − an) = 0 ⇔ (an+2 − an)(an+2 − 6an+1 + an) = 0.
Vì an+2 > an nên
an+2 − 6an+1 + an = 0 (∀ n ∈ N∗
). (3)
Vì a1, a2 ∈ Z và do (3) nên an ∈ Z với mọi n ∈ N∗
. (Chứng minh bằng
phương pháp quy nạp)
Bài tập 2.2.2. Cho ba số nguyên a, b, c thỏa mãn điều kiện a2
= b + 1. Dãy số
(un) được xác định như sau:



u0 = 0
un+1 = aun + bu2
n + c2, n = 0, 1, 2, ...
Chứng minh rằng mọi số hạng của dãy số trên đều là số nguyên.
Lời giải. Từ giả thiết ta có un+1 − aun = bu2
n + c2, n = 0, 1, 2, ...Suy ra
u2
n+1 − 2aunun+1 + a2
u2
n = bu2
n + c2
. (1)
Với giả thiết a2
= b + 1 thì (1) suy ra
(a2
− b)u2
n+1 − 2aunun+1 + a2
u2
n = (a2
− 1)u2
n + c2
hay
u2
n + 2aunun+1 + a2
u2
n+1 = bu2
n+1 + c2
. (2)
Từ giả thiết ta có bu2
n+1 + c2
= (un+2 − aun+1)2
nên (2) suy ra
(un − aun+1)2
= (un+2 − aun+1)2
. (3)
37
Do un+2 − aun+1 ≥ 0 nên (3) suy ra
un+2 − aun+1 = |un − an+1| ⇒ un+2 = aun+1 + |un − an+1|. (4)
Do u0 = 0 nên suy ra u1 = au0 + bu2
0 + c2 =
√
c2 = |c|. Do đó u0, u1 ∈ Z.
Vậy từ (4) suy ra un ∈ Z với mọi n ∈ N.
Nhận xét. Ta cũng có thể giải bài toán này bằng cách khác như sau: Ta có
u2
n+1 − 2aunun+1 + u2
n(a2
− b) − c2
= 0
hay
u2
n+1 − 2aunun+1 + u2
n − c2
= 0. (5)
Trong (5) thay n bởi n + 1 ta có
u2
n+2 − 2aun+1un+2 + u2
n+1 − c2
= 0. (6)
Trừ từng vế của (6) cho (5) ta được
u2
n+2 − u2
n − 2aun+1un+2 + 2aunun+1 = 0
hay
(un+2 − un)(un+2 + un − 2aun+1) = 0. (7)
Từ (7) suy ra un+2 = un hoặc un+2 = 2aun+1 − un. Từ đó do u0, u1 ∈ Z
nên un ∈ Z với mọi n = 1, 2, ...
Từ bài toán này có thể cho nhiều bài toán với các giá trị a, b, c cụ thể.
Chẳng hạn, chứng minh rằng mọi số hạng của các dãy số sau đều là số nguyên.
1)



u0 = 0
un+1 = 5un + 24u2
n + 9
2)



u0 = 0
un+1 = 4un + 15u2
n + 36
Ta cũng có thể dựa vào cách chứng minh để đưa ra các bài toán sau:
38
3)



u0 = 1
un+1 = 5un + 24u2
n + 25
4)



u0 = 2
un+1 = 3un + 8u2
n + 9
Bài tập 2.2.3. Tìm số nguyên dương k sao cho dãy số sau gồm toàn các số
nguyên:



a1 = 1
an+1 = 5an + ka2
n − 8 với mọi n = 1, 2, 3, ... (1)
Lời giải. Từ giả thiết ta có a2 = 5 +
√
k − 8. Đặt
√
k − 8 = t (t ∈ N) khi đó
a2 = 5 + t, theo (1) ta có
a3 = 5(5 + t) + (t2 + 8)(5 + t)2 − 8.
Để a3 ∈ Z thì ta phải có f(t) = (t2
+ 8)(5 + t)2
− 8 = q2
(q ∈ N). Ta có
f(t) = t4
+ 10t3
+ 33t2
+ 80t + 192.
Do (t2
+5t+4)2
< f(t) < (t2
+5t+14)2
và f(t)
... 2 nên suy ra q = t2
+5t+v
(5 ≤ v ≤ 13, v ∈ N). Từ đó v ∈ {6, 8, 10, 12}. Thử trực tiếp ta được v = 8 và
q = t2
+ 5t + 8 ⇒ t = 4 ⇒ k = 24.
Ngược lại với k = 24 thì
an+1 = 5an + 24a2
n − 8 ⇒ a2
n+1 − 10anan+1 + a2
n + 8 = 0. (2)
Thay n bởi n + 1 ta có
a2
n+2 − 10an+1an+2 + a2
n+1 + 8 = 0. (3)
39
Trừ từng vế (2) và (3) ta được (an − an+2)(an+2 + an − 10an+1) = 0. Do
(an) là dãy tăng thực sự nên an < an+2. Do đó an+2 = 10an+1 − an, trong đó
a1 = 1, a2 = 9. Vậy dãy (an) gồm toàn các số nguyên khi và chỉ khi k = 24.
Bài tập 2.2.4 (Olympic Bungari 1978). Cho dãy số (an):



a1, a2 ∈ Z;
a2
1 + a2
2 + a
a1a2
∈ Z
an+2 =
a2
n+1 + a
an
, với mọi n ∈ N∗
, an = 0
(a là số cho trước).
Chứng minh rằng dãy số gồm toàn các số nguyên.
Lời giải. Từ giả thiết ta có anan+2 = a2
n+1 + a với mọi n, suy ra
an+1an−1 = a2
n + a
với mọi n ≥ 2. Trừ hai đẳng thức này vế theo vế ta có
anan+2 + a2
n = an+1an−1 + a2
n+1.
Hay
an+2 + an
an+1
=
an+1 + an−1
an
⇒
an+1 + an−1
an
= t (hằng số với mọi n ≥ 2).
Hay an+1 = tan − an−1. Mặt khác
a3 =
a2
2 + a
a1
⇒ t =
a3 + a1
a2
=
a2
1 + a2
2 + a
a1a2
∈ Z.
Vì a1, a2, t ∈ Z suy ra an ∈ Z với mọi n ∈ N∗
(Chứng minh bằng quy nạp).
Bài tập 2.2.5. Xét các dãy số (an), n ≥ 1 xác định bởi:



a1 = a2 = 1, a3 = 199
an+1 =
1989 + anan−1
an−2
, (n ≥ 3).
Chứng minh tất cả các số hạng của dãy đều là số nguyên dương.
Lời giải. Giả thiết suy ra
an+1an−2 = 1989 + anan−1.
40
Thay n bởi n − 1 ta được
anan−3 = 1989 + an−1an−2.
Trừ hai đẳng thức trên vế theo vế ta được
an+1an−2 − anan−1 = anan−3 − an−1an−2
suy ra
an+1 + an−1
an
=
an−1 + an−3
an−2
với n ≥ 4.
• Nếu n chẵn
an+1 + an−1
an
=
an−1 + an−3
an−2
= ... =
a3 + a1
a2
= 200.
• Nếu n lẻ
an+1 + an−1
an
=
an−1 + an−3
an−2
= ... =
a4 + a2
a3
= 11.
Suy ra an+1 =



200an − an−1 nếu n chẵn
11an − an−1 nếu n lẻ.
Từ công thức trên, bằng phương pháp chứng minh quy nạp với a1, a2, a3 là
các số nguyên dương suy ra an+1 là số nguyên dương với mọi n ≥ 1.
Bài tập 2.2.6. Cho dãy (un) được xác định bởi:



u1 = u2 = 0
un+1 =
u2
n + 10un − 5un−1 + 50
un−1 + 5
(n ≥ 2).
Chứng minh rằng un là số nguyên chia hết cho 5 với mọi n = 0, 1, 2, ...
Lời giải.
un+1 =
u2
n + 10un − 5un−1 + 50
un−1 + 5
⇔ un+1 + 5 =
(un + 5)2
+ 50
un−1 + 5
⇔
1
5
un+1 + 1 =
1
5
un + 1
2
+ 2
1
5
un−1 + 1
.
41
Đặt xn =
1
5
un + 1, dãy đã cho được viết lại



x1 = x2 = 1
xn+1 =
x2
n + 2
xn−1
, (n ≥ 2).
Ta chứng minh dãy (xn) nguyên.
+/ Với n = 2, x3 = 3 ∈ Z.
+/ Giả sử mệnh đề đúng với mọi n = k ≥ 2, tức là xk+1 ∈ Z. Ta chứng
minh xk+2 ∈ Z. Ta có
xk+1xk−1 = x2
k + 2, xk+2xk = x2
k+1 + 2 ⇒ xk+2xk − xk+1xk−1 = x2
k+1 − x2
k
⇒
xk
xk+1 + xk−1
=
xk+1
xk+2 + xk
.
Tương tự, ta được:
xk
xk+1 + xk−1
=
xk+1
xk+2 + xk
= ... =
x2
x3 + x1
=
1
4
. Từ đó
suy ra
xk+1
xk+2 + xk
=
1
4
⇒ xk+2 = 4xk+1 − xk.
Từ giả thiết quy nạp xk, xk+1 ∈ Z nên xk+2 ∈ Z, ta có điều phải chứng
minh. Vậy ta có
1
5
un ∈ Z nên un là số nguyên chia hết cho 5.
Bài tập 2.2.7. Giả sử dãy số nguyên (an) thỏa mãn a4 = 4 và
1
a1a2a3
+
1
a2a3a4
+ · · · +
1
anan+1an+2
=
(n + 3)an
4an+1an+2
với mọi n nguyên, n ≥ 2. Chứng minh rằng an = n với mọi n nguyên dương.
Lời giải. Ta viết lại hệ thức truy hồi dưới dạng
(n + 2)an−1
4anan+1
+
1
anan+1an+2
=
(n + 3)an
4an+1an+2
⇔ (n + 2)an+2 =
(n + 3)a2
n − 4
an−1
với mọi n ≥ 3. Cho n = 2 vào đẳng thức ban đầu, ta được 4(a1 +4) = 5a1a2
2 suy
ra rằng a1 là ước của 16 và 5 là ước của a1 + 4. Từ đó suy ra a1 = 16, a2 = 1
hoặc a1 = 1, a2 = 2.
Trường hợp 1. Nếu a1 = 16, a2 = 1 thì 5a5 = 6a2
3 − 4, a3a6 = 18 và 7a7 =
2a2
5 − 1 khi cho n = 3, 4, 5 tương ứng. Vì a3 ≡ ±2(mod5) và a3 là ước số của
18 nên a3 = 3 hoặc a3 = 18. Kiểm tra trực tiếp cho thấy cả hai đều không phải
42
là nghiệm của bài toán.
Trường hợp 2. Nếu a1 = 1, a2 = 2. Khi đó thay n = 3, 4 ta được 5a5 +2 = 3a2
3,
a3a6 = 18. Một lần nữa ta lại có a3 ≡ 2(mod5) và a3 là ước của 18 nên suy ra
hai trường hượp a3 = 3, a6 = 6 hoặc a3 = 18, a6 = 1. Trong trường hợp thứ hai
ta có a5 = 194, mâu thuẫn với đẳng thức 8a8 =
9a2
6 − 4
a5
. Trường hợp thứ hai ta
suy ra a5 = 5 và như vậy ai = i với mọi i = 1, 2, 3, 4, 5, 6. Bây giờ bằng phương
pháp quy nạp dễ dàng chứng minh rằng an = n với mọi n nguyên dương.
Bình luận. Bài này là đề thi chọn đội tuyển Bulgaria năm 2006. Đây là dạng
toán về dãy số nguyên. Có thể phát biểu bài toán dưới dạng khác: Tìm tất cả
các giá trị a1, a2, a3 để dãy số cho bởi công thức truy hồi ban đầu và điều kiện
a4 = 4 chứa gồm toàn các số nguyên.
Dưới đây là một số bài toán tương tự.
1. (VMO 1996) Hãy xác định tất cả các hàm số f : N∗
→ N∗
thỏa mãn
đẳng thức:
f(n) + f(n + 1) = f(n + 2)f(n + 3) − 1996
với mọi n ∈ N∗
.
2. (VMO 1997) Hỏi có bao nhiêu hàm số f : N∗
→ N∗
thỏa mãn điều kiện
sau:
f(n)f(n + 2) = [f(n + 1)]2
+ 1997
với mọi n ∈ N∗
.
Các bài toán tiếp theo sẽ liên quan đến việc biểu diễn các số hạng của dãy
số theo tổng lũy thừa các số nguyên. Phương pháp thường dùng là sử dụng định
lý Fermat để chỉ ra các phần dư.
Bài tập 2.2.8. Cho dãy số (xn) xác định bởi:



x1 = 5, x2 = 7
xn+1 = x3
n + 6xn−1 + 3 · 22008
.
43
Chứng minh rằng xn không thể biểu diễn được dưới dạng tổng của các lũy thừa
bậc 6 của ba số nguyên dương.
Lời giải. Xét A = a6
+ b6
+ c6
. Theo định lý Fermat
x13
≡ x(mod13) ⇔ x(x6
− 1)(x6
+ 1) ≡ 0(mod13)
⇔





x6
≡ 0 (mod13)
x6
≡ 1 (mod13)
x6
≡ −1 (mod13)
Vậy bộ thặng dư của A mod 13 là: S = {0, ±1, ±2, ±3}. Ta có
212
≡ 1(mod13) ⇒ 22004
≡ 1(mod13) (do 2004
... 12).
Nên 22008
= 24
· 22004
≡ 24
(mod13) ≡ 3(mod13) ⇒ 3 · 22008
≡ 9(mod13).
Nên xn+1 ≡ x3
n + 6xn−1 + 9(mod13). Ta có x3 ≡ 382 ≡ 5(mod13), x4 ≡ 176 ≡
7(mod13) nên số dư của xn khi chia cho 13 tuần hoàn với chu kỳ 2. Từ đó suy
ra xn ≡ xn−2(mod13). Do đó



x2n ≡ x2 ≡ 7(mod13)
x2n+1 ≡ x1 ≡ 5(mod13)
⇒


xn ≡ 5 (mod13)
xn ≡ 7 (mod13)
Mà 5, 7 /∈ S. Vậy xn không thể biểu diễn được dưới dạng tổng của lũy thừa
bậc 6 của ba số nguyên dương.
Bài tập 2.2.9. Cho dãy (an), (n = 0, 1, 2, ...) được xác định bởi:



a0 = 2
an+1 = 4an + 15a2
n − 60.
Hãy xác định số hạng tổng quát của an, chứng minh rằng số
1
5
(a2n + 8) có thể
biểu diễn thành tổng bình phương của ba số nguyên liên tiếp với mọi n ≥ 1.
Lời giải. Theo bài ta có
a2
n+1 − 8anan+1 + a2
n + 60 = 0. (1)
44
Thay n bởi n − 1 ta được
a2
n − 8an−1an + a2
n−1 + 60 = 0. (2)
Trừ theo từng vế (1) cho (2) được
a2
n+1 − a2
n−1 + 8an−1an − 8anan+1 = 0
hay
(an+1 − an)(an+1 − 8an + an−1) = 0.
Từ giả thiết, ta có an+1 > 4an > 16an−1 suy ra an+1 − an > 0. Do đó
an+1 − 8an + an−1 = 0. Ta có phương trình đặc trưng t2
− 8t + 1 = 0 có hai
nghiệm là t1 = 4 +
√
15, t2 = 4 −
√
15. Từ đó
an = (4 +
√
15)n
+ (4 −
√
15)n
.
Bây giờ ta chứng minh
1
5
(a2n + 8) có thể biểu diễn thành tổng của ba số
nguyên liên tiếp với mọi n ≥ 1. Thật vậy, với mỗi n ≥ 1, thì tồn tại k ∈ N để:
(4 +
√
15)n
− (4 −
√
15)n
=
√
15k ⇒ [(4 +
√
15)n
− (4 −
√
15)n
]2
= 15k2
hay (4 +
√
15)2n
+ (4 −
√
15)2n
= 15k2
+ 2. Nên
1
5
(a2n + 8) =
1
5
[(4 +
√
15)2n
+ (4 −
√
15)2n
+ 8]
= 3k2
+ 2 = (k − 1)2
+ k2
+ (k + 1)2
.
Vậy ta có điều phải chứng minh.
Bài tập 2.2.10 (IMO 2005). Xét dãy số a1, a2, ... được xác định như sau:
an = 2n
+ 3n
+ 6n
− 1, (n = 1, 2, ...).
Tìm tất cả các số nguyên dương nguyên tố cùng nhau với mọi số hạng của dãy
số trên.
Lời giải. Ta khẳng định số 1 là số cần tìm duy nhất. Ta chỉ cần chứng minh
45
rằng mỗi số nguyên tố p là ước của một số hạng an nào đó. Khẳng định này
đúng với p = 2, p = 3 vì a2 = 48.
Bây giờ, ta xét số nguyên tố p > 3 bất kỳ. Theo định lý Fermat nhỏ, số dư
của phép chia 2p−1
, 3p−1
, 6p−1
trong phép chia cho p đều là 1. Suy ra số dư
trong phép chia của tổng 3 · 2p−1
+ 2 · 3p−1
+ 6p−1
trong phép chia cho p là
3 + 2 + 1 = 6. Do đó, tổng
3 · 2p−1
+ 2 · 3p−1
+ 6p−1
= 6 · 2p−2
+ 6 · 3p−2
+ 6 · 6p−2
có số dư 6 trong phép chia cho p. Suy ra ap−2 = 2p−2
+ 3p−2
+ 6p−2
− 1 chia
hết cho p. Ta có điều phải chứng minh.
2.3. Tính chính phương
Với tính chất này ta thường tìm số hạng tổng quát của dãy số, đưa biểu
thức cần chứng minh về bình phương đủ của một số nguyên. Với một số bài
toán tổng quát ta có thể đặc biệt hóa để có bài toán mới, ngược lại với một bài
toán cụ thể ta có thể tổng quát hóa để được một dạng toán.
Bài tập 2.3.1. Cho dãy số (an):



a0 = 0, a1 = 1 (1)
an+1 = 2an − an−1 + 1 (2)
Chứng minh rằng 4an+2an + 1 là số chính phương (n ≥ 1).
Lời giải. Cách 1. Xét phương trình đặc trưng λ2
−2λ+1 = 0 ⇔ λ = 1 (nghiệm
kép). Ta tìm g(n) = an2
sao cho g(n+1)−2g(n)+g(n−1) = 1 với mọi n ∈ N∗
.
Giải ra ta có g(n) =
n2
2
hay a∗
n =
n2
2
là một nghiệm riêng của phương trình
(2). Do đó (2) có nghiệm tổng quát là:
an = C1 + nC2 +
n2
2
.
46
Theo (1), a0 = 0 suy ra C1 = 0, a1 = 1 nên C2 +
1
2
= 1 ⇒ C2 =
1
2
. Vậy
an =
1
2
· n +
n2
2
=
n(n + 1)
2
.
Do đó
4an+2an + 1 = 4 ·
(n + 2)(n + 3)
2
·
n(n + 1)
2
+ 1
= n(n + 1)(n + 2)(n + 3) + 1
= (n2
+ 3n)(n2
+ 3n + 2) + 1 = (n2
+ 3n + 1)2
(đpcm).
Cách 2. Từ công thức truy hồi của dãy ta thay n + 1 bởi n ta được
an = 2an−1 − an−2 + 1. (3)
Trừ vế theo vế đẳng thức (3) và (2) ta được
an−1 − 3an + 3an−1 − an−2 = 0.
Xét phương trình đặc trưng
λ3
− 3λ2
+ 3λ − 1 = 0 ⇔ λ = 1.
Suy ra an = α+βn+γn2
. Do a0 = 0, a1 = 1, a2 = 3 nên ta tìm được α = 0,
β = γ =
1
2
⇒ an =
n(n + 1)
2
. Do đó
4an+2an + 1 = 4 ·
(n + 2)(n + 3)
2
·
n(n + 1)
2
+ 1
= n(n + 1)(n + 2)(n + 3) + 1
= (n2
+ 3n)(n2
+ 3n + 2) + 1 = (n2
+ 3n + 1)2
(đpcm).
Nhận xét. Ta có thể tìm số hạng tổng quát mà không cần phương pháp sai
phân, cách làm này sẽ gần gũi hơn với chương trình học phổ thông ban cơ bản.
Đặt bn = an+1 − an. Từ giả thiết ta có an+1 − an = an − an−1 + 1. Do đó
47
bn = bn−1 + 1. Từ đó tìm được bn = 1 + n (do (bn) là cấp số cộng với công sai
bằng 1, b0 = 1). Ta có
an =
n−1
k=0
(ak+1 − ak) + a0 =
n−1
k=0
bk = n +
n−1
k=0
k = n +
n(n − 1)
2
=
n(n + 1)
2
.
Bài tập 2.3.2. Cho dãy số (an) xác định bởi:



a1 = 3
n(2n − 1)an+1 = (2n + 1)(n + 1)an − (2n − 1)(2n + 1) với mọi n ≥ 1.
Chứng minh rằng an + 1 là số chính phương với mọi n = 1, 2, 3, ...
Lời giải. Nhận xét: Dãy số được cho bởi công thức truy hồi khá phức tạp, song
nếu dùng phương pháp đổi sang dãy số phụ thì việc tìm số hạng tổng quát sẽ
dễ dàng. Từ giả thiết, ta có
2n(2n − 1)an+1 = (2n + 1)(2n + 2)an − 2(2n − 1)(2n + 1).
Chia hai vế đẳng thức cho (2n − 1)2n(2n + 1)(2n + 2) ta được
an+1
(2n + 1)(2n + 2)
=
an
(2n − 1)2n
−
2
2n(2n + 2)
.
Đặt
an
(2n − 1)2n
= bn thì ta có
bn+1 = bn −
2
2n(2n + 2)
⇔ bn+1 −
1
2n + 2
= bn −
1
2n
, ∀ n ∈ N∗
.
Do đó
bn −
1
2n
= bn−1 −
1
2(n − 1)
= ... = b1 −
1
2
.
Suy ra bn =
1
2n
+ b1 −
1
2
. Từ đó
an = 2n − 1 + (2n − 1)2n
a1
2
−
1
2
= 4n2
− 1.
Vậy an + 1 = 4n2
là số chính phương.
Nhận xét: Thực chất bài toán này chỉ là tìm số hạng tổng quát dựa vào
48
dãy số phụ, việc chứng minh số chính phương là khá rõ ràng.
Nếu chọn các giá trị khác nhau của a1 ta được các bài toán khác. Chẳng
hạn
• a1 = 1 thì an = 2n − 1. Bài toán có thể yêu cầu chứng minh dãy gồm
toàn các số tự nhiên lẻ.
• a1 = 2 thì an = (2n − 1)(n + 1). Bài toán có thể yêu cầu chứng minh an
chẵn khi n lẻ và an lẻ khi n chẵn.
Bài tập 2.3.3. Cho dãy số nguyên (an) (n = 0, 1, 2, ...) thỏa mãn:
an+2 + an−1 = 2(an+1 + an) với n = 1, 2, ...
Chứng tỏ rằng tồn tại số nguyên M không phụ thuộc n sao cho M + 4an+1an
là số chính phương với mọi n ∈ N.
Lời giải. Đặt un = an+2 − an+1 − an với mọi n ≥ 0. Từ giả thiết suy ra
un = un−1 + 2an ⇒ u2
n = (un−1 + 2an)2
= u2
n−1 + 4un−1an + 4a2
n
= u2
n−1 + 4(an+1 − an − an−1)an + 4a2
n
= u2
n−1 + 4an+1an − 4anan−1.
Suy ra u2
n − 4an+1an = u2
n−1 − 4anan−1 với mọi n ≥ 1. Từ đó
u2
n − 4an+1an = (a2 − a1 − a0)2
− 4a1a0
là hằng số. Gọi hằng số đó là M. Khi đó M + 4an+1an = u2
n với mọi n ≥ 0.
Nhận xét. Ta có thể cho a0, a1, a2 các giá trị cụ thể để được các bài toán
mới. Chẳng hạn:
1) Cho dãy số nguyên (an) thỏa mãn:



a0 = 0, a1 = 1, a2 = 1
an+2 + an−1 = 2(an+1 + an).
Chứng minh rằng an+1an là số chính phương.
49
2) Cho dãy số nguyên (an) thỏa mãn:



a0 = 0, a1 = 1, a2 = 5
an+2 + an−1 = 2(an+1 + an).
Chứng minh rằng 5 + 4an+1an là số chính phương.
Bài tập 2.3.4. Cho dãy số (an) (n = 1, 2, ...) được xác định bởi:



a1 = 1, a2 = 3
an+1 = (n + 2)an − (n + 1)an−1 với mọi n ≥ 2.
Tìm các giá trị của n để an là số chính phương.
Lời giải. Ta sẽ giải bài toán tổng quát: Cho số nguyên p ≥ 2. Cho dãy số (an)
(n = 1, 2, ...) được xác định bởi a1 = 1, a2 = 3,
an+1 = (n + 2)an − (n + 1)an−1
với mọi n ≥ 2. Hãy xác định các giá trị của n để an là lũy thừa p của số tự
nhiên.
Lời giải. Với mỗi n ≥ 2, đặt bn = an −an−1. Khi đó từ công thức xác định dãy
(an), ta có bn = n · bn−1 với mọi n ≥ 3. Kết hợp với b2 = a2 − a1 = 3 − 1 = 2!.
Ta được bn = n! với mọi n ≥ 2. Ta có
an =
n
k=2
(ak − ak−1) + a1 =
n
k=2
bk + 1 =
n
k=1
k!.
Kết hợp với a1 = 1 = 1!. Ta được
an =
n
k=1
k! với mọi n ≥ 1. (1)
Xét p = 2. Từ (1) ta có an ≡ 3(mod10) với mọi n ≥ 5 nên an khác a2
với
mọi n ≥ 5 (vì các số chính phương chỉ có thể có tận cùng bởi 0, 1, 4, 5, 6, 9).
Với n = 1, 2, 3, 4 bằng cách thử trực tiếp ta thấy an là số chính phương khi và
50
chỉ khi n = 1, n = 3.
Xét p > 2. Khi đó an ≡ 0(mod3) với mọi n ≥ 2 (suy ra từ (1)) nên điều
kiện cần để tồn tại a ∈ N sao cho an = ap
là an ≡ 0(mod27) hoặc an = 1.
Từ (1) ta có an > 1 với mọi n ≥ 2 và an = a8 +
n
k=9
k! với mọi n ≥ 9.
Suy ra an ≡ a8(mod27) với mọi n ≥ 0. Mà a8 = 46233 ≡ 1(mod27) nên
an ≡ 1(mod27) với mọi n ≥ 8. Như vậy với mọi n ≥ 8 đều không tồn tại a ∈ N
sao cho an = ap
.
Với 1 ≤ n ≤ 7 bằng cách thử trực tiếp ta thấy chỉ có duy nhất giá trị n = 1
là giá trị cần tìm.
Nhận xét. Bài đã ra là trường hợp đặc biệt của bài toán khái quát khi
p = 2. Theo đó tất cả các giá trị n thỏa mãn là n = 1, n = 3.
Bài tập 2.3.5 (HSG QG bảng B 1998). Cho các số nguyên a, b. Xét dãy số
nguyên (an), (n = 0, 1, 2, ...) được xác định bởi:



a0 = a, a1 = b, a2 = 2b − a + 2
an+3 = 3an+2 − 3an+1 + an với n = 0, 1, 2, ...
a) Tìm số hạng tổng quát an.
b) Tìm tất cả các số nguyên a, b để an là số chính phương với mọi n ≥ 1998.
Lời giải. a) Từ điều kiện đề bài ta có
an+3 − an+2 = 2(an+2 − an+1) − (an+1 − an). (1)
Đặt an+1 − an = bn với mọi n ≥ 0 thì (1) trở thành
bn+2 = 2bn+1 − bn ⇒ bn+2 − bn+1 = bn+1 − bn = ... = b1 − b0
= a2 − 2a1 + a0 = 2
suy ra bn+1 = bn + 2 với mọi n. Vậy công thức tổng quát của bn là
bn = b0 + 2n = 2n + b − a.
51
Từ đó
an − a0 =
n−1
k=0
(ak+1 − ak) =
n−1
k=0
bk = 2
n−1
k=0
k + n(b − a)
= n(n − 1) + n(b − a).
Vậy an = n2
+ n(b − a − 1) + a.
b) Giả sử an = n2
+ n(b − a − 1) + a = u2
(n ≥ 1998, u > 0). Ta có
4u2
= 4n2
+ 4n(b − a − 1) + 4a = [2n + (b − a − 1)]2
+ 4a − (b − a − 1)2
.
Đặt v = 2n + b − a − 1 và d = 4a − (b − a − 1)2
, ta có
d = 4u2
− v2
= (2u + v)(2u − v).
Với n đủ lớn thì v = 2n + b − a − 1 > 0.
Nếu d khác 0 thì 2u − v khác 0 và
|d| = |(2u − v)(2u + v)| ≥ |2u + v| ≥ 2u + v ≥ v = 2n + b − a − 1.
Vì d là hằng số nên điều này không thể xảy ra khi n đủ lớn. Vậy d = 0. Suy
ra 4a = (b − a − 1)2
. Đặt b − a − 1 = 2t thì a = t2
và b = a + 1 + 2t = (t + 1)2
.
Đảo lại nếu a = t2
và b = (t + 1)2
thì
an = n2
+ n(b − a − 1) + a = n2
+ 2tn + t2
= (n + t)2
.
Kết luận an = u2
⇔ a = t2
và b = (t + 1)2
.
Ta có thể tìm số hạng tổng quát bằng phương pháp sai phân nhờ việc xét
phương trình đặc trưng:
λ3
− 3λ2
+ 3λ − 1 = 0.
Từ kết quả bài toán ta cũng có thể cho t các giá trị khác nhau để được bài
toán mới. Chẳng hạn: Xét dãy số nguyên (an), (n = 0, 1, 2, ...) được xác định
52
bởi: 


a0 = 4, a1 = 9, a2 = 16
an+3 = 3an+2 − 3an+1 + an với n = 0, 1, 2, ...
Chứng minh rằng an là số chính phương với mọi n ≥ 1998.
Bài tập 2.3.6 (chọn đội tuyển QG 2006). Cho dãy số (an) được xác định
bởi:
a0 = 1, an+1 =
1
2
an +
1
3an
với mọi n = 1, 2, 3, ...
Chứng minh rằng với mọi số nguyên n, số An =
3
3a2
n − 1
là một số chính phương.
Lời giải. Trước hết ta sẽ chứng minh rằng
3An+1 = 4An(An + 3) với mọi n nguyên dương. (*)
Thật vậy, ta có An + 3 =
3
3a2
n − 1
+ 3 =
9a2
n
3a2
n − 1
nên
4An(An + 3) = 4An ·
9a2
n
3a2
n − 1
=
108a2
n
(3a2
n − 1)2
.
Mặt khác 3An+1 =
9
3a2
n+1 − 1
=
9
3 ·
1
22
an +
1
3an
2
− 1
=
108a2
n
(3a2
n − 1)2
. Do
đó (∗) được chứng minh, tức là 3An+1 = 4An(An + 3) với mọi n nguyên dương.
Hơn nữa, ta tính được A1 = 9 nên dễ dàng thấy An chia hết cho 3 với mọi n.
Tiếp theo, ta chứng minh bằng quy nạp rằng
An
3
+ 1 là số chính phương với mọi n. (**)
+ Với n = 1 thì
An
3
+ 1 = 4 là số chính phương nên (**) đúng.
+ Giả sử (**) đúng với n = k, tức là tồn tại số tự nhiên p sao cho
Ak
3
+ 1 = p2
. Ta có
Ak+1
3
+ 1 =
4Ak(Ak + 3)
9
+ 1 = 4 ·
Ak
3
· 1 +
Ak
3
= 4p2
(p2
− 1) + 1 = (2p2
− 1)2
53
cũng là một số chính phương nên (**) cũng đúng với n = k + 1. Do đó (**)
được chứng minh. Từ
3An+1 = 4An(An + 3) ⇒ An+1 = 4An
An
3
+ 1
và (**) ta dễ dàng chứng minh bằng quy nạp An là số chính phương với mọi n.
Bài tập 2.3.7. Dãy số (un) được xác định bởi:



u0 = 9, u1 = 161
un = 18un−1 − un−2 với n = 2, 3, ...
Chứng minh rằng
u2
n − 1
5
là số chính phương với mọi số tự nhiên n.
Lời giải. Nhận xét rằng mọi số hạng của dãy đều là số nguyên.
Với n = 0 ta có
u2
0 − 1
5
=
92
− 1
5
= 16 = 42
.
Với n = 1 ta có
u2
1 − 1
5
=
1612
− 1
5
= 5184 = 722
.
Vậy bài toán đúng với n = 0, 1. Xét với n ≥ 2 ta có
un = 18un−1 − un−2 ⇔ un − 9un−1 = 9un−1 − un−2
⇔ (un − 9un−1)2
= (9un−1 − un−2)2
hay
u2
n − 18unun−1 = u2
n−2 − 18un−1un−2. (1)
Lần lượt thay n = 2, 3, ... vào (1) ta được



u2
2 − 18u2u1 = u2
0 − 18u1u0
u2
3 − 18u3u2 = u2
1 − 18u2u1
....................................
u2
n − 18unun−1 = u2
n−2 − 18un−1un−2
Cộng theo vế các đẳng thức trên ta thu được
u2
n + u2
n−1 − 18unun−1 = u2
1 + u2
0 − 18u1u0. (2)
54
Thay u0 = 9, u1 = 161 vào (2) ta được u2
n + u2
n−1 − 18unun−1 = −80 hay
(9un − un−1)2
80
= u2
n − 1.
Do các số hạng của dãy đều nguyên nên (9un − un−1)2
... 42
· 5 nên
(9un − un−1)
... 4 · 5. Vì thế 9un − un−1 = 20a (a ∈ Z). Suy ra
u2
n − 1
5
= a2
(đpcm).
Cách khác. Sử dụng phương trình đặc trưng t2
− 18t + 1 = 0 để tìm số
hạng tổng quát
un =
1
2
[(9 +
√
80)n+1
+ (9 −
√
80)n+1
].
Từ đó suy ra
u2
n − 1
5
=
1
20
[(9 +
√
80)n+1
− (9 −
√
80)n+1
]2
. Tiếp tục sử dụng
khai triển Newton ta thu được điều phải chứng minh.
Bài tập 2.3.8. Cho dãy số nguyên dương (xn) xác định như sau:



x0 = 2, x1 = 16
xn = 6xn−1 − xn−2 + 4, n = 2, 3, ...
Chứng minh rằng
1
2
x2
n + xn là số chính phương với mọi n = 0, 1, 2, ...
Lời giải. Đặt xn = un − 1. Ta có



u0 = 3, u1 = 17
un = 6un−1 − un−2, n = 2, 3, ...
Ta chứng minh u2
n − 1 chia hết cho 2 và thương số là số chính phương với mọi
n = 0, 1, 2, .... Ta có u2
0 − 1 = 8, u2
1 − 1 = 288. Rõ ràng kết luận đúng khi n = 0,
n = 1. Xét với n ≥ 2. Ta có un − 3un−1 = 3un−1 − un−2. Bình phương hai vế
đẳng thức trên ta có
u2
n + 9u2
n−1 − 6unun−1 = 9u2
n−1 − 6un−1un−2 + u2
n−2
hay
u2
n − 6unun−1 = −6un−1un−2 + u2
n−2 ∀ n ≥ 2. (1)
55
Lần lượt thay n = 2, 3, ..., k ta có



u2
2 − 6u2u1 = −6u1u0 = u2
0
u2
3 − 6u3u2 = −6u2u1 + u2
1
............................
u2
k − 6ukuk−1 = −6uk−1uk−2 + u2
k−2.
Cộng từng vế k − 1 đẳng thức trên ta có
u2
k + u2
k−1 − 6ukuk−1 = −6u1u0 + u2
1 + u2
0. (2)
Do u0 = 3, u1 = 17 nên từ (2) ta có
u2
k + u2
k−1 − 6ukuk−1 = −8 ⇒ 8(u2
k − 1) = (3uk − uk−1)2
⇒ u2
k − 1 =
(3uk − uk−1)2
8
, k = 2, 3, ...
(3)
Do u0, u1 ∈ Z và un = 6un−1 − un−2 suy ra uk ∈ Z với mọi k = 0, 1, 2, ... Vì
vậy từ (3) suy ra (3uk −uk−1)2
... 8 suy ra 3uk −uk−1
... 4. Suy ra 3uk −uk−1 = 4ak
(với ak ∈ Z). Thay lại vào (3) suy ra u2
k − 1 = 2a2
k với ak ∈ Z. Vì ak ∈ Z nên
suy ra điều phải chứng minh.
Bài tập 2.3.9. Cho dãy số (un) xác định như sau:
u1 = 1995, u2 = 1996, un+1 = un(un − 1) + 2 với mọi n = 2, 3, ....
Chứng minh rằng (u2
1 + 1)(u2
2 + 1) · · · (u2
1996 + 1) − 1 là số chính phương.
Lời giải. Đặt Sk = (u2
1 + 1)(u2
2 + 1) · · · (u2
k + 1) − 1. Ta chứng minh
Sk = (uk+1 − 1)2
. (1)
Thật vậy với k = 1 ta có
S1 = (u2
1 + 1) − 1 = 19952
= (1996 − 1)2
= (u2 − 1)2
.
Vậy (1) đúng với k = 1. Giả sử (1) đúng với n = k, tức là
Sn = (un+1 − 1)2
. (2)
56
Ta có
Sn+1 = (u2
1 + 1)(u2
2 + 1) · · · (u2
n+1 + 1) − 1 = (Sn + 1)(u2
n+1 + 1) − 1. (3)
Do giả thiết quy nạp ta có
Sn+1 = [(un+1 − 1)2
+ 1](u2
n+1 + 1) − 1
= [(u2
n+1 + 1) − 2un+1 + 1](u2
n+1 + 1) − 1
= (u2
n+1 + 1)2
− 2un+1(u2
n+1 + 1) + u2
n+1
= (u2
n+1 + 1 − un+1)2
= (un+2 − 1)2
.
(4)
Từ (4) suy ra (1) đúng khi k = n + 1. Vậy (1) đúng với mọi k. Do đó
S1996 = (u2
1 + 1)(u2
2 + 1) · · · (u2
1996 + 1) − 1 = (u1997 − 1)2
. (5)
Vì u1, u2 nguyên nên un+1 = un(un − 1) + 2 nguyên với mọi n. Do đó S1996
là số chính phương.
2.4. Bài tập
Bài tập 2.4.1. Cho dãy số (an) xác định bởi
a1 = a2 = 12
√
2, an+1 = −an a2
n−1 + 1 + an−1 a2
n + 1 với mọi n ≥ 2.
Chứng minh rằng 2 + 2 a2
n + 1 là số nguyên dương với mọi n ≥ 1.
Bài tập 2.4.2. Chứng minh rằng mọi số hạng của dãy số {an} xác định bởi:
a0 = 1, an+1 = 2an + 3a2
n − 2
đều nguyên.
Bài tập 2.4.3. Dãy số (un) được xác định bởi:



u0 = 2, u1 = 5
un+1un−1 − u2
n = 6n−1
với mọi n = 1, 2, 3, ...
Chứng minh rằng mọi số hạng của dãy đều là các số nguyên.
57
Bài tập 2.4.4. Cho dãy số (xn) được xác định bởi:



x0 = 1
xn+1 =
3xn + 5x2
n − 4
2
với mọi n = 0, 1, 2, ...
Chứng minh rằng mọi số hạng của dãy đều là số nguyên.
Bài tập 2.4.5. Dãy số không âm (un), n = 0, 1, 2, ... thỏa mãn điều kiện sau:



u1 = 1
um+n + um−n =
1
2
(u2m + u2n) với mọi m ≥ n; m, n ∈ N
Chứng minh rằng mọi phần tử của dãy là số chính phương.
Bài tập 2.4.6. Dãy số (un) xác định bởi:



u1 = 1, u2 = 3
un = (n + 1)un−1 − nun−2 với mọi n = 3, 4, 5, ...
Tìm các số hạng của dãy chia hết cho 11.
Bài tập 2.4.7. Cho p, q là hai số nguyên dương cho trước. Dãy số (un) được
xác định bởi: 


u1 = p + q − 1
un+1 = p + q −
pq
un
n = 1, 2, ...
Đặt Pn = u1u2 · · · un. Chứng minh rằng với mọi n = 1, 2, ... thì Pn là số nguyên.
Bài tập 2.4.8. Cho dãy số (un) được xác định bởi
u0 = 3, u1 = 11, un+2 = 2un+1 + 7un với n = 0, 1, 2, ...
Tìm các số nguyên dương lẻ a sao cho với mọi số nguyên dương m, n tùy ý luôn
tìm được số nguyên dương k thỏa mãn uk
n − a chia hết cho 2m
.
58
Bài tập 2.4.9. Cho dãy số (un) thỏa mãn:
un+2 =
unun+1
2un − un+1
(n = 1, 2, ...)
Hãy tìm điều kiện cần và đủ đối với u1, u2 để trong dãy đã cho có vô hạn số
hạng là số nguyên.
Bài tập 2.4.10. Cho dãy số (un) xác định như sau:
u1 = 20, u2 = 100 và un+1 = 4un + un−1 − 1976 n = 2, 3, ...
Chứng minh rằng tồn tại ít nhất một số của dãy số chia hết cho 1996.
59
Chương 3
Giới hạn của dãy số
3.1. Giới hạn của tổng
Các bài toán về tìm giới hạn của tổng ta thu gọn tổng đó bằng cách
phân tích hạng tử tổng quát thành hiệu các hạng tử nối tiếp nhau để các hạng
tử có thể triệt tiêu, cuối cùng đưa tổng đó về biểu thức chỉ còn chứa xn, sau đó
tìm lim xn.
Bài tập 3.1.1. Cho dãy số (xn) (n = 1, 2, ...) được xác định như sau:
x1 = 1 và xn+1 = xn(xn + 1)(xn + 2)(xn + 3) + 1 với n = 1, 2, ...
Đặt yn =
n
i=1
1
xi + 2
, (n = 1, 2, ...). Tìm lim
n→+∞
yn.
Lời giải. Ta có x2 = 5 và xn > 0 với mọi n = 1, 2, ...
xn+1 = xn(xn + 1)(xn + 2)(xn + 3) + 1
= (x2
n + 3xn)(x2
n + 3xn + 2) + 1
= x2
n + 3xn + 1.
(1)
60
Từ đó suy ra
xn+1 + 1 = x2
n + 3x2
n + 2 = (xn + 1)(xn + 2)
⇒
1
xn+1 + 1
=
1
(xn + 1)(xn + 2)
=
1
xn + 1
−
1
xn + 2
⇒
1
xn + 2
=
1
xn + 1
−
1
xn+1 + 1
.
Do đó
yn =
n
i=1
1
xi + 2
=
n
i=1
1
xi + 1
−
1
xi+1 + 1
=
1
x1 + 1
−
1
xn+1 + 1
=
1
2
−
1
xn+1 + 1
.
Từ (1) xk+1 = x2
k + 3xk + 1 > 3xk ≥ 3 · 3k−1
= 3k
. Ta dễ dàng chứng minh
bằng quy nạp rằng
xn > 3n−1
. (2)
Nên lim
n→+∞
yn =
1
2
(vì do (2) xn+1 > 3n
). .
Ta có thể chứng minh lim xn = +∞ với cách khác: Dễ thấy (xn) là dãy
tăng, giả sử rằng lim xn = a (a ≥ 1). Nên ta có
a = a(a + 1)(a + 2)(a + 3) + 1.
Suy ra a2
= a(a + 1)(a + 2)(a + 3) + 1 hay a4
+ 6a3
+ 10a2
+ 6a + 1 = 0. Rõ
ràng phương trình này không có nghiệm nào thỏa mãn a ≥ 1. Vậy lim xn = +∞.
Bài tập 3.1.2 (HSG QG năm 2009). Cho dãy số (xn), n = 1, 2, ... xác định
bởi: 


x1 =
1
2
xn =
x2
n−1 + 4xn−1 + xn−1
2
, (n = 2, 3, ...)
Chứng minh rằng dãy (yn) (n = 1, 2, ...) với yn =
n
i=1
1
x2
i
có giới hạn hữu hạn
và tìm giới hạn đó.
61
Lời giải. Từ giả thiết ta có xn > 0 với mọi n ≥ 1. Ta có
xn − xn−1 =
x2
n−1 + 4xn−1 + xn−1
2
− xn−1 =
x2
n−1 + 4xn−1 − xn−1
2
> 0
với mọi n ≥ 2. Do đó dãy (xn) tăng. Giả sử lim xn = a thì a > 0 và
a =
√
a2 + 4a + a
2
⇔ a = 0 (vô lý).
Vậy lim xn = +∞.
Từ xn =
x2
n−1 + 4xn−1 + xn−1
2
với mọi n ≥ 2 suy ra
x2
n = (xn + 1)xn−1 ⇒
1
x2
n
=
1
xn−1
−
1
xn
với mọi n ≥ 2.
Do đó
yn =
n
i=1
1
x2
i
=
1
x2
1
+
1
x1
−
1
x2
+
1
x2
−
1
x3
+ · · · +
1
xn−1
−
1
xn
=
1
x2
1
+
1
x1
−
1
xn
= 6 −
1
xn
với mọi n ≥ 2.
Suy ra yn < 6 với mọi n ≥ 1 và dãy (yn) tăng vì yn = yn−1 +
1
xn
> yn−1.
Vậy (yn) có giới hạn hữu hạn và
lim
n→+∞
yn = lim
n→+∞
6 −
1
xn
= 6.
Bài tập 3.1.3. Xét dãy số (xn) (n = 1, 2, 3, ...) xác định bởi:
x1 = 2 và xn+1 =
1
2
(x2
n + 1) với mọi n = 1, 2, 3, ...
Đặt Sn =
1
1 + x1
+
1
1 + x2
+ · · · +
1
1 + xn
. Tìm lim
n→+∞
Sn.
Lời giải. Ta có thể tổng quát hóa bài toán như sau: Cho dãy (un) thỏa mãn



u1 = a
un+1 =
u2
n + (b + c)un + c2
b − c
.
62
Ta chứng minh Sn =
n
i=1
1
ui + b
=
1
u1 + c
−
1
un+1 + c
. Thật vậy ta có
un+1 =
u2
n + (b + c)un + c2
b − c
⇒ un+1 + c =
u2
n + (b + c)un + bc
b − c
=
(un + b)(un + c)
b − c
.
Từ đó
1
un+1 + c
=
1
un + c
−
1
un + b
⇒
1
un + b
=
1
un + c
−
1
un+1 + c
.
Khai triển và ước lượng được
1
u1 + b
=
1
u1 + c
−
1
u2 + c
1
u2 + b
=
1
u2 + c
−
1
u3 + c
............................
1
un + b
=
1
un + c
−
1
un+1 + c
.
Do đó Sn =
1
u1 + c
−
1
un+1 + c
. Từ đó vận dụng vào bài toán trên với b = 1,
c = −1 ta có
Sn =
1
x1 − 1
−
1
xn+1 − 1
= 1 −
1
xn+1 − 1
.
Mà xn+1 − xn =
1
2
(xn − 1)2
> 0 với mọi n ∈ N∗
nên dãy (xn) là dãy tăng.
Giả sử lim
n→+∞
xn = a (a > 2) thì 2a = a2
+ 1 ⇒ a = 1 (vô lý). Vậy
lim
n→+∞
xn = +∞. Do đó lim
n→+∞
Sn = 1.
Bài tập 3.1.4. Cho dãy số (xn) được xác định bởi:
x1 = 1, xn+1 =
(2xn + 1)2012
2012
+ xn
với n là số nguyên dương. Đặt
un =
(2x1 + 1)2011
2x2 + 1
+
(2x2 + 1)2011
2x3 + 1
+
(2x3 + 1)2011
2x4 + 1
+ · · · +
(2xn + 1)2011
2xn+1 + 1
.
63
Tính lim un.
Lời giải. Ta có xn+1 − xn =
(2xn + 1)2012
2012
với mọi n ≥ 1. Suy ra
1
2xn + 1
−
1
2xn+1 + 1
=
2(xn+1 − xn)
(2xn + 1)(2xn+1 + 1)
=
(2xn + 1)2011
1006(2xn+1 + 1)
⇒
n
i=1
(2xi + 1)2011
2xi+1 + 1
= 1006
n
i=1
1
2xi + 1
−
1
2xi+1 + 1
= 1006
1
2x1 + 1
−
1
2xn+1 + 1
.
Mặt khác xn+1 − xn ≥ 0 nên (xn) là dãy số tăng với mọi n ≥ 1. Nếu (xn)
bị chặn thì lim xn tồn tại. Đặt lim xn = a ⇒ a ≥ 1 và
a =
(2a + 1)2012
2012
+ a (vô lý).
Suy ra (xn) không bị chặn, hay lim xn = +∞, suy ra
lim
n→+∞
1
2xn+1 + 1
= 0.
Vậy
lim
n→+∞
un =
1006
3
.
Bài tập 3.1.5. Cho dãy số (xn) với n = 1, 2, ... được xác định bởi:



x1 = a, (a > 1), x2 = 1
xn+2 = xn − ln xn (n ∈ N∗
).
Đặt Sn =
n−1
k=1
(n − k) ln
√
x2k−1 (n ≥ 2). Tìm lim
n→+∞
Sn
n
.
Lời giải. Nhận xét rằng x2n = 1, n = 1, 2, ...do ln 1 = 0 suy ra lim
n→+∞
x2n = 1.
Tiếp theo, ta chứng minh dãy (x2n+1) cũng có giới hạn là 1. Xét hàm số
f(x) = x − ln x
liên tục và đồng biến trong (1; +∞) vì f (x) = 1 −
1
x
> 0 với mọi x > 1.
Trước hết ta chứng minh bằng phương pháp quy nạp, dãy (x2n+1) bị chặn
64
dưới bởi 1. Theo giả thiết thì x1 = a > 1, giả sử x2k+1 > 1 thì
f(x2k+1) > f(1) > 1
nên hiển nhiên x2k+3 > 1 tức dãy (x2n+1) bị chặn dưới bởi 1.
Tiếp theo ta chứng minh dãy (x2n+1) là dãy giảm. Thật vậy do x2n+1 > 1
nên ln x2n+1 > 0. Vì thế x2n+3 − x2n+1 = − ln x2n+1 < 0, tức là dãy (x2n+1) là
dãy giảm.
Từ đó suy ra (x2n+1) có giới hạn c = lim
n→+∞
x2n+1. Chuyển qua giới hạn dãy
số, ta được
c = c − ln c ⇔ ln c = 0 ⇔ c = 1.
Vậy dãy số (xn) có giới hạn là 1. Theo định lý Cessaro, ta có
lim
n→+∞
x1 + x2 + · · · + x2n
2n
= 1
hay
lim
n→+∞
(x1 + x3 + · · · + x2n+1) + (x2 + x4 + · · · + x2n)
2n
= 1
⇔ lim
n→+∞
nx1 − (n − 1) ln x1 − (n − 2) ln x3 − · · · − ln x2n−3 + n
2n
= 1
⇔ lim
n→+∞
a
2
−
Sn
n
+
1
2
= 1 hay lim
n→+∞
Sn
n
=
a − 1
2
.
3.2. Dãy con và sự hội tụ của dãy số
Khi khảo sát sự hội tụ của dãy số ta thường sử dụng các định lý về tính
đơn điệu và bị chặn, nếu dãy không đơn điệu thì xét dãy với chỉ số chẵn, chỉ số
lẻ. Tuy nhiên có những dãy số phức tạp, tăng giảm bất thường, trong trường
hợp như thể ta thường xây dựng các dãy số phụ đơn điệu, chứng minh các dãy
số phụ có giới hạn, sau đó chứng minh dãy số ban đầu có cùng giới hạn, các dãy
số phụ phải được xây dựng từ dãy số chính.
65
Nhận xét. Mọi dãy con của dãy hội tụ đều hội tụ và ngược lại nếu
lim x2n = lim x2n+1 = a
thì lim xn = a.
Một cách tổng quát ta có: Cho số nguyên m ≥ 2 nếu lim xmn+i = a với mọi
i = 0, 1, 2, ..., m − 1 thì lim xn = a.
Bài tập 3.2.1. Dãy số (xn) được xác định bởi công thức:



x0 = x1 = 1
5xn+2 = xn + 2xn+1.
Chứng minh rằng dãy (xn) hội tụ.
Lời giải. Xét dãy số (an) được xác định bởi: a0 = 1, an+1 =
2an
3
, dễ thấy (an)
giảm dần về 0. Ta chứng tỏ rằng
max{x2n, x2n+1} ≤ an, với mọi n. (1)
Thật vậy, (1) đúng với n = 0 và n = 1. Giả sử (1) đúng với n và do (an) là
dãy giảm nên
5x2n+2 = x2n + 2x2n+1 ≤ 3an ⇒ x2n+2 ≤ an+1.
Và 5x2n+3 = x2n+1 + 2x2n+2 ≤ an + 2an+1 ≤ 3an ⇒ x2n+3 ≤ an+1. Như
vậy (1) đúng với n + 1 hay (1) đúng với mọi n = 0, 1, 2, ... Dễ thấy xn > 0 với
mọi n và từ (1) theo nguyên lý kẹp ta có
lim x2n = lim x2n+1 = 0 ⇒ lim xn = 0.
Nhận xét: Việc đưa vào dãy phụ (an) có tác dụng chặn cả hai dãy con
(x2n) và (x2n+1) và làm chúng cùng hội tụ về một điểm.
Có thể sử dụng phương pháp sai phân tìm được số hạng tổng quát
xn = C1
1 −
√
6
5
n
+ C2
1 +
√
6
5
n
.
Thay các giá trị của x0, x1 để tìm C1, C2 từ đó tìm được lim xn = 0.
66
Bài tập 3.2.2. Dãy (xn) được xác định bởi:



x0, x1, x2 ∈ (0; 1)
3xn+3 = x2
n + x2
n+2.
Chứng minh rằng dãy (xn) hội tụ.
Lời giải. Ta xét dãy số (an) xác định bởi:
a0 = max{x0, x1, x2}, an+1 =
2a2
n
3
.
Dễ thấy dãy số (an) giảm dần về 0. Ta chứng tỏ rằng
max{x3n, x3n+1, x3n+2} ≤ an với mọi n. (1)
Thật vậy (1) đúng với n = 0, 1, 2, ... Giả sử (1) đúng với n và do (an) là dãy
giảm nên ta có
3x3n+3 = x2
3n + x2
3n+2 ≤ 2a2
n ⇒ x3n+3 ≤ an+1
3x3n+4 = x2
3n+1 + x2
3n+3 ≤ a2
n + a2
n+1 ≤ 2a2
n ⇒ x3n+4 ≤ an+1
3x3n+5 = x2
3n+2 + x2
3n+4 ≤ a2
n + a2
n+1 ≤ 2a2
n ⇒ x3n+5 ≤ an+1.
Như vậy (1) đúng với n + 1, theo nguyên lý quy nạp, (1) được chứng minh.
Dễ thấy xn > 0. Từ đó theo nguyên lý kẹp giữa ta có lim x3n+i = 0 (i = 0, 1, 2)
do đó lim xn = 0.
Từ các cách chọn dãy số phụ như trên ta có các dãy số sau đều hội tụ
về 0 với x0, x1, x2, x3 đều thuộc (0; 1)
3xn+3 = x2
n + xn+1xn+2, 3xn+3 = x2
n + xnxn+1,
3xn+3 =
x2
n + x2
n+2
2
+ x2
n+1, 6xn+4 = xn+1xn+2 + x2
n + 2xnxn+1.
67
Bài tập 3.2.3. Dãy (xn) được xác định bởi:



x0, x1, x2 > 0
xn+3 =
√
xn + xn+2.
Chứng minh rằng dãy (xn) hội tụ.
Lời giải. Ta xây dựng hai dãy (an) và (bn) như sau:



a0 = max{x0, x1, x2, 2}
an+1 =
√
2an



b0 = min{x0, x1, x2, 2}
bn+1 =
√
2bn.
Dãy (an) là dãy giảm dần về 2, dãy (bn) tăng dần về 2. Bằng quy nạp dễ
dàng chứng minh được
bn+1 ≤ min{x3n, x3n+1, x3n+2} ≤ max{x3n, x3n+1, x3n+2} ≤ an với mọi n.
Từ đó dẫn đến lim x3n = lim x3n+1 = lim x3n+2 = 2 suy ra lim xn = 2.
Bài tập 3.2.4. Cho dãy (xn) (n = 0, 1, 2, ...) được xác định như sau:



x0, x1, x2 là các số dương cho trước
xn+2 =
√
xn+1 +
√
xn +
√
xn−1 với mọi n ≥ 1.
Chứng minh rằng dãy (xn) hội tụ và tìm giới hạn của dãy.
Lời giải. Ta xây dựng hai dãy (an) và (bn) như sau:



a0 = max{x0, x1, x2, 9}
an+1 = 3
√
an, n = 0, 1, 2, ...



b0 = min{x0, x1, x2, 9}
bn+1 = 3
√
bn, n = 0, 1, 2, ...
Dãy (an) là dãy giảm dần về 9, dãy (bn) tăng dần về 9 suy ra
lim
n→+∞
an = lim
n→+∞
bn = 9.
Ta chứng minh
bn+1 ≤ min{x3n, x3n+1, x3n+2} ≤ max{x3n, x3n+1, x3n+2} ≤ an với mọi n.
(1)
68
Thật vậy với n = 0 thì (1) hiển nhiên đúng. Giả sử (1) đúng với n = k, khi
đó với n = k + 1 ta có
bn ≤ bn+1 = 3 bn ≤ x3k+3 =
√
x3k+2 +
√
x3k+1 +
√
x3k ≤
≤ 3
√
an = an+1 ≤ an
bn ≤ bn+1 = 3 bn ≤ x3k+4 =
√
x3k+3 +
√
x3k+2 +
√
x3k+1 ≤
≤ 3
√
an = an+1 ≤ an
bn ≤ bn+1 = 3 bn ≤ x3k+5 =
√
x3k+4 +
√
x3k+3 +
√
x3k+2 ≤
≤ 3
√
an = an+1 ≤ an.
Vậy (1) cũng đúng với n = k + 1. Theo nguyên lý quy nạp thì (1) đúng với
mọi số tụ nhiên n. Từ đó theo nguyên lý kẹp ta có
lim
n→+∞
x3n = lim
n→+∞
x3n+1 = lim
n→+∞
x3n+2 = lim
n→+∞
an
= lim
n→+∞
bn = 9 ⇒ lim
n→+∞
xn = 9.
Dưới đây là một số bài toán tìm giới hạn dãy số dạng xn+1 = f(xn) (dãy số
xác định như vậy gọi là cho dưới dạng lặp). Đây là dạng toán thường gặp nhất
trong các bài toán về tìm giới hạn dãy số, dãy số hoàn toàn được xác định khi
biết f và giá trị ban đầu x0. Do vậy sự hội tụ của dãy số phụ thuộc vào tính
chất của f(x) và x0. Một đặc điểm quan trọng khác của dãy số dạng này là nếu
a là giới hạn của dãy số thì a là nghiệm của phương trình x = f(x).
Bài tập 3.2.5. Cho dãy số (xn) được xác định như sau:
x1 = 0, xn+1 =
1
27
xn
với mọi n ∈ N∗
.
Chứng minh rằng dãy số (xn) có giới hạn và tìm giới hạn đó.
Lời giải. Nhận xét rằng xn ≥ 0 với mọi n ∈ N∗
. Xét hàm số
f(x) =
1
27
x
nghịch biến trong khoảng [0; +∞). Khi đó xn+1 = f(xn) với mọi
n ∈ N∗
và f(x) ≤ f(0) nên 0 ≤ xn ≤ 1. Ta có x1 = 0, x2 = 1, x3 =
1
27
nên
69
x1 ≤ x3 và
x4 = f(x3) ≤ f(x1) = x2
Bây giờ ta chứng minh bằng phương pháp quy nạp x2n−1 ≤ x2n+1 và
x2n+2 ≤ x2n với n ∈ N∗
.
Thật vậy giả sử có x2n−1 ≤ x2n+1 thì f(x2n−1) ≥ f(x2n+1) nên x2n ≥ x2n+2
và vì vậy f(x2n) ≤ f(x2n+2) suy ra x2n+1 ≤ x2n+3.
Tương tự, giả sử có x2n ≥ x2n+2 thì f(x2n) ≤ f(x2n+2) suy ra
x2n+1 ≤ x2n+3. Vì vậy f(x2n+1) ≥ f(x2n+3) suy ra x2n+2 ≥ x2n+4.
Vậy dãy (x2n−1) là dãy tăng và dãy (x2n) là dãy giảm và đều thuộc [0; 1]
nên có giới hạn hữu hạn:
lim
n→+∞
x2n = a; lim
n→+∞
x2n−1 = b.
Và a = lim
n→+∞
x2n+2 = lim
n→+∞
f(x2n+1) = lim
n→+∞
f(f(x2n)) = f(f(a)) nên
a =
1
27
( 1
27 )a
⇒ a =
1
3
.
Tương tự ta cũng tìm được b =
1
3
. Vậy a = b =
1
3
nên lim
n→+∞
xn =
1
3
.
Bài tập 3.2.6 (HSG QG 2008). Cho dãy số thực (xn) xác định như sau:
x1 = 0, x2 = 2 và xn+2 = 2−xn
+
1
2
với mọi n = 1, 2, 3, ...
Chứng minh rằng dãy số (xn) có giới hạn hữu hạn và tìm giới hạn đó.
Lời giải. Xét hàm số f(x) = 2−x
+
1
2
xác định trên R. Với mỗi n ∈ N∗
, ta có
xn+4 = f(xn+2) = f(f(xn))
hay xn+4 = g(xn), trong đó g là hàm số xác định trên R và
g(x) = f(f(x)) với mọi x ∈ R. (1)
Dễ thấy hàm số f giảm trên R, do đó hàm số g tăng trên R. Vì thế từ (1)
suy ra với mỗi k ∈ {1; 2; 3; 4}, dãy (x4n+k), n ∈ N là dãy đơn điệu. Hơn nữa,
70
từ cách xác định dãy (xn) dễ thấy 0 ≤ xn ≤ 2 với mọi n ∈ N∗
. Do đó với mỗi
k ∈ {1; 2; 3; 4}, dãy (x4n+k) là dãy hội tụ.
Với mỗi k ∈ {1; 2; 3; 4}, đặt lim
n→+∞
x4n+k = ak ta có 0 ≤ ak ≤ 2. Hơn nữa do
hàm số g liên tục trên R nên từ (1) suy ra
g(ak) = ak. (2)
Xét hàm số h(x) = g(x) − x trên [0; 2]. Ta có
h (x) = 2−(f(x)+x)
· (ln 2)2
− 1 < 0
với mọi x ∈ [0; 2] (do f(x)+x > 0 với mọi x ∈ [0; 2]). Suy ra, hàm số h giảm trên
[0; 2]. Vì thế có nhiều nhất một điểm x ∈ [0; 2] sao cho h(x) = 0 hay g(x) = x.
Mà g(1) = 1 nên từ (2) ta được ak = 1 với mọi k ∈ {1; 2; 3; 4}.
Từ đây, vì dãy (xn) là hợp của bốn dãy con (x4n+k) nên dãy (xn) hội tụ và
lim
n→+∞
xn = 1.
Bài tập 3.2.7. Cho dãy số thực (xn) xác định bởi:



x1 = 2007
xn+1 =
√
3 +
xn
x2
n − 1
với mọi n ≥ 1.
1/ Chứng minh rằng dãy (xn) bị chặn.
2/ Chứng minh rằng dãy (xn) có giới hạn và tìm giới hạn đó.
Lời giải. Hiển nhiên xn >
√
3.
xn+1 =
√
3 +
xn
x2
n − 1
=
√
3 + 1 +
1
x2
n − 1
<
√
3 +
√
2 với mọi n ≥ 1.
Vậy xn ≤ 2007 với mọi n, suy ra dãy (xn) bị chặn. Hàm
f(x) =
√
3 +
x
√
x2 − 1
=
√
3 + 1 +
1
x2 − 1
71
nghịch biến trên [
√
3; +∞) nên chứng minh được các dãy con (x2n) và (x2n+1)
là đơn điệu. Theo 1/ các dãy đó bị chặn nên có
lim x2n = a, lim x2n+1 = b.
Từ xn+1 =
√
3 +
xn
x2
n − 1
qua giới hạn, ta có



a =
√
3 +
b
√
b2 − 1
b =
√
3 +
a
√
a2 − 1
⇒ a +
a
√
a2 − 1
= b +
b
√
b2 − 1
. (*)
Xét g(x) = x +
x
√
x2 − 1
có g (x) = 1 −
1
(x2 − 1)
√
x2 − 1
> 0 với mọi x ≥
√
3
nên g(x) đồng biến. Nên từ (∗) suy ra a = b hay
lim x2n = lim x2n+1 ⇒ lim xn = a = b.
Lúc đó a là nghiệm của phương trình
x =
√
3 +
x
√
x2 − 1
⇔ x =
√
3 +
√
15
2
⇒ lim xn =
√
3 +
√
15
2
.
Bài tập 3.2.8. Cho dãy số (xn) thỏa mãn:



x1 =
√
a, a > 2
xn+1 = a −
√
a + xn với mọi n ∈ N∗
.
Chứng minh rằng dãy (xn) có giới hạn hữu hạn.
Lời giải. Bằng quy nạp ta chứng minh được rằng 0 ≤ xn ≤
√
a với mọi n ∈ N∗
.
Xét hàm f(x) = a −
√
a + x, với mọi x ∈ [0;
√
a], có xn+1 = f(xn) và
f (x) = −
1
4 a −
√
a + x
√
a + x
< 0, với mọi x ∈ [0;
√
a].
Suy ra f(x) là hàm nghịch biến.
Do đó dãy (xn) được tách thành hai dãy con (x2n) và (x2n+1), trong đó một
72
dãy tăng và một dãy giảm, mặt khác lại có dãy (xn) bị chặn nên tồn tại
lim x2n = α, lim x2n+1 = β, trong đó α, β là nghiệm của phương trình
f(f(x)) = x ⇔ a − a + a −
√
a + x = x.
Xét hàm F(x) = a − a + a −
√
a + x − x, với x ∈ [0;
√
a]. Ta có
F (x) =
1
a − a + a −
√
a + x a + a −
√
a + x a −
√
a + x
√
a + x
−1.
Với x ∈ [0;
√
a], ta có
a −
√
a + x ·
√
a + x ≥ a − a +
√
a ·
√
a > a − a +
√
a +
1
4
·
√
a =
= a −
√
a −
1
2
·
√
a =
√
a −
1
2
2
−
3
4
·
√
2 >
√
2 −
1
2
2
−
3
4
·
√
2 >
> 0, 12 > 0, 3.
Thay vai trò của x bởi a −
√
a + x chứng minh tương tự ta có
a − a + a −
√
a + x · a + a −
√
a + x > 0, 3.
Suy ra F (x) < −0, 9 < 0 nên F(x) là hàm nghịch biến, lại có F(0) > 0,
F(
√
a) < 0 nên phương trình F(x) = 0 có nghiệm duy nhất. Do đó α = β. Suy
ra
lim x2n = lim x2n+1 = lim xn.
Suy ra lim xn = T với T thỏa mãn f(f(T)) = T.
3.3. Dãy số xác định bởi phương trình
Dãy số có mối quan hệ chặt chẽ với phương trình điều này thấy rõ qua
hai nội dung cơ bản là phương trình sai phân tuyến tính được giải bằng phương
73
trình đặc trưng, giới hạn của dãy số cũng thường được giải ra từ phương trình.
Đây là một trong các nội dung quan trọng nhất của phần dãy số.
Với dạng toán tìm giới hạn của dãy số có liên quan đến phương trình ta
thường xét tính đơn điệu của hàm số, áp dụng định lý Lagrange và định lý về
giới hạn kẹp giữa.
Bài tập 3.3.1. Giả sử xn ∈ (0; 1) là nghiệm của phương trình
1
x
+
1
x − 1
+ · · · +
1
x − n
= 0.
Chứng minh rằng dãy (xn) hội tụ. Tìm giới hạn đó.
Nhận xét. xn được xác định duy nhất vì hàm số
fn(x) =
1
x
+
1
x − 1
+ · · · +
1
x − n
liên tục và đơn điệu trên (0; 1). Tuy nhiên, ta không thể xác định được giá trị
cụ thể của xn. Rất may mắn, để chứng minh tính hội tụ của xn, ta không cần
đến điều đó. Chỉ cần chứng minh tính đơn điệu và bị chặn là đủ. Với tính bị
chặn là hiển nhiên vì 0 < xn < 1. Với tính đơn điệu, ta chú ý một chút đến mối
liên hệ giữa fn(x) và fn+1(x):
fn+1(x) = fn(x) +
1
x − n − 1
.
Đây chính là chìa khóa để chứng minh tính đơn điệu của xn.
Lời giải. xn được xác định duy nhất vì hàm số
fn(x) =
1
x
+
1
x − 1
+ · · · +
1
x − n
liên tục và đơn điệu trên (0; 1).
Để chứng minh dãy hội tụ, ta chứng minh dãy (xn) bị chặn và đơn điệu.
Hiển nhiên dãy bị chặn vì 0 < xn < 1. Bây giờ, ta chứng minh dãy (xn) đơn
điệu. Ta thấy 0 < xn < 1 nên
fn+1(xn) = fn(xn) +
1
xn − n − 1
=
1
xn − n − 1
< 0.
74
Luận văn: Một số bài toán về dãy số, HAY, 9đ
Luận văn: Một số bài toán về dãy số, HAY, 9đ
Luận văn: Một số bài toán về dãy số, HAY, 9đ
Luận văn: Một số bài toán về dãy số, HAY, 9đ
Luận văn: Một số bài toán về dãy số, HAY, 9đ
Luận văn: Một số bài toán về dãy số, HAY, 9đ
Luận văn: Một số bài toán về dãy số, HAY, 9đ
Luận văn: Một số bài toán về dãy số, HAY, 9đ
Luận văn: Một số bài toán về dãy số, HAY, 9đ
Luận văn: Một số bài toán về dãy số, HAY, 9đ
Luận văn: Một số bài toán về dãy số, HAY, 9đ

More Related Content

What's hot

Topo daicuong1[1]
Topo daicuong1[1]Topo daicuong1[1]
Topo daicuong1[1]
Bui Loi
 
Cân bằng hệ số trong bđt AM-GM
Cân bằng hệ số trong bđt AM-GMCân bằng hệ số trong bđt AM-GM
Cân bằng hệ số trong bđt AM-GMNguyễn Việt Long
 
Phương pháp giải phương trình lượng giác
Phương pháp giải phương trình lượng giácPhương pháp giải phương trình lượng giác
Phương pháp giải phương trình lượng giác
Duy Anh Nguyễn
 
Kỹ thuật nhân liên hợp
Kỹ thuật nhân liên hợpKỹ thuật nhân liên hợp
Kỹ thuật nhân liên hợp
tuituhoc
 
đại số tuyến tính 2 ( không gian eculid )
đại số tuyến tính 2 ( không gian eculid )đại số tuyến tính 2 ( không gian eculid )
đại số tuyến tính 2 ( không gian eculid )
Bui Loi
 
74774655 chuỗi-ham
74774655 chuỗi-ham74774655 chuỗi-ham
74774655 chuỗi-ham
Vinh Phan
 
Bài Giảng Đại Số Tuyến Tính - ĐH Thăng Long
Bài Giảng Đại Số Tuyến Tính - ĐH Thăng LongBài Giảng Đại Số Tuyến Tính - ĐH Thăng Long
Bài Giảng Đại Số Tuyến Tính - ĐH Thăng Long
Hoàng Như Mộc Miên
 
Bài toán số học liên quan tới lũy thữa
Bài toán số học liên quan tới lũy thữaBài toán số học liên quan tới lũy thữa
Bài toán số học liên quan tới lũy thữaThế Giới Tinh Hoa
 
Toan a1 -_bai_giang
Toan a1 -_bai_giangToan a1 -_bai_giang
Toan a1 -_bai_giangxuanhoa88
 
Gt khong gian_metric Nguyen Hoang
Gt khong gian_metric Nguyen HoangGt khong gian_metric Nguyen Hoang
Gt khong gian_metric Nguyen Hoang
Bui Loi
 
Luận văn: Giải bài toán Dirichlet đối với phương trình Elliptic, 9đ
Luận văn: Giải bài toán Dirichlet đối với phương trình Elliptic, 9đLuận văn: Giải bài toán Dirichlet đối với phương trình Elliptic, 9đ
Luận văn: Giải bài toán Dirichlet đối với phương trình Elliptic, 9đ
Dịch vụ viết bài trọn gói ZALO 0917193864
 
Bài tập số phức
Bài tập số phứcBài tập số phức
Bài tập số phức
phuonganhtran1303
 
Ứng dụng tích phân tính giới hạn của dãy số
Ứng dụng tích phân tính giới hạn của dãy sốỨng dụng tích phân tính giới hạn của dãy số
Ứng dụng tích phân tính giới hạn của dãy số
Sirô Tiny
 
19 phương phap chứng minh bất đẳng thức
19 phương phap chứng minh bất đẳng thức19 phương phap chứng minh bất đẳng thức
19 phương phap chứng minh bất đẳng thứcThế Giới Tinh Hoa
 
Luận văn: Phép biến đổi phân tuyến tính, HAY, 9đ
Luận văn: Phép biến đổi phân tuyến tính, HAY, 9đLuận văn: Phép biến đổi phân tuyến tính, HAY, 9đ
Luận văn: Phép biến đổi phân tuyến tính, HAY, 9đ
Dịch vụ viết bài trọn gói ZALO 0917193864
 
Giai nhanh phuong phap tinh
Giai nhanh phuong phap tinhGiai nhanh phuong phap tinh
Giai nhanh phuong phap tinh
Pham Huy
 
Hinh hoc so cap va thuc hanh giai toan
Hinh hoc so cap va thuc hanh giai toanHinh hoc so cap va thuc hanh giai toan
Hinh hoc so cap va thuc hanh giai toan
TamPhan59
 
Bộ sưu tập bất đẳng thức của võ quốc bá cẩn
Bộ sưu tập bất đẳng thức của võ quốc bá cẩnBộ sưu tập bất đẳng thức của võ quốc bá cẩn
Bộ sưu tập bất đẳng thức của võ quốc bá cẩn
Thế Giới Tinh Hoa
 

What's hot (20)

Topo daicuong1[1]
Topo daicuong1[1]Topo daicuong1[1]
Topo daicuong1[1]
 
Cân bằng hệ số trong bđt AM-GM
Cân bằng hệ số trong bđt AM-GMCân bằng hệ số trong bđt AM-GM
Cân bằng hệ số trong bđt AM-GM
 
Phương pháp giải phương trình lượng giác
Phương pháp giải phương trình lượng giácPhương pháp giải phương trình lượng giác
Phương pháp giải phương trình lượng giác
 
Kỹ thuật nhân liên hợp
Kỹ thuật nhân liên hợpKỹ thuật nhân liên hợp
Kỹ thuật nhân liên hợp
 
đại số tuyến tính 2 ( không gian eculid )
đại số tuyến tính 2 ( không gian eculid )đại số tuyến tính 2 ( không gian eculid )
đại số tuyến tính 2 ( không gian eculid )
 
74774655 chuỗi-ham
74774655 chuỗi-ham74774655 chuỗi-ham
74774655 chuỗi-ham
 
Bài Giảng Đại Số Tuyến Tính - ĐH Thăng Long
Bài Giảng Đại Số Tuyến Tính - ĐH Thăng LongBài Giảng Đại Số Tuyến Tính - ĐH Thăng Long
Bài Giảng Đại Số Tuyến Tính - ĐH Thăng Long
 
Bài toán số học liên quan tới lũy thữa
Bài toán số học liên quan tới lũy thữaBài toán số học liên quan tới lũy thữa
Bài toán số học liên quan tới lũy thữa
 
Toan a1 -_bai_giang
Toan a1 -_bai_giangToan a1 -_bai_giang
Toan a1 -_bai_giang
 
Gt khong gian_metric Nguyen Hoang
Gt khong gian_metric Nguyen HoangGt khong gian_metric Nguyen Hoang
Gt khong gian_metric Nguyen Hoang
 
Chuong01
Chuong01Chuong01
Chuong01
 
Luận văn: Giải bài toán Dirichlet đối với phương trình Elliptic, 9đ
Luận văn: Giải bài toán Dirichlet đối với phương trình Elliptic, 9đLuận văn: Giải bài toán Dirichlet đối với phương trình Elliptic, 9đ
Luận văn: Giải bài toán Dirichlet đối với phương trình Elliptic, 9đ
 
Bài tập số phức
Bài tập số phứcBài tập số phức
Bài tập số phức
 
Ứng dụng tích phân tính giới hạn của dãy số
Ứng dụng tích phân tính giới hạn của dãy sốỨng dụng tích phân tính giới hạn của dãy số
Ứng dụng tích phân tính giới hạn của dãy số
 
19 phương phap chứng minh bất đẳng thức
19 phương phap chứng minh bất đẳng thức19 phương phap chứng minh bất đẳng thức
19 phương phap chứng minh bất đẳng thức
 
Luận văn: Phép biến đổi phân tuyến tính, HAY, 9đ
Luận văn: Phép biến đổi phân tuyến tính, HAY, 9đLuận văn: Phép biến đổi phân tuyến tính, HAY, 9đ
Luận văn: Phép biến đổi phân tuyến tính, HAY, 9đ
 
Giai nhanh phuong phap tinh
Giai nhanh phuong phap tinhGiai nhanh phuong phap tinh
Giai nhanh phuong phap tinh
 
Bdt thuần nhất
Bdt thuần nhấtBdt thuần nhất
Bdt thuần nhất
 
Hinh hoc so cap va thuc hanh giai toan
Hinh hoc so cap va thuc hanh giai toanHinh hoc so cap va thuc hanh giai toan
Hinh hoc so cap va thuc hanh giai toan
 
Bộ sưu tập bất đẳng thức của võ quốc bá cẩn
Bộ sưu tập bất đẳng thức của võ quốc bá cẩnBộ sưu tập bất đẳng thức của võ quốc bá cẩn
Bộ sưu tập bất đẳng thức của võ quốc bá cẩn
 

Similar to Luận văn: Một số bài toán về dãy số, HAY, 9đ

Luận văn: Một số lớp bài toán về loại phương trình hàm, HAY - Gửi miễn phí qu...
Luận văn: Một số lớp bài toán về loại phương trình hàm, HAY - Gửi miễn phí qu...Luận văn: Một số lớp bài toán về loại phương trình hàm, HAY - Gửi miễn phí qu...
Luận văn: Một số lớp bài toán về loại phương trình hàm, HAY - Gửi miễn phí qu...
Dịch vụ viết bài trọn gói ZALO: 0909232620
 
Luận văn: Một số lớp bài toán về phương trình hàm, HAY, 9đ
Luận văn: Một số lớp bài toán về phương trình hàm, HAY, 9đLuận văn: Một số lớp bài toán về phương trình hàm, HAY, 9đ
Luận văn: Một số lớp bài toán về phương trình hàm, HAY, 9đ
Dịch vụ viết bài trọn gói ZALO: 0909232620
 
Luận văn: Một số phương pháp giải phương trình hàm, HOT, 9đ
Luận văn: Một số phương pháp giải phương trình hàm, HOT, 9đLuận văn: Một số phương pháp giải phương trình hàm, HOT, 9đ
Luận văn: Một số phương pháp giải phương trình hàm, HOT, 9đ
Dịch vụ viết bài trọn gói ZALO: 0909232620
 
Luận văn: Một số phương pháp giải phương trình hàm, HOT, 9đ
Luận văn: Một số phương pháp giải phương trình hàm, HOT, 9đLuận văn: Một số phương pháp giải phương trình hàm, HOT, 9đ
Luận văn: Một số phương pháp giải phương trình hàm, HOT, 9đ
Dịch vụ viết bài trọn gói ZALO: 0909232620
 
Luận văn: Giải số phương trình vi phân đại số bằng đa bước, 9đ
Luận văn: Giải số phương trình vi phân đại số bằng đa bước, 9đLuận văn: Giải số phương trình vi phân đại số bằng đa bước, 9đ
Luận văn: Giải số phương trình vi phân đại số bằng đa bước, 9đ
Dịch vụ viết bài trọn gói ZALO 0917193864
 
Sach bat dang thuc rat hay
Sach bat dang thuc rat haySach bat dang thuc rat hay
Sach bat dang thuc rat hayTuân Ngô
 
Bất đẳng thức mathscope
Bất đẳng thức mathscopeBất đẳng thức mathscope
Bất đẳng thức mathscopePhúc Võ
 
Một Số Vấn Đề Chọn Lọc Về Dãy Số.doc
Một Số Vấn Đề Chọn Lọc Về Dãy Số.docMột Số Vấn Đề Chọn Lọc Về Dãy Số.doc
Một Số Vấn Đề Chọn Lọc Về Dãy Số.doc
Dịch vụ viết đề tài trọn gói 0934.573.149
 
Sachbatdangthucrathay 131117005237-phpapp01
Sachbatdangthucrathay 131117005237-phpapp01Sachbatdangthucrathay 131117005237-phpapp01
Sachbatdangthucrathay 131117005237-phpapp01sungalung
 
Luận văn thạc sĩ: Quy hoạch toàn phương, HAY, 9đ
Luận văn thạc sĩ: Quy hoạch toàn phương, HAY, 9đLuận văn thạc sĩ: Quy hoạch toàn phương, HAY, 9đ
Luận văn thạc sĩ: Quy hoạch toàn phương, HAY, 9đ
Dịch Vụ Viết Bài Trọn Gói ZALO 0917193864
 
Luận văn: Các dạng phương trình lượng giác, HOT - Gửi miễn phí qua zalo=> 090...
Luận văn: Các dạng phương trình lượng giác, HOT - Gửi miễn phí qua zalo=> 090...Luận văn: Các dạng phương trình lượng giác, HOT - Gửi miễn phí qua zalo=> 090...
Luận văn: Các dạng phương trình lượng giác, HOT - Gửi miễn phí qua zalo=> 090...
Dịch vụ viết bài trọn gói ZALO: 0909232620
 
Luận văn: Các dạng phương trình lượng giác, HAY, 9đ
Luận văn: Các dạng phương trình lượng giác, HAY, 9đLuận văn: Các dạng phương trình lượng giác, HAY, 9đ
Luận văn: Các dạng phương trình lượng giác, HAY, 9đ
Dịch vụ viết bài trọn gói ZALO: 0909232620
 
Một số lớp bài toán tối ưu không lồi, Thuật toán và ứng dụng.pdf
Một số lớp bài toán tối ưu không lồi, Thuật toán và ứng dụng.pdfMột số lớp bài toán tối ưu không lồi, Thuật toán và ứng dụng.pdf
Một số lớp bài toán tối ưu không lồi, Thuật toán và ứng dụng.pdf
Man_Ebook
 
Luận văn: Đa tạp tâm của hệ tam phân mũ không đều, HOT
Luận văn: Đa tạp tâm của hệ tam phân mũ không đều, HOTLuận văn: Đa tạp tâm của hệ tam phân mũ không đều, HOT
Luận văn: Đa tạp tâm của hệ tam phân mũ không đều, HOT
Dịch Vụ Viết Bài Trọn Gói ZALO 0917193864
 
Luận văn: Đa tạp tâm của hệ tam phân mũ không đều, HAY, 9đ
Luận văn: Đa tạp tâm của hệ tam phân mũ không đều, HAY, 9đLuận văn: Đa tạp tâm của hệ tam phân mũ không đều, HAY, 9đ
Luận văn: Đa tạp tâm của hệ tam phân mũ không đều, HAY, 9đ
Dịch vụ viết bài trọn gói ZALO 0917193864
 
Đề tài: Giải tích trên thang thời gian của mô hình ngẫu nhiên, HOT
Đề tài: Giải tích trên thang thời gian của mô hình ngẫu nhiên, HOTĐề tài: Giải tích trên thang thời gian của mô hình ngẫu nhiên, HOT
Đề tài: Giải tích trên thang thời gian của mô hình ngẫu nhiên, HOT
Dịch vụ viết bài trọn gói ZALO: 0909232620
 
Luận văn: Sáu phương pháp giải các bài toán phổ thông, HOT
Luận văn: Sáu phương pháp giải các bài toán phổ thông, HOTLuận văn: Sáu phương pháp giải các bài toán phổ thông, HOT
Luận văn: Sáu phương pháp giải các bài toán phổ thông, HOT
Dịch Vụ Viết Bài Trọn Gói ZALO 0917193864
 
Luận văn: Sáu phương pháp giải các bài toán phổ thông, HAY, 9đ
Luận văn: Sáu phương pháp giải các bài toán phổ thông, HAY, 9đLuận văn: Sáu phương pháp giải các bài toán phổ thông, HAY, 9đ
Luận văn: Sáu phương pháp giải các bài toán phổ thông, HAY, 9đ
Dịch Vụ Viết Bài Trọn Gói ZALO 0917193864
 
M T So Dạng Toán Ve Dãy So Sinh B I Các Hàm So Sơ Cap.docx
M T So Dạng Toán Ve Dãy So Sinh B I Các Hàm So Sơ Cap.docxM T So Dạng Toán Ve Dãy So Sinh B I Các Hàm So Sơ Cap.docx
M T So Dạng Toán Ve Dãy So Sinh B I Các Hàm So Sơ Cap.docx
DV Viết Luận văn luanvanmaster.com ZALO 0973287149
 

Similar to Luận văn: Một số bài toán về dãy số, HAY, 9đ (20)

Luận văn: Một số lớp bài toán về loại phương trình hàm, HAY - Gửi miễn phí qu...
Luận văn: Một số lớp bài toán về loại phương trình hàm, HAY - Gửi miễn phí qu...Luận văn: Một số lớp bài toán về loại phương trình hàm, HAY - Gửi miễn phí qu...
Luận văn: Một số lớp bài toán về loại phương trình hàm, HAY - Gửi miễn phí qu...
 
Luận văn: Một số lớp bài toán về phương trình hàm, HAY, 9đ
Luận văn: Một số lớp bài toán về phương trình hàm, HAY, 9đLuận văn: Một số lớp bài toán về phương trình hàm, HAY, 9đ
Luận văn: Một số lớp bài toán về phương trình hàm, HAY, 9đ
 
Luận văn: Một số phương pháp giải phương trình hàm, HOT, 9đ
Luận văn: Một số phương pháp giải phương trình hàm, HOT, 9đLuận văn: Một số phương pháp giải phương trình hàm, HOT, 9đ
Luận văn: Một số phương pháp giải phương trình hàm, HOT, 9đ
 
Luận văn: Một số phương pháp giải phương trình hàm, HOT, 9đ
Luận văn: Một số phương pháp giải phương trình hàm, HOT, 9đLuận văn: Một số phương pháp giải phương trình hàm, HOT, 9đ
Luận văn: Một số phương pháp giải phương trình hàm, HOT, 9đ
 
Luận văn: Giải số phương trình vi phân đại số bằng đa bước, 9đ
Luận văn: Giải số phương trình vi phân đại số bằng đa bước, 9đLuận văn: Giải số phương trình vi phân đại số bằng đa bước, 9đ
Luận văn: Giải số phương trình vi phân đại số bằng đa bước, 9đ
 
Sach bat dang thuc rat hay
Sach bat dang thuc rat haySach bat dang thuc rat hay
Sach bat dang thuc rat hay
 
Bất đẳng thức mathscope
Bất đẳng thức mathscopeBất đẳng thức mathscope
Bất đẳng thức mathscope
 
Một Số Vấn Đề Chọn Lọc Về Dãy Số.doc
Một Số Vấn Đề Chọn Lọc Về Dãy Số.docMột Số Vấn Đề Chọn Lọc Về Dãy Số.doc
Một Số Vấn Đề Chọn Lọc Về Dãy Số.doc
 
Sachbatdangthucrathay 131117005237-phpapp01
Sachbatdangthucrathay 131117005237-phpapp01Sachbatdangthucrathay 131117005237-phpapp01
Sachbatdangthucrathay 131117005237-phpapp01
 
Luận văn thạc sĩ: Quy hoạch toàn phương, HAY, 9đ
Luận văn thạc sĩ: Quy hoạch toàn phương, HAY, 9đLuận văn thạc sĩ: Quy hoạch toàn phương, HAY, 9đ
Luận văn thạc sĩ: Quy hoạch toàn phương, HAY, 9đ
 
Luận văn: Các dạng phương trình lượng giác, HOT - Gửi miễn phí qua zalo=> 090...
Luận văn: Các dạng phương trình lượng giác, HOT - Gửi miễn phí qua zalo=> 090...Luận văn: Các dạng phương trình lượng giác, HOT - Gửi miễn phí qua zalo=> 090...
Luận văn: Các dạng phương trình lượng giác, HOT - Gửi miễn phí qua zalo=> 090...
 
Luận văn: Các dạng phương trình lượng giác, HAY, 9đ
Luận văn: Các dạng phương trình lượng giác, HAY, 9đLuận văn: Các dạng phương trình lượng giác, HAY, 9đ
Luận văn: Các dạng phương trình lượng giác, HAY, 9đ
 
Một số lớp bài toán tối ưu không lồi, Thuật toán và ứng dụng.pdf
Một số lớp bài toán tối ưu không lồi, Thuật toán và ứng dụng.pdfMột số lớp bài toán tối ưu không lồi, Thuật toán và ứng dụng.pdf
Một số lớp bài toán tối ưu không lồi, Thuật toán và ứng dụng.pdf
 
Luận văn: Đa tạp tâm của hệ tam phân mũ không đều, HOT
Luận văn: Đa tạp tâm của hệ tam phân mũ không đều, HOTLuận văn: Đa tạp tâm của hệ tam phân mũ không đều, HOT
Luận văn: Đa tạp tâm của hệ tam phân mũ không đều, HOT
 
Luận văn: Đa tạp tâm của hệ tam phân mũ không đều, HAY, 9đ
Luận văn: Đa tạp tâm của hệ tam phân mũ không đều, HAY, 9đLuận văn: Đa tạp tâm của hệ tam phân mũ không đều, HAY, 9đ
Luận văn: Đa tạp tâm của hệ tam phân mũ không đều, HAY, 9đ
 
Hoán vị lặp tổ hợp
Hoán vị lặp tổ hợpHoán vị lặp tổ hợp
Hoán vị lặp tổ hợp
 
Đề tài: Giải tích trên thang thời gian của mô hình ngẫu nhiên, HOT
Đề tài: Giải tích trên thang thời gian của mô hình ngẫu nhiên, HOTĐề tài: Giải tích trên thang thời gian của mô hình ngẫu nhiên, HOT
Đề tài: Giải tích trên thang thời gian của mô hình ngẫu nhiên, HOT
 
Luận văn: Sáu phương pháp giải các bài toán phổ thông, HOT
Luận văn: Sáu phương pháp giải các bài toán phổ thông, HOTLuận văn: Sáu phương pháp giải các bài toán phổ thông, HOT
Luận văn: Sáu phương pháp giải các bài toán phổ thông, HOT
 
Luận văn: Sáu phương pháp giải các bài toán phổ thông, HAY, 9đ
Luận văn: Sáu phương pháp giải các bài toán phổ thông, HAY, 9đLuận văn: Sáu phương pháp giải các bài toán phổ thông, HAY, 9đ
Luận văn: Sáu phương pháp giải các bài toán phổ thông, HAY, 9đ
 
M T So Dạng Toán Ve Dãy So Sinh B I Các Hàm So Sơ Cap.docx
M T So Dạng Toán Ve Dãy So Sinh B I Các Hàm So Sơ Cap.docxM T So Dạng Toán Ve Dãy So Sinh B I Các Hàm So Sơ Cap.docx
M T So Dạng Toán Ve Dãy So Sinh B I Các Hàm So Sơ Cap.docx
 

More from Dịch Vụ Viết Bài Trọn Gói ZALO 0917193864

Yếu Tố Tự Truyện Trong Truyện Ngắn Thạch Lam Và Thanh Tịnh.doc
Yếu Tố Tự Truyện Trong Truyện Ngắn Thạch Lam Và Thanh Tịnh.docYếu Tố Tự Truyện Trong Truyện Ngắn Thạch Lam Và Thanh Tịnh.doc
Yếu Tố Tự Truyện Trong Truyện Ngắn Thạch Lam Và Thanh Tịnh.doc
Dịch Vụ Viết Bài Trọn Gói ZALO 0917193864
 
Từ Ngữ Biểu Thị Tâm Lí – Tình Cảm Trong Ca Dao Người Việt.doc
Từ Ngữ Biểu Thị Tâm Lí – Tình Cảm Trong Ca Dao Người Việt.docTừ Ngữ Biểu Thị Tâm Lí – Tình Cảm Trong Ca Dao Người Việt.doc
Từ Ngữ Biểu Thị Tâm Lí – Tình Cảm Trong Ca Dao Người Việt.doc
Dịch Vụ Viết Bài Trọn Gói ZALO 0917193864
 
Quản Lý Hoạt Động Dạy Học Các Môn Khoa Học Tự Nhiên Theo Chuẩn Kiến Thức Và K...
Quản Lý Hoạt Động Dạy Học Các Môn Khoa Học Tự Nhiên Theo Chuẩn Kiến Thức Và K...Quản Lý Hoạt Động Dạy Học Các Môn Khoa Học Tự Nhiên Theo Chuẩn Kiến Thức Và K...
Quản Lý Hoạt Động Dạy Học Các Môn Khoa Học Tự Nhiên Theo Chuẩn Kiến Thức Và K...
Dịch Vụ Viết Bài Trọn Gói ZALO 0917193864
 
Quản Lý Thu Thuế Giá Trị Gia Tăng Đối Với Doanh Nghiệp Ngoài Quốc Doanh Trên ...
Quản Lý Thu Thuế Giá Trị Gia Tăng Đối Với Doanh Nghiệp Ngoài Quốc Doanh Trên ...Quản Lý Thu Thuế Giá Trị Gia Tăng Đối Với Doanh Nghiệp Ngoài Quốc Doanh Trên ...
Quản Lý Thu Thuế Giá Trị Gia Tăng Đối Với Doanh Nghiệp Ngoài Quốc Doanh Trên ...
Dịch Vụ Viết Bài Trọn Gói ZALO 0917193864
 
Thu Hút Nguồn Nhân Lực Trình Độ Cao Vào Các Cơ Quan Hành Chính Nhà Nước Tỉnh ...
Thu Hút Nguồn Nhân Lực Trình Độ Cao Vào Các Cơ Quan Hành Chính Nhà Nước Tỉnh ...Thu Hút Nguồn Nhân Lực Trình Độ Cao Vào Các Cơ Quan Hành Chính Nhà Nước Tỉnh ...
Thu Hút Nguồn Nhân Lực Trình Độ Cao Vào Các Cơ Quan Hành Chính Nhà Nước Tỉnh ...
Dịch Vụ Viết Bài Trọn Gói ZALO 0917193864
 
Quản Trị Rủi Ro Tín Dụng Trong Cho Vay Doanh Nghiệp Tại Ngân Hàng Thương Mại ...
Quản Trị Rủi Ro Tín Dụng Trong Cho Vay Doanh Nghiệp Tại Ngân Hàng Thương Mại ...Quản Trị Rủi Ro Tín Dụng Trong Cho Vay Doanh Nghiệp Tại Ngân Hàng Thương Mại ...
Quản Trị Rủi Ro Tín Dụng Trong Cho Vay Doanh Nghiệp Tại Ngân Hàng Thương Mại ...
Dịch Vụ Viết Bài Trọn Gói ZALO 0917193864
 
Vaporisation Of Single And Binary Component Droplets In Heated Flowing Gas St...
Vaporisation Of Single And Binary Component Droplets In Heated Flowing Gas St...Vaporisation Of Single And Binary Component Droplets In Heated Flowing Gas St...
Vaporisation Of Single And Binary Component Droplets In Heated Flowing Gas St...
Dịch Vụ Viết Bài Trọn Gói ZALO 0917193864
 
Quản Lý Hoạt Động Dạy Học Các Trường Thpt Trên Địa Bàn Huyện Sơn Hà Tỉnh Quản...
Quản Lý Hoạt Động Dạy Học Các Trường Thpt Trên Địa Bàn Huyện Sơn Hà Tỉnh Quản...Quản Lý Hoạt Động Dạy Học Các Trường Thpt Trên Địa Bàn Huyện Sơn Hà Tỉnh Quản...
Quản Lý Hoạt Động Dạy Học Các Trường Thpt Trên Địa Bàn Huyện Sơn Hà Tỉnh Quản...
Dịch Vụ Viết Bài Trọn Gói ZALO 0917193864
 
Tác Giả Hàm Ẩn Trong Tiểu Thuyết Nguyễn Việt Hà.doc
Tác Giả Hàm Ẩn Trong Tiểu Thuyết Nguyễn Việt Hà.docTác Giả Hàm Ẩn Trong Tiểu Thuyết Nguyễn Việt Hà.doc
Tác Giả Hàm Ẩn Trong Tiểu Thuyết Nguyễn Việt Hà.doc
Dịch Vụ Viết Bài Trọn Gói ZALO 0917193864
 
Quản Trị Rủi Ro Tín Dụng Trong Cho Vay Ngắn Hạn Tại Ngân Hàng Công Thƣơng Chi...
Quản Trị Rủi Ro Tín Dụng Trong Cho Vay Ngắn Hạn Tại Ngân Hàng Công Thƣơng Chi...Quản Trị Rủi Ro Tín Dụng Trong Cho Vay Ngắn Hạn Tại Ngân Hàng Công Thƣơng Chi...
Quản Trị Rủi Ro Tín Dụng Trong Cho Vay Ngắn Hạn Tại Ngân Hàng Công Thƣơng Chi...
Dịch Vụ Viết Bài Trọn Gói ZALO 0917193864
 
Quản Lý Nhà Nước Về Nuôi Trồng Thủy Sản Nước Ngọt Trên Địa Bàn Thành Phố Hải ...
Quản Lý Nhà Nước Về Nuôi Trồng Thủy Sản Nước Ngọt Trên Địa Bàn Thành Phố Hải ...Quản Lý Nhà Nước Về Nuôi Trồng Thủy Sản Nước Ngọt Trên Địa Bàn Thành Phố Hải ...
Quản Lý Nhà Nước Về Nuôi Trồng Thủy Sản Nước Ngọt Trên Địa Bàn Thành Phố Hải ...
Dịch Vụ Viết Bài Trọn Gói ZALO 0917193864
 
Song Song Hóa Các Thuật Toán Trên Mạng Đồ Thị.doc
Song Song Hóa Các Thuật Toán Trên Mạng Đồ Thị.docSong Song Hóa Các Thuật Toán Trên Mạng Đồ Thị.doc
Song Song Hóa Các Thuật Toán Trên Mạng Đồ Thị.doc
Dịch Vụ Viết Bài Trọn Gói ZALO 0917193864
 
Ứng Dụng Số Phức Trong Các Bài Toán Sơ Cấp.doc
Ứng Dụng Số Phức Trong Các Bài Toán Sơ Cấp.docỨng Dụng Số Phức Trong Các Bài Toán Sơ Cấp.doc
Ứng Dụng Số Phức Trong Các Bài Toán Sơ Cấp.doc
Dịch Vụ Viết Bài Trọn Gói ZALO 0917193864
 
Vai Trò Của Cái Bi Trong Giáo Dục Thẩm Mỹ.doc
Vai Trò Của Cái Bi Trong Giáo Dục Thẩm Mỹ.docVai Trò Của Cái Bi Trong Giáo Dục Thẩm Mỹ.doc
Vai Trò Của Cái Bi Trong Giáo Dục Thẩm Mỹ.doc
Dịch Vụ Viết Bài Trọn Gói ZALO 0917193864
 
Quản Lý Hoạt Động Giáo Dục Ngoài Giờ Lên Lớp Ở Các Trường Thcs Huyện Chư Păh ...
Quản Lý Hoạt Động Giáo Dục Ngoài Giờ Lên Lớp Ở Các Trường Thcs Huyện Chư Păh ...Quản Lý Hoạt Động Giáo Dục Ngoài Giờ Lên Lớp Ở Các Trường Thcs Huyện Chư Păh ...
Quản Lý Hoạt Động Giáo Dục Ngoài Giờ Lên Lớp Ở Các Trường Thcs Huyện Chư Păh ...
Dịch Vụ Viết Bài Trọn Gói ZALO 0917193864
 
Thu Hút Vốn Đầu Tư Vào Lĩnh Vực Nông Nghiệp Trên Địa Bàn Tỉnh Gia Lai.doc
Thu Hút Vốn Đầu Tư Vào Lĩnh Vực Nông Nghiệp Trên Địa Bàn Tỉnh Gia Lai.docThu Hút Vốn Đầu Tư Vào Lĩnh Vực Nông Nghiệp Trên Địa Bàn Tỉnh Gia Lai.doc
Thu Hút Vốn Đầu Tư Vào Lĩnh Vực Nông Nghiệp Trên Địa Bàn Tỉnh Gia Lai.doc
Dịch Vụ Viết Bài Trọn Gói ZALO 0917193864
 
Quản Lý Hoạt Động Dạy Học Ngoại Ngữ Tại Các Trung Tâm Ngoại Ngữ - Tin Học Trê...
Quản Lý Hoạt Động Dạy Học Ngoại Ngữ Tại Các Trung Tâm Ngoại Ngữ - Tin Học Trê...Quản Lý Hoạt Động Dạy Học Ngoại Ngữ Tại Các Trung Tâm Ngoại Ngữ - Tin Học Trê...
Quản Lý Hoạt Động Dạy Học Ngoại Ngữ Tại Các Trung Tâm Ngoại Ngữ - Tin Học Trê...
Dịch Vụ Viết Bài Trọn Gói ZALO 0917193864
 
Quản Trị Rủi Ro Tín Dụng Trong Cho Vay Doanh Nghiệp Tại Ngân Hàng Thƣơng Mại ...
Quản Trị Rủi Ro Tín Dụng Trong Cho Vay Doanh Nghiệp Tại Ngân Hàng Thƣơng Mại ...Quản Trị Rủi Ro Tín Dụng Trong Cho Vay Doanh Nghiệp Tại Ngân Hàng Thƣơng Mại ...
Quản Trị Rủi Ro Tín Dụng Trong Cho Vay Doanh Nghiệp Tại Ngân Hàng Thƣơng Mại ...
Dịch Vụ Viết Bài Trọn Gói ZALO 0917193864
 
Tạo Việc Làm Cho Thanh Niên Trên Địa Bàn Quận Thanh Khê, Thành Phố Đà Nẵng.doc
Tạo Việc Làm Cho Thanh Niên Trên Địa Bàn Quận Thanh Khê, Thành Phố Đà Nẵng.docTạo Việc Làm Cho Thanh Niên Trên Địa Bàn Quận Thanh Khê, Thành Phố Đà Nẵng.doc
Tạo Việc Làm Cho Thanh Niên Trên Địa Bàn Quận Thanh Khê, Thành Phố Đà Nẵng.doc
Dịch Vụ Viết Bài Trọn Gói ZALO 0917193864
 
Quản Trị Rủi Ro Tín Dụng Trong Cho Vay Trung Và Dài Hạn Tại Ngân Hàng Thương ...
Quản Trị Rủi Ro Tín Dụng Trong Cho Vay Trung Và Dài Hạn Tại Ngân Hàng Thương ...Quản Trị Rủi Ro Tín Dụng Trong Cho Vay Trung Và Dài Hạn Tại Ngân Hàng Thương ...
Quản Trị Rủi Ro Tín Dụng Trong Cho Vay Trung Và Dài Hạn Tại Ngân Hàng Thương ...
Dịch Vụ Viết Bài Trọn Gói ZALO 0917193864
 

More from Dịch Vụ Viết Bài Trọn Gói ZALO 0917193864 (20)

Yếu Tố Tự Truyện Trong Truyện Ngắn Thạch Lam Và Thanh Tịnh.doc
Yếu Tố Tự Truyện Trong Truyện Ngắn Thạch Lam Và Thanh Tịnh.docYếu Tố Tự Truyện Trong Truyện Ngắn Thạch Lam Và Thanh Tịnh.doc
Yếu Tố Tự Truyện Trong Truyện Ngắn Thạch Lam Và Thanh Tịnh.doc
 
Từ Ngữ Biểu Thị Tâm Lí – Tình Cảm Trong Ca Dao Người Việt.doc
Từ Ngữ Biểu Thị Tâm Lí – Tình Cảm Trong Ca Dao Người Việt.docTừ Ngữ Biểu Thị Tâm Lí – Tình Cảm Trong Ca Dao Người Việt.doc
Từ Ngữ Biểu Thị Tâm Lí – Tình Cảm Trong Ca Dao Người Việt.doc
 
Quản Lý Hoạt Động Dạy Học Các Môn Khoa Học Tự Nhiên Theo Chuẩn Kiến Thức Và K...
Quản Lý Hoạt Động Dạy Học Các Môn Khoa Học Tự Nhiên Theo Chuẩn Kiến Thức Và K...Quản Lý Hoạt Động Dạy Học Các Môn Khoa Học Tự Nhiên Theo Chuẩn Kiến Thức Và K...
Quản Lý Hoạt Động Dạy Học Các Môn Khoa Học Tự Nhiên Theo Chuẩn Kiến Thức Và K...
 
Quản Lý Thu Thuế Giá Trị Gia Tăng Đối Với Doanh Nghiệp Ngoài Quốc Doanh Trên ...
Quản Lý Thu Thuế Giá Trị Gia Tăng Đối Với Doanh Nghiệp Ngoài Quốc Doanh Trên ...Quản Lý Thu Thuế Giá Trị Gia Tăng Đối Với Doanh Nghiệp Ngoài Quốc Doanh Trên ...
Quản Lý Thu Thuế Giá Trị Gia Tăng Đối Với Doanh Nghiệp Ngoài Quốc Doanh Trên ...
 
Thu Hút Nguồn Nhân Lực Trình Độ Cao Vào Các Cơ Quan Hành Chính Nhà Nước Tỉnh ...
Thu Hút Nguồn Nhân Lực Trình Độ Cao Vào Các Cơ Quan Hành Chính Nhà Nước Tỉnh ...Thu Hút Nguồn Nhân Lực Trình Độ Cao Vào Các Cơ Quan Hành Chính Nhà Nước Tỉnh ...
Thu Hút Nguồn Nhân Lực Trình Độ Cao Vào Các Cơ Quan Hành Chính Nhà Nước Tỉnh ...
 
Quản Trị Rủi Ro Tín Dụng Trong Cho Vay Doanh Nghiệp Tại Ngân Hàng Thương Mại ...
Quản Trị Rủi Ro Tín Dụng Trong Cho Vay Doanh Nghiệp Tại Ngân Hàng Thương Mại ...Quản Trị Rủi Ro Tín Dụng Trong Cho Vay Doanh Nghiệp Tại Ngân Hàng Thương Mại ...
Quản Trị Rủi Ro Tín Dụng Trong Cho Vay Doanh Nghiệp Tại Ngân Hàng Thương Mại ...
 
Vaporisation Of Single And Binary Component Droplets In Heated Flowing Gas St...
Vaporisation Of Single And Binary Component Droplets In Heated Flowing Gas St...Vaporisation Of Single And Binary Component Droplets In Heated Flowing Gas St...
Vaporisation Of Single And Binary Component Droplets In Heated Flowing Gas St...
 
Quản Lý Hoạt Động Dạy Học Các Trường Thpt Trên Địa Bàn Huyện Sơn Hà Tỉnh Quản...
Quản Lý Hoạt Động Dạy Học Các Trường Thpt Trên Địa Bàn Huyện Sơn Hà Tỉnh Quản...Quản Lý Hoạt Động Dạy Học Các Trường Thpt Trên Địa Bàn Huyện Sơn Hà Tỉnh Quản...
Quản Lý Hoạt Động Dạy Học Các Trường Thpt Trên Địa Bàn Huyện Sơn Hà Tỉnh Quản...
 
Tác Giả Hàm Ẩn Trong Tiểu Thuyết Nguyễn Việt Hà.doc
Tác Giả Hàm Ẩn Trong Tiểu Thuyết Nguyễn Việt Hà.docTác Giả Hàm Ẩn Trong Tiểu Thuyết Nguyễn Việt Hà.doc
Tác Giả Hàm Ẩn Trong Tiểu Thuyết Nguyễn Việt Hà.doc
 
Quản Trị Rủi Ro Tín Dụng Trong Cho Vay Ngắn Hạn Tại Ngân Hàng Công Thƣơng Chi...
Quản Trị Rủi Ro Tín Dụng Trong Cho Vay Ngắn Hạn Tại Ngân Hàng Công Thƣơng Chi...Quản Trị Rủi Ro Tín Dụng Trong Cho Vay Ngắn Hạn Tại Ngân Hàng Công Thƣơng Chi...
Quản Trị Rủi Ro Tín Dụng Trong Cho Vay Ngắn Hạn Tại Ngân Hàng Công Thƣơng Chi...
 
Quản Lý Nhà Nước Về Nuôi Trồng Thủy Sản Nước Ngọt Trên Địa Bàn Thành Phố Hải ...
Quản Lý Nhà Nước Về Nuôi Trồng Thủy Sản Nước Ngọt Trên Địa Bàn Thành Phố Hải ...Quản Lý Nhà Nước Về Nuôi Trồng Thủy Sản Nước Ngọt Trên Địa Bàn Thành Phố Hải ...
Quản Lý Nhà Nước Về Nuôi Trồng Thủy Sản Nước Ngọt Trên Địa Bàn Thành Phố Hải ...
 
Song Song Hóa Các Thuật Toán Trên Mạng Đồ Thị.doc
Song Song Hóa Các Thuật Toán Trên Mạng Đồ Thị.docSong Song Hóa Các Thuật Toán Trên Mạng Đồ Thị.doc
Song Song Hóa Các Thuật Toán Trên Mạng Đồ Thị.doc
 
Ứng Dụng Số Phức Trong Các Bài Toán Sơ Cấp.doc
Ứng Dụng Số Phức Trong Các Bài Toán Sơ Cấp.docỨng Dụng Số Phức Trong Các Bài Toán Sơ Cấp.doc
Ứng Dụng Số Phức Trong Các Bài Toán Sơ Cấp.doc
 
Vai Trò Của Cái Bi Trong Giáo Dục Thẩm Mỹ.doc
Vai Trò Của Cái Bi Trong Giáo Dục Thẩm Mỹ.docVai Trò Của Cái Bi Trong Giáo Dục Thẩm Mỹ.doc
Vai Trò Của Cái Bi Trong Giáo Dục Thẩm Mỹ.doc
 
Quản Lý Hoạt Động Giáo Dục Ngoài Giờ Lên Lớp Ở Các Trường Thcs Huyện Chư Păh ...
Quản Lý Hoạt Động Giáo Dục Ngoài Giờ Lên Lớp Ở Các Trường Thcs Huyện Chư Păh ...Quản Lý Hoạt Động Giáo Dục Ngoài Giờ Lên Lớp Ở Các Trường Thcs Huyện Chư Păh ...
Quản Lý Hoạt Động Giáo Dục Ngoài Giờ Lên Lớp Ở Các Trường Thcs Huyện Chư Păh ...
 
Thu Hút Vốn Đầu Tư Vào Lĩnh Vực Nông Nghiệp Trên Địa Bàn Tỉnh Gia Lai.doc
Thu Hút Vốn Đầu Tư Vào Lĩnh Vực Nông Nghiệp Trên Địa Bàn Tỉnh Gia Lai.docThu Hút Vốn Đầu Tư Vào Lĩnh Vực Nông Nghiệp Trên Địa Bàn Tỉnh Gia Lai.doc
Thu Hút Vốn Đầu Tư Vào Lĩnh Vực Nông Nghiệp Trên Địa Bàn Tỉnh Gia Lai.doc
 
Quản Lý Hoạt Động Dạy Học Ngoại Ngữ Tại Các Trung Tâm Ngoại Ngữ - Tin Học Trê...
Quản Lý Hoạt Động Dạy Học Ngoại Ngữ Tại Các Trung Tâm Ngoại Ngữ - Tin Học Trê...Quản Lý Hoạt Động Dạy Học Ngoại Ngữ Tại Các Trung Tâm Ngoại Ngữ - Tin Học Trê...
Quản Lý Hoạt Động Dạy Học Ngoại Ngữ Tại Các Trung Tâm Ngoại Ngữ - Tin Học Trê...
 
Quản Trị Rủi Ro Tín Dụng Trong Cho Vay Doanh Nghiệp Tại Ngân Hàng Thƣơng Mại ...
Quản Trị Rủi Ro Tín Dụng Trong Cho Vay Doanh Nghiệp Tại Ngân Hàng Thƣơng Mại ...Quản Trị Rủi Ro Tín Dụng Trong Cho Vay Doanh Nghiệp Tại Ngân Hàng Thƣơng Mại ...
Quản Trị Rủi Ro Tín Dụng Trong Cho Vay Doanh Nghiệp Tại Ngân Hàng Thƣơng Mại ...
 
Tạo Việc Làm Cho Thanh Niên Trên Địa Bàn Quận Thanh Khê, Thành Phố Đà Nẵng.doc
Tạo Việc Làm Cho Thanh Niên Trên Địa Bàn Quận Thanh Khê, Thành Phố Đà Nẵng.docTạo Việc Làm Cho Thanh Niên Trên Địa Bàn Quận Thanh Khê, Thành Phố Đà Nẵng.doc
Tạo Việc Làm Cho Thanh Niên Trên Địa Bàn Quận Thanh Khê, Thành Phố Đà Nẵng.doc
 
Quản Trị Rủi Ro Tín Dụng Trong Cho Vay Trung Và Dài Hạn Tại Ngân Hàng Thương ...
Quản Trị Rủi Ro Tín Dụng Trong Cho Vay Trung Và Dài Hạn Tại Ngân Hàng Thương ...Quản Trị Rủi Ro Tín Dụng Trong Cho Vay Trung Và Dài Hạn Tại Ngân Hàng Thương ...
Quản Trị Rủi Ro Tín Dụng Trong Cho Vay Trung Và Dài Hạn Tại Ngân Hàng Thương ...
 

Recently uploaded

GIAO TRINH TRIET HOC MAC - LENIN (Quoc gia).pdf
GIAO TRINH TRIET HOC MAC - LENIN (Quoc gia).pdfGIAO TRINH TRIET HOC MAC - LENIN (Quoc gia).pdf
GIAO TRINH TRIET HOC MAC - LENIN (Quoc gia).pdf
LngHu10
 
GIÁO TRÌNH 2-TÀI LIỆU SỬA CHỮA BOARD MONO TỦ LẠNH MÁY GIẶT ĐIỀU HÒA.pdf
GIÁO TRÌNH 2-TÀI LIỆU SỬA CHỮA BOARD MONO TỦ LẠNH MÁY GIẶT ĐIỀU HÒA.pdfGIÁO TRÌNH 2-TÀI LIỆU SỬA CHỮA BOARD MONO TỦ LẠNH MÁY GIẶT ĐIỀU HÒA.pdf
GIÁO TRÌNH 2-TÀI LIỆU SỬA CHỮA BOARD MONO TỦ LẠNH MÁY GIẶT ĐIỀU HÒA.pdf
Điện Lạnh Bách Khoa Hà Nội
 
Nghiên cứu cơ chế và động học phản ứng giữa hợp chất Aniline (C6H5NH2) với gố...
Nghiên cứu cơ chế và động học phản ứng giữa hợp chất Aniline (C6H5NH2) với gố...Nghiên cứu cơ chế và động học phản ứng giữa hợp chất Aniline (C6H5NH2) với gố...
Nghiên cứu cơ chế và động học phản ứng giữa hợp chất Aniline (C6H5NH2) với gố...
Nguyen Thanh Tu Collection
 
DS thi KTHP HK2 (dot 3) nam hoc 2023-2024.pdf
DS thi KTHP HK2 (dot 3) nam hoc 2023-2024.pdfDS thi KTHP HK2 (dot 3) nam hoc 2023-2024.pdf
DS thi KTHP HK2 (dot 3) nam hoc 2023-2024.pdf
thanhluan21
 
98 BÀI LUYỆN NGHE TUYỂN SINH VÀO LỚP 10 TIẾNG ANH DẠNG TRẮC NGHIỆM 4 CÂU TRẢ ...
98 BÀI LUYỆN NGHE TUYỂN SINH VÀO LỚP 10 TIẾNG ANH DẠNG TRẮC NGHIỆM 4 CÂU TRẢ ...98 BÀI LUYỆN NGHE TUYỂN SINH VÀO LỚP 10 TIẾNG ANH DẠNG TRẮC NGHIỆM 4 CÂU TRẢ ...
98 BÀI LUYỆN NGHE TUYỂN SINH VÀO LỚP 10 TIẾNG ANH DẠNG TRẮC NGHIỆM 4 CÂU TRẢ ...
Nguyen Thanh Tu Collection
 
BAI TAP ON HE LOP 2 LEN 3 MON TIENG VIET.pdf
BAI TAP ON HE LOP 2 LEN 3 MON TIENG VIET.pdfBAI TAP ON HE LOP 2 LEN 3 MON TIENG VIET.pdf
BAI TAP ON HE LOP 2 LEN 3 MON TIENG VIET.pdf
phamthuhoai20102005
 
30 - ĐỀ THI HSG - HÓA HỌC 9 - NĂM HỌC 2021 - 2022.pdf
30 - ĐỀ THI HSG - HÓA HỌC 9 - NĂM HỌC 2021 - 2022.pdf30 - ĐỀ THI HSG - HÓA HỌC 9 - NĂM HỌC 2021 - 2022.pdf
30 - ĐỀ THI HSG - HÓA HỌC 9 - NĂM HỌC 2021 - 2022.pdf
ngocnguyensp1
 
Khoá luận tốt nghiệp ngành Truyền thông đa phương tiện Xây dựng kế hoạch truy...
Khoá luận tốt nghiệp ngành Truyền thông đa phương tiện Xây dựng kế hoạch truy...Khoá luận tốt nghiệp ngành Truyền thông đa phương tiện Xây dựng kế hoạch truy...
Khoá luận tốt nghiệp ngành Truyền thông đa phương tiện Xây dựng kế hoạch truy...
https://www.facebook.com/garmentspace
 
Ảnh hưởng của nhân sinh quan Phật giáo đến đời sống tinh thần Việt Nam hiện nay
Ảnh hưởng của nhân sinh quan Phật giáo đến đời sống tinh thần Việt Nam hiện nayẢnh hưởng của nhân sinh quan Phật giáo đến đời sống tinh thần Việt Nam hiện nay
Ảnh hưởng của nhân sinh quan Phật giáo đến đời sống tinh thần Việt Nam hiện nay
chinhkt50
 
CHUYÊN ĐỀ BỒI DƯỠNG HỌC SINH GIỎI KHOA HỌC TỰ NHIÊN 9 CHƯƠNG TRÌNH MỚI - PHẦN...
CHUYÊN ĐỀ BỒI DƯỠNG HỌC SINH GIỎI KHOA HỌC TỰ NHIÊN 9 CHƯƠNG TRÌNH MỚI - PHẦN...CHUYÊN ĐỀ BỒI DƯỠNG HỌC SINH GIỎI KHOA HỌC TỰ NHIÊN 9 CHƯƠNG TRÌNH MỚI - PHẦN...
CHUYÊN ĐỀ BỒI DƯỠNG HỌC SINH GIỎI KHOA HỌC TỰ NHIÊN 9 CHƯƠNG TRÌNH MỚI - PHẦN...
Nguyen Thanh Tu Collection
 
CÁC BIỆN PHÁP KỸ THUẬT AN TOÀN KHI XÃY RA HỎA HOẠN TRONG.pptx
CÁC BIỆN PHÁP KỸ THUẬT AN TOÀN KHI XÃY RA HỎA HOẠN TRONG.pptxCÁC BIỆN PHÁP KỸ THUẬT AN TOÀN KHI XÃY RA HỎA HOẠN TRONG.pptx
CÁC BIỆN PHÁP KỸ THUẬT AN TOÀN KHI XÃY RA HỎA HOẠN TRONG.pptx
CNGTRC3
 

Recently uploaded (11)

GIAO TRINH TRIET HOC MAC - LENIN (Quoc gia).pdf
GIAO TRINH TRIET HOC MAC - LENIN (Quoc gia).pdfGIAO TRINH TRIET HOC MAC - LENIN (Quoc gia).pdf
GIAO TRINH TRIET HOC MAC - LENIN (Quoc gia).pdf
 
GIÁO TRÌNH 2-TÀI LIỆU SỬA CHỮA BOARD MONO TỦ LẠNH MÁY GIẶT ĐIỀU HÒA.pdf
GIÁO TRÌNH 2-TÀI LIỆU SỬA CHỮA BOARD MONO TỦ LẠNH MÁY GIẶT ĐIỀU HÒA.pdfGIÁO TRÌNH 2-TÀI LIỆU SỬA CHỮA BOARD MONO TỦ LẠNH MÁY GIẶT ĐIỀU HÒA.pdf
GIÁO TRÌNH 2-TÀI LIỆU SỬA CHỮA BOARD MONO TỦ LẠNH MÁY GIẶT ĐIỀU HÒA.pdf
 
Nghiên cứu cơ chế và động học phản ứng giữa hợp chất Aniline (C6H5NH2) với gố...
Nghiên cứu cơ chế và động học phản ứng giữa hợp chất Aniline (C6H5NH2) với gố...Nghiên cứu cơ chế và động học phản ứng giữa hợp chất Aniline (C6H5NH2) với gố...
Nghiên cứu cơ chế và động học phản ứng giữa hợp chất Aniline (C6H5NH2) với gố...
 
DS thi KTHP HK2 (dot 3) nam hoc 2023-2024.pdf
DS thi KTHP HK2 (dot 3) nam hoc 2023-2024.pdfDS thi KTHP HK2 (dot 3) nam hoc 2023-2024.pdf
DS thi KTHP HK2 (dot 3) nam hoc 2023-2024.pdf
 
98 BÀI LUYỆN NGHE TUYỂN SINH VÀO LỚP 10 TIẾNG ANH DẠNG TRẮC NGHIỆM 4 CÂU TRẢ ...
98 BÀI LUYỆN NGHE TUYỂN SINH VÀO LỚP 10 TIẾNG ANH DẠNG TRẮC NGHIỆM 4 CÂU TRẢ ...98 BÀI LUYỆN NGHE TUYỂN SINH VÀO LỚP 10 TIẾNG ANH DẠNG TRẮC NGHIỆM 4 CÂU TRẢ ...
98 BÀI LUYỆN NGHE TUYỂN SINH VÀO LỚP 10 TIẾNG ANH DẠNG TRẮC NGHIỆM 4 CÂU TRẢ ...
 
BAI TAP ON HE LOP 2 LEN 3 MON TIENG VIET.pdf
BAI TAP ON HE LOP 2 LEN 3 MON TIENG VIET.pdfBAI TAP ON HE LOP 2 LEN 3 MON TIENG VIET.pdf
BAI TAP ON HE LOP 2 LEN 3 MON TIENG VIET.pdf
 
30 - ĐỀ THI HSG - HÓA HỌC 9 - NĂM HỌC 2021 - 2022.pdf
30 - ĐỀ THI HSG - HÓA HỌC 9 - NĂM HỌC 2021 - 2022.pdf30 - ĐỀ THI HSG - HÓA HỌC 9 - NĂM HỌC 2021 - 2022.pdf
30 - ĐỀ THI HSG - HÓA HỌC 9 - NĂM HỌC 2021 - 2022.pdf
 
Khoá luận tốt nghiệp ngành Truyền thông đa phương tiện Xây dựng kế hoạch truy...
Khoá luận tốt nghiệp ngành Truyền thông đa phương tiện Xây dựng kế hoạch truy...Khoá luận tốt nghiệp ngành Truyền thông đa phương tiện Xây dựng kế hoạch truy...
Khoá luận tốt nghiệp ngành Truyền thông đa phương tiện Xây dựng kế hoạch truy...
 
Ảnh hưởng của nhân sinh quan Phật giáo đến đời sống tinh thần Việt Nam hiện nay
Ảnh hưởng của nhân sinh quan Phật giáo đến đời sống tinh thần Việt Nam hiện nayẢnh hưởng của nhân sinh quan Phật giáo đến đời sống tinh thần Việt Nam hiện nay
Ảnh hưởng của nhân sinh quan Phật giáo đến đời sống tinh thần Việt Nam hiện nay
 
CHUYÊN ĐỀ BỒI DƯỠNG HỌC SINH GIỎI KHOA HỌC TỰ NHIÊN 9 CHƯƠNG TRÌNH MỚI - PHẦN...
CHUYÊN ĐỀ BỒI DƯỠNG HỌC SINH GIỎI KHOA HỌC TỰ NHIÊN 9 CHƯƠNG TRÌNH MỚI - PHẦN...CHUYÊN ĐỀ BỒI DƯỠNG HỌC SINH GIỎI KHOA HỌC TỰ NHIÊN 9 CHƯƠNG TRÌNH MỚI - PHẦN...
CHUYÊN ĐỀ BỒI DƯỠNG HỌC SINH GIỎI KHOA HỌC TỰ NHIÊN 9 CHƯƠNG TRÌNH MỚI - PHẦN...
 
CÁC BIỆN PHÁP KỸ THUẬT AN TOÀN KHI XÃY RA HỎA HOẠN TRONG.pptx
CÁC BIỆN PHÁP KỸ THUẬT AN TOÀN KHI XÃY RA HỎA HOẠN TRONG.pptxCÁC BIỆN PHÁP KỸ THUẬT AN TOÀN KHI XÃY RA HỎA HOẠN TRONG.pptx
CÁC BIỆN PHÁP KỸ THUẬT AN TOÀN KHI XÃY RA HỎA HOẠN TRONG.pptx
 

Luận văn: Một số bài toán về dãy số, HAY, 9đ

  • 1. ĐẠI HỌC QUỐC GIA HÀ NỘI TRƯỜNG ĐẠI HỌC KHOA HỌC TỰ NHIÊN Nguyễn Thành Giáp MỘT SỐ BÀI TOÁN VỀ DÃY SỐ LUẬN VĂN THẠC SĨ KHOA HỌC Hà Nội - 2011
  • 2. ĐẠI HỌC QUỐC GIA HÀ NỘI TRƯỜNG ĐẠI HỌC KHOA HỌC TỰ NHIÊN Nguyễn Thành Giáp MỘT SỐ BÀI TOÁN VỀ DÃY SỐ Chuyên ngành: Phương pháp toán sơ cấp Mã số: 60.46.40 LUẬN VĂN THẠC SĨ KHOA HỌC Người hướng dẫn khoa học: TS. Phạm Văn Quốc Hà Nội - 2011
  • 3. Mục lục Lời nói đầu 3 Chương 1. Một số kiến thức chuẩn bị 5 1.1. Dãy số . . . . . . . . . . . . . . . . . . . . . . . . . . . . . . . . . . . . . . . . . . . . . . . . . . . . . . . . . . . . 5 1.1.1. Định nghĩa . . . . . . . . . . . . . . . . . . . . . . . . . . . . . . . . . . . . . . . . . . . . . . . . . . 5 1.1.2. Cách cho một dãy số . . . . . . . . . . . . . . . . . . . . . . . . . . . . . . . . . . . . . . . . 6 1.1.3. Một vài dãy số đặc biệt . . . . . . . . . . . . . . . . . . . . . . . . . . . . . . . . . . . . . . 6 1.1.4. Giới hạn của dãy số . . . . . . . . . . . . . . . . . . . . . . . . . . . . . . . . . . . . . . . . . 8 1.2. Sơ lược về phương pháp sai phân . . . . . . . . . . . . . . . . . . . . . . . . . . . . . . . . . 11 1.3. Số học . . . . . . . . . . . . . . . . . . . . . . . . . . . . . . . . . . . . . . . . . . . . . . . . . . . . . . . . . . . 14 1.3.1. Đồng dư thức . . . . . . . . . . . . . . . . . . . . . . . . . . . . . . . . . . . . . . . . . . . . . . 14 1.3.2. Một số định lý cơ bản của số học . . . . . . . . . . . . . . . . . . . . . . . . . . . 15 Chương 2. Tính chất số học của dãy số 17 2.1. Tính chia hết . . . . . . . . . . . . . . . . . . . . . . . . . . . . . . . . . . . . . . . . . . . . . . . . . . . . . 17 2.2. Tính chất số nguyên . . . . . . . . . . . . . . . . . . . . . . . . . . . . . . . . . . . . . . . . . . . . . . 36 1
  • 4. 2.3. Tính chính phương . . . . . . . . . . . . . . . . . . . . . . . . . . . . . . . . . . . . . . . . . . . . . . . . 46 2.4. Bài tập . . . . . . . . . . . . . . . . . . . . . . . . . . . . . . . . . . . . . . . . . . . . . . . . . . . . . . . . . . . 57 Chương 3. Giới hạn của dãy số 60 3.1. Giới hạn của tổng . . . . . . . . . . . . . . . . . . . . . . . . . . . . . . . . . . . . . . . . . . . . . . . . . 60 3.2. Dãy con và sự hội tụ của dãy số . . . . . . . . . . . . . . . . . . . . . . . . . . . . . . . . . . . 65 3.3. Dãy số xác định bởi phương trình . . . . . . . . . . . . . . . . . . . . . . . . . . . . . . . . . 73 3.4. Bài tập . . . . . . . . . . . . . . . . . . . . . . . . . . . . . . . . . . . . . . . . . . . . . . . . . . . . . . . . . . . 81 Kết luận 84 Tài liệu tham khảo 85 2
  • 5. Lời nói đầu Dãy số là một lĩnh vực khó và rất rộng, trong các đề thi học sinh giỏi quốc gia, quốc tế cũng thường xuất hiện các bài toán về dãy số. Để giải được các bài toán về dãy số đòi hỏi người làm toán phải có kiến thức tổng hợp về số học, đại số, giải tích. Các vấn đề liên quan đến dãy số cũng rất đa dạng và cũng có nhiều tài liệu viết về vấn đề này, các tài liệu này cũng thường viết khá rộng về các vấn đề của dãy số, các vấn đề được quan tâm nhiều hơn là các tính chất số hoc và tính chất giải tích của dãy số. Tính chất số học của dãy số thể hiện như tính chia hết, tính nguyên, tính chính phương. . . , tính chất giải tích có nhiều dạng nhưng quan trọng là các bài toán tìm giới hạn dãy số. Các bài toán về dãy số thường là các bài toán hay và khó, tác giả luận văn đã sưu tầm, chọn lọc và phân loại theo từng chủ đề. Luận văn với đề tài “Một số bài toán về dãy số” có mục đích trình bày một cách hệ thống, chi tiết tính chất số học của dãy số, giới hạn dãy số. Luận văn được trình bày với 3 chương. Chương 1. Một số kiến thức chuẩn bị. Chương này hệ thống lại kiến thức cơ bản nhất về dãy số, số học, phương pháp sai phân sẽ được dùng để giải quyết các bài toán trong các chương sau. Chương 2. Tính chất số học của dãy số. Chương này trình bày một số vấn đề về tính chất số học của dãy số như tính chia hết, tính nguyên, tính chính phương. . . và nêu ra các phương pháp giải toán, phân tích các bài toán cụ thể. 3
  • 6. Chương 3. Giới hạn của dãy số. Chương này đề cập đến một số bài toán về giới hạn dãy số như: Giới hạn của tổng, dãy con và sự hội tụ của dãy số, dãy số xác định bởi phương trình cùng với phương pháp giải cụ thể cho từng dạng toán. Luận văn được hoàn thành với sự quan tâm giúp đỡ, hướng dẫn khoa học của TS. Phạm Văn Quốc, thày đã tận tình chỉ bảo cách tập nghiên cứu khoa học, cách làm và trình bày bản luận văn này đồng thời thày cũng có nhiều ý kiến quý báu để hoàn thành luận văn. Tác giả xin bày tỏ lòng cảm ơn sâu sắc nhất tới thày. Nhân dịp này tác giả cũng xin cảm ơn khoa Toán – Cơ – Tin học, phòng Sau đại học, phòng Công tác chính trị sinh viên trường Đại học Khoa học Tự nhiên – Đại học Quốc gia Hà nội đã tạo điều kiện giúp đỡ tác giả trong suốt hai năm học cũng như trong quá trình làm luận văn, cảm ơn Ban giám hiệu, các bạn đồng nghiệp trường THPT Nguyễn Trung Ngạn đã giúp đỡ cho tác giả trong công tác và trong học tập thời gian qua, tác giả cũng xin cảm ơn gia đình, bạn bè đã cổ vũ, động viên tác giả vượt qua mọi khó khăn để hoàn thành bản luận văn này. Hà Nội, ngày 25 tháng 11 năm 2011 Học viên Nguyễn Thành Giáp 4
  • 7. Chương 1 Một số kiến thức chuẩn bị 1.1. Dãy số 1.1.1. Định nghĩa Mỗi hàm số u xác định trên tập các số nguyên dương N∗ được gọi là một dãy số vô hạn (gọi tắt là dãy số). Kí hiệu: u :N∗ −→ R n −→ u(n) Dãy số thường được viết dưới dạng khai triển: u1, u2, ..., un, .... Trong đó un = u(n) và gọi u1 là số hạng đầu, un là số hạng thứ n và là số hạng tổng quát của dãy số. Mỗi hàm số u xác định trên tập M = 1, 2, 3, . . . , m với m ∈ N∗ được gọi là một dãy số hữu hạn. Dạng khai triển của nó là u1, u2, ..., um trong đó u1 là số hạng đầu, um là số hạng cuối. Dãy số (un) được gọi là: • Dãy đơn điệu tăng nếu un+1 > un với mọi n = 1, 2, ... 5
  • 8. • Dãy đơn điệu không giảm nếu un+1 ≥ un với mọi n = 1, 2, ... • Dãy đơn điệu giảm nếu un+1 < un với mọi n = 1, 2, ... • Dãy đơn điệu không tăng nếu un+1 ≤ un với mọi n = 1, 2, ... Dãy (un) được gọi là: • Dãy số bị chặn trên nếu tồn tại số M sao cho un < M, với mọi N = 1, 2, ... • Dãy số bị chặn dưới nếu tồn tại số m sao cho un > m, với mọi N = 1, 2, ... • Dãy số bị chặn nếu nó vừa bị chặn trên vừa bị chặn dưới. Dãy số (un) được gọi là tuần hoàn với chu kỳ k nếu un+k = un, với mọi n ∈ N. Dãy số (un) được gọi là dãy dừng nếu tồn tại một số N0 sao cho un = C với mọi n ≥ N0. (C là hằng số, gọi là hằng số dừng) 1.1.2. Cách cho một dãy số Dãy số cho bằng công thức của số hạng tổng quát: Ví dụ xét dãy số (un) được cho bởi un = 1 √ 5 1 + √ 5 2 n − 1 √ 5 1 − √ 5 2 n . Dãy số cho bằng phương pháp truy hồi: Dãy số (un) được xác định bởi    u1 = 1, u2 = 50 un+1 = 4un + 5un−1 − 1975, n = 2, 3, 4, ... Dãy số cho bằng phương pháp mô tả: Ví dụ xét dãy số (un) được cho bởi: a1 = 19, a2 = 98. Với mỗi số nguyên n ≥ 1, xác định an+2 bằng số dư của phép chia an + an+1 cho 100. 6
  • 9. 1.1.3. Một vài dãy số đặc biệt 1.1.3.1. Cấp số cộng Định nghĩa 1.1.1. Dãy số u1, u2, u3, ... được gọi là một cấp số cộng với công sai d (d khác 0) nếu un = un−1 + d với mọi n = 2, 3, ... Tính chất: 1. un = u1 + (n − 1)d. 2. uk = uk−1 + uk+1 2 với mọi k = 2, 3, ... 3. Nếu cấp số cộng có hữu hạn phần tử u1, u2, ..., un thì u1 +un = uk +un−k với mọi k = 2, 3, ..., n − 1 và Sn = u1 + u2 + · · · + un = n 2 (u1 + un) = n 2 [2u1 + (n − 1)d]. 1.1.3.2. Cấp số nhân Định nghĩa 1.1.2. Dãy số u1, u2, u3, ... được gọi là một cấp số nhân với công bội q (q khác 0 và khác 1) nếu un = un−1q với mọi n = 2, 3, ... Tính chất: 1. un = u1qn−1 với mọi n = 2, 3, .... 2. u2 k = uk−1 · uk+1 với mọi k = 2, 3, ... 3. Sn = u1 + u2 + · · · + un = u1(qn − 1) q − 1 . 1.1.3.3. Dãy Fibonacci Định nghĩa 1.1.3. Dãy u1, u2, ... được xác định như sau:    u1 = 1, u2 = 1 un = un−1 + un−2, với mọi n = 2, 3, ... 7
  • 10. được gọi là dãy Fibonacci. Bằng phương pháp sai phân có thể tìm được công thức tổng quát của dãy là: un = 1 √ 5 1 + √ 5 2 n − 1 √ 5 1 − √ 5 2 n . 1.1.4. Giới hạn của dãy số Định nghĩa 1.1.4. Ta nói rằng dãy số (un) có giới hạn là hằng số thực a hữu hạn nếu với mọi số dương ε (có thể bé tùy ý), luôn tồn tại chỉ số n0 ∈ N (n0 có thể phụ thuộc vào ε và vào dãy số (un) đang xét), sao cho với mọi chỉ số n ∈ N, n ≥ n0 ta luôn có |un − a| < ε. Khi đó kí hiệu lim n→+∞ un = a hoặc lim un = a và còn nói rằng dãy số (un) hội tụ về a. Dãy số không hội tụ gọi là dãy phân kì. Định lý 1.1.5. Nếu một dãy số hội tụ thì giới hạn của nó là duy nhất. Định lý 1.1.6 (Tiêu chuẩn hội tụ Weierstrass). a) Một dãy số đơn điệu và bị chặn thì hội tụ. b) Một dãy số tăng và bị chặn trên thì hội tụ. c) Một dãy số giảm và bị chặn dưới thì hội tụ. Định lý 1.1.7. Nếu (un) → a và (vn) ⊂ (un), (vn) = C thì (vn) → a. Định lý 1.1.8 (định lý kẹp giữa về giới hạn). Nếu với mọi n ≥ n0 ta đều có un ≤ xn ≤ vn và lim un = lim vn = a thì lim xn = a. Định lý 1.1.9 (định lý Lagrange). Nếu hàm số f(x) liên tục trên [a; b] và có đạo hàm trong khoảng (a; b) thì tồn tại c ∈ (a; b) thỏa mãn: f(b) − f(a) = f (c)(b − a). Định lý 1.1.10 (định lý trung bình Cesaro). Nếu dãy số (un) có giới hạn hữu hạn là a thì dãy các trung bình cộng u1 + u2 + · · · + un n cũng có giới hạn 8
  • 11. là a. Định lý này có thể phát biểu dưới dạng tương đương sau: Định lý 1.1.11 (định lý Stolz). Nếu lim n→+∞ (un+1 − un) = a thì lim n→+∞ un n = a. Chứng minh. Ta chỉ cần chứng minh cho trường hợp a = 0. Vì lim n→+∞ (un+1 − un) = a nên với mọi ε > 0 luôn tồn tại N0 sao cho với mọi n ≥ N0, ta có |un+1 − un| < ε. Khi đó, với mọi n > N0 ta có un n ≤ 1 n |uN0 | + |uN0+1 − uN0 | + · · · + |un − un−1| < |uN0 | · 1 n + (n − N0) · ε n Giữ N0 cố định, ta có thể tìm được N1 > N0 sao cho 1 N1|uN0 | < ε. Khi đó với mọi n > N1, ta sẽ có un n < 2ε. Vậy nên lim n→+∞ un n = 0. Định lý 1.1.12. Cho f : D → D là hàm liên tục. Khi đó 1) Phương trình f(x) = x có nghiệm ⇔ phường trình fn(x) = x có nghiệm. 2) Gọi α, β là các mút trái, mút phải của D. Biết lim x→α+ [f(x) − x] và lim x→β− [f(x)−x] cùng dương hoặc cùng âm. Khi đó, phương trình f(x) = x có nghiệm duy nhất ⇔ phương trình fn(x) = x có nghiệm duy nhất. Trong đó fn(x) = f(............f (n−1) lần f (x)). Chứng minh. 1) a) nếu x0 là nghiệm của phương trình f(x) = x thì x0 cũng là nghiệm của phương trình fn(x) = x. b) Nếu phương trình f(x) = x vô nghiệm thì f(x)−x > 0 hoặc f(x)−x < 0 với mọi x ∈ D. Do đó fn(x) − x > 0 hoặc fn(x) − x < 0 với mọi x ∈ D nên phương trình fn(x) = x cũng vô nghiệm. 2) a) Giả sử phương trình f(x) = x có nghiệm duy nhất là x0 thì đây cũng 9
  • 12. là một nghiệm của phương trình fn(x) = x. Đặt F(x) = f(x) − x, do F(x) liên tục trên (x0; β) và (α; x0) nên F(x) giữ nguyên một dấu. Nếu lim x→α+ [f(x)−x] và lim x→β− [f(x)−x] cùng dương thì F(x) > 0 trong khoảng (x0; β) và (α; x0), suy ra f(x) > x với mọi x ∈ D {x0}. Xét x1 ∈ D {x0} suy ra f(x1) > x1 ⇒ f(f(x1)) > f(x1) > x1. Từ đó, ta có fn(x1) > x1 nên x1 không là nghiệm của phương trình fn(x) = x. Vậy phương trình fn(x) = x có nghiệm duy nhất x = x0. Trường hợp lim x→α+ [f(x) − x] và lim x→β− [f(x) − x] cùng âm được chứng minh tương tự. b) Ta thấy mọi nghiệm của phương trình f(x) = x đều là nghiệm của phương trình fn(x) = x, do đó nếu phương trình fn(x) = x có nghiệm duy nhất thì phương trình f(x) = x có nghiệm duy nhất. Định lý 1.1.13. Cho hàm f : D → D là hàm đồng biến, dãy (xn) thỏa mãn f(xn) = xn+1 với mọi n ∈ N∗ . Khi đó: a) Nếu x1 < x2 thì dãy (xn) tăng. b) Nếu x1 > x2 thì dãy (xn) giảm. Chứng minh. a) Ta chứng minh bằng phương pháp quy nạp. Với n = 1 ta có x1 < x2, mệnh đề đúng. Giả sử mệnh đề đúng với n = k (k ≥ 1) tức là xk < xk+1. Do f(·) là hàm đồng biến nên f(xk) < f(xk+1), suy ra xk+1 < xk+2. b) Chứng minh tương tự. Định lý 1.1.14. Cho hàm f : D → D là hàm nghịch biến, dãy (xn) thỏa mãn xn+1 = f(xn) với mọi n ∈ N∗ . Khi đó: a) Các dãy (x2n+1) và (x2n) đơn điệu, trong đó có một dãy tăng và một dãy giảm. b) Nếu dãy (xn) bị chặn thì tồn tại α = lim x2n và β = lim x2n+1. 10
  • 13. c) Nếu f(x) liên tục thì α và β là các nghiệm của phương trình f(f(x)) = x. (1.1) Vì vậy nếu (1.1) có nghiệm duy nhất thì α = β và lim xn = α = β. Chứng minh. a) Vì f(x) là hàm nghịch biến nên f(f(x)) là hàm đồng biến. Áp dụng định lý 1.1.6, ta có điều phải chứng minh. b) Suy ra từ a). c) Ta có f(f(x2n)) = f(x2n+1) = x2n+2 và lim f(f(x2n)) = lim x2n+2 = α, lim x2n = α do f(x) liên tục nên f(f(α)) = α. Chứng minh tương tự ta có f(f(β)) = β. Vậy α, β là nghiệm của phương trình f(f(x)) = x. 1.2. Sơ lược về phương pháp sai phân Định nghĩa 1.2.1. Cho hàm số y = f(x) xác định trên R. Đặt xk = x0 + kh (k ∈ N∗ ) với x0 ∈ R, h ∈ R bất kỳ, cho trước. Gọi yk = f(xk), khi đó hiệu số ∆yk := yk+1 − yk, k ∈ N∗ được gọi là sai phân cấp một của hàm số f(x). Hiệu số ∆2 yk := ∆yk+1 − ∆yk = ∆(∆yk) (k ∈ N∗ ) được gọi là sai phân cấp hai của hàm số f(x). Tổng quát ∆i yk := ∆i−1 yk+1 − ∆i−1 yk = ∆(∆i−1 yk), k ∈ N∗ được gọi là sai phân cấp i của hàm số f(x) (i = 1, 2, ..., n, ...) Mệnh đề 1.2.2. Sai phân mọi cấp đều có thể biểu diễn theo các giá trị của hàm số: y0, y1, y2, ..., yn, ... 11
  • 14. Định nghĩa 1.2.3. Phương trình sai phân (cấp k) là một hệ thức tuyến tính chứa sai phân cấp k f(yn, ∆yn, ∆2 yn, ..., ∆k yn) = 0 (1.2) Vì sai phân các cấp có thể biểu diễn theo các giá trị của hàm số nên ta có thể viết phương trình dạng a0yn+k + a1yn+k−1 + · · · + akyn = f(n) (1.3) trong đó a0, a1, ..., ak, f(n) là các giá trị đã biết, còn yn, yn+1, ..., yn+k là các giá trị chưa biết. Hàm số yn thỏa mãn (1.2) gọi là nghiệm của phương trình sai phân tuyến tính (1.2). Phương trình a0yn+k + a1yn+k−1 + · · · + akyn = f(n) được gọi là phương trình sai phân tuyến tính cấp k. Để giải phương trình này, chúng ta làm như sau: Bước 1. Giải phương trình sai phân tuyến tính thuần nhất tương ứng a0yn+k + a1yn+k−1 + · · · + akyn = 0. (1.4) Để giải phương trình trên, ta xét phương trình đặc trưng a0λk + a1λk−1 + · · · + ak−1λ + ak = 0. (*) Khi đó nếu (∗) có k nghiệm thực khác nhau λ1, λ2, ..., λk thì nghiệm tổng quát của (1.4) là ˆyn = c1λn 1 + c2λn 2 + · · · + ckλn k (1.5) trong đó c1, c2, .., ck là các hằng số tùy ý. Nếu (∗) có nghiệm thực λj bội s thì nghiệm tổng quát của (1.4) là ˆyn = s−1 i=1 cj+ini λn j + k i=1,i=j ciλn i . 12
  • 15. Nếu phương trình (∗) có nghiệm phức đơn λj = r(cos θ + i sin θ) thì cũng có nghiệm λj = r(cos θ − i sin θ). Đặt λj+1 = λj. Để thu được công thức tổng quát, trong công thức (1.5) ta thay bộ phận cjλn j +cj+1λn j+1 bởi bộ phận tương ứng cjrn cos nθ + cj+1rn sin nθ. Nếu phương trình (∗) có nghiệm phức bội s: λj = λj+s+1 = ... = λj+2s−1 = r(cos θ − i sin θ). Trong trường hợp này để thu được công thức nghiệm tổng quát, trong công thức (1.5) ta thay bộ phận cjλn j + cj+1λn j+1 + · · · + cj+2s−1λn j+2s−1 bởi bộ phận tương ứng s−1 i=0 cj+ini rn cos nθ+ s−1 i=0 cj+s+ini rn sin nθ Bước 2. Tìm nghiệm tổng quát của phương trình sai phân tuyến tính cấp k. Nghiệm tổng quát có dạng yn = ˆyn + y∗ n trong đó yn là nghiệm của phương trình sai phân tuyến tính cấp k, ˆyn là nghiệm của phương trình sai phân tuyến tính thuần nhất tương ứng và y∗ n là một nghiệm riêng của phương trình không thuần nhất. 13
  • 16. 1.3. Số học 1.3.1. Đồng dư thức 1.3.1.1. Định nghĩa Nếu hai số nguyên a và b khi chia cho m (m = 0) mà có cùng số dư thì ta nói a đồng dư với b theo modun m, kí hiệu là a ≡ b(mod m). Như vậy a ≡ b(mod m) ⇔ a − b chia hết cho m. Hệ thức dạng: a ≡ b(mod m) gọi là một đồng dư thức, a gọi là vế trái của đồng dư thức, b gọi là vế phải, còn m gọi là môđun. 1.3.1.2. Một số tính chất Kí hiệu a, b, c, d, m, ... là các số nguyên dương (Z+ ), ta luôn có: Tính chất 1. • a ≡ a(mod m). • a ≡ b(mod m) ⇔ b ≡ a(mod m). • a ≡ b(mod m) và b ≡ c(mod m) thì a ≡ c(mod m) Tính chất 2. Nếu a ≡ b(mod m) và c ≡ d(mod m) thì: • a ± c ≡ b ± d(mod m), • ac ≡ bd(mod m), • Nếu d là một ước chung của a, b, m thì a d ≡ b d mod m d . Tính chất 3. Nếu a ≡ b(mod m) và c ∈ Z+ thì ac ≡ bc(mod mc). 1.3.1.3. Một số kiến thức liên quan • Với mọi a, b ∈ Z+ (a = b) và n là số tự nhiên: (an − bn ) ...(a − b); 14
  • 17. • Trong n số nguyên liên tiếp (n ≥ 1) có một và chỉ một số chia hết cho n; • Lấy n + 1 số nguyên bất kỳ (n ≥ 1) đem chia cho n thì phải có hai số khi chia cho n có cùng số dư (theo nguyên lý Dirichlet). • Tìm m chữ số tận cùng của số A là tìm số dư khi chia A cho 10m . 1.3.2. Một số định lý cơ bản của số học 1.3.2.1. Định lý Euler Hàm số Euler-µ(m): Cho hàm số µ(m) được xác định như sau • m = 1, ta có µ(m) = 1, • m > 1 thì µ(m) là các số tự nhiên không vượt quá m − 1 và nguyên tố với m. Công thức tính µ(m): Bước 1. Nếu m = pα (p là số nguyên tố, α là số tự nhiên khác 0). Ta có µ(m) = µ(pα ) = pα 1 − 1 p . Bước 2. Nếu m = pα1 1 pα2 2 · · · pαn n (pi là các số nguyên tố, αi là số tự nhiên khác 0). Ta có µ(m) = m 1 − 1 p1 1 − 1 p2 · · · 1 − 1 pn . Định lý 1.3.1 (định lý Euler). Cho m là số tự nhiên khác 0 và a là số nguyên tố với m. Khi ấy, ta có aµ(m) ≡ 1(modm). 15
  • 18. 1.3.2.2. Định lý Fermat Định lý 1.3.2 (định lý Fermat). Cho p là một số nguyên tố và a là một số nguyên không chia hết cho p. Khi ấy, ta có ap−1 ≡ 1(modp). Đối với số nguyên a bất kỳ ta có ap ≡ a(modp). 16
  • 19. Chương 2 Tính chất số học của dãy số Dãy số nguyên là phần quan trọng trong lý thuyết dãy số. Ngoài các vấn đề chung như tìm số hạng tổng quát của dãy số, tìm công thức tính tổng n số hạng đầu tiên. . . các bài toán về dãy số thường quan tâm đến tính chất số học của dãy số như tính chia hết, đồng dư, nguyên tố, chính phương, nguyên tố cùng nhau. . . Các bài toán về dãy số nguyên rất đa dạng, trong nhiều trường hợp dãy số chỉ là cái bề ngoài còn bản chất bài toán là bài toán số học. 2.1. Tính chia hết Một số bài toán về sự chia hết của các số hạng của dãy số thường được giải bằng cách xác định số hạng tổng quát của dãy số sau đó dựa vào các định lý về đồng dư để chứng minh sự chia hết. Việc xác định số hạng tổng quát của dãy số thường được tìm bằng phương pháp sai phân hoặc thông qua dãy số phụ để đưa về phương trình sai phân thuần nhất. 17
  • 20. Bài tập 2.1.1. Dãy số (un) được xác định như sau:    u1 = 1, u2 = 50 un+1 = 4un + 5un−1 − 1975, n = 2, 3, 4, ... Chứng minh rằng u1996 chia hết cho 1997. Lời giải. Ta tìm số hạng tổng quát của dãy số bằng phương pháp sai phân. Công thức truy hồi của dãy là tuyến tính nhưng không thuần nhất, do vậy ta đặt un = vn + c để có công thức truy hồi vn+1 = 4vn + 5vn−1. Ta có vn+1 + c = 4(vn + c) + 5(vn−1 + c) − 1975 ⇔ vn+1 = 4vn + 5vn−1 + 8c − 1975. Do đó c = 1975 8 . Khi đó dãy (vn) được xác định như sau:    v1 = − 1919 8 , v2 = − 1575 8 vn+1 = 4vn + 5vn−1, n = 2, 3, 4, ... Phương trình đặc trưng: x2 − 4x − 5 = 0 có hai nghiệm x1 = 5 và x2 = −1 nên vn = c15n +c2(−1)n . Cho n = 1, n = 2 ta tìm được c1 = − 1747 120 , c2 = 2005 12 . Do đó vn = − 1747 120 · 5n + 2005 12 · (−1)n ⇒ un = − 1747 120 · 5n + 2005 12 · (−1)n + 1975 8 . Với n = 1996, ta có u1996 = − 1747 120 · 51996 + 9935 24 = −1747 · 51996 + 49675 120 . Do 1997 là số nguyên tố nên 51996 ≡ 1(mod1997). Nên −1747 · 51996 + 49675 ≡ 0(mod1997). Lại có 51996 = 25998 ≡ 1(mod3) nên −1747 · 51996 + 49675 ≡ −1747 + 49675 ≡ 0(mod3). 18
  • 21. Và 51996 = 25998 ≡ 1(mod8) nên −1747 · 51996 + 49675 ≡ −1747 + 49675 ≡ 0(mod8). Hơn nữa, rõ ràng −1747 · 51996 + 49675 ≡ 0(mod5) mà (3, 5, 8) = 1 nên suy ra −1747 · 51996 + 49675 ...120. Mặt khác (120, 1997) = 1 nên −1747 · 51996 + 49675 120 chia hết cho 1997, hay là u1996 chia hết cho 1997. Bài tập 2.1.2 (HSG QG 2011). Cho dãy số nguyên (an) được xác định bởi:    a0 = 1, a1 = −1 an = 6an−1 + 5an−2 với mọi n ≥ 2. Chứng minh rằng a2012 − 2010 chia hết cho 2011. Lời giải. Cách 1. Xét dãy số nguyên (bn) được xác định bởi:    b0 = 1, b1 = −1 bn = 6bn−1 + 2016bn−2 với mọi n ≥ 2. Dễ thấy rằng với mọi n ≥ 0, ta có an ≡ bn(mod2011). Phương trình đặc trưng của dãy (bn): x2 − 6x − 2016 = 0 ⇔ (x − 48)(x + 42) = 0 ⇒ x2 = 48, x1 = −42. Suy ra số hạng tổng quát của dãy (bn) có dạng: bn = C1 · (−42)n + C2 · (48)n . Từ các điều kiện ban đầu của dãy (bn), ta có hệ:    C1 + C2 = 1 42C1 − 48C2 = 1 ⇒    C1 = 49 90 C2 = 41 90 . 19
  • 22. Vì vậy bn = 49 · (−42)n + 41 · 48n 90 với mọi n ≥ 0. Vì 2011 là số nguyên tố nên theo định lý Fermat nhỏ, ta có (−42)2010 ≡ 482010 ≡ 1(mod2011). Do đó 90b2012 = 49 · (−42)2012 + 41 · 482012 ≡ 49 · (−42)2 + 41 · 482 ≡ 90b2(mod2011). Suy ra b2012 ≡ b2(mod2011) (vì (90, 2011) = 1). Mà b2 = 6b1 + 2016b0 nên b2012 ≡ 2010(mod2011). Vì thế a2012 ≡ 2010(mod2011). Cách 2. Số hạng tổng quát của dãy (an): an = 1 2 − 2 √ 14 (3 + √ 14)n + 1 2 + 2 √ 14 (3 − √ 14)n . (1) Đặt p = 2011, ta có ap+1 = 1 2 − 2 √ 14 (3 + √ 14)p+1 + 1 2 + 2 √ 14 (3 − √ 14)p+1 . Do (3 + √ 14)p+1 = Ap+1 + Bp+1 · √ 14 và (3 − √ 14)p+1 = Ap+1 − Bp+1 · √ 14 trong đó Ap+1 = p+1 2 i=0 C2i p+1 · 32i · 14 p+1 2 − i (2) và Bp+1 = p+1 2 i=1 C2i−1 p+1 · 32i−1 · 14 p+1 2 − i (3) nên ap+1 = Ap+1 − 4Bp+1. (4) 20
  • 23. Do p nguyên tố nên Ck p ≡ 0(modp) với mọi k = 1, p − 1. Do đó từ Ck p+1 = Ck p + Ck−1 p , suy ra Ck p+1 ≡ 0(modp) với mọi k = 2, p − 1. Vì vậy từ (2) và (3) ta được Ap+1 ≡ 14 p+1 2 + 3p+1 (modp). Và Bp+1 ≡ 3(p + 1) 14 p−1 2 + 3p−1 ≡ 3 14 p−1 2 + 3p−1 (modp). (5) Do đó từ (4) suy ra ap+1 ≡ − 3p + 2 · 14 p−1 2 (modp). Mặt khác, ta có 452 ≡ 14(modp) và (45, 14) = 1, theo định lý Fermat nhỏ ta có: 3p ≡ 3(modp) và 14 p−1 2 ≡ 45p−1 ≡ 1(modp). Do đó từ (5) ta được a2012 = ap+1 ≡ −3 + 2 = −1 ≡ 2010(mod2011). Việc tìm số hạng tổng quát của dãy số cũng có thể thông qua biến đổi liên tiếp các số hạng phụ thuộc nhau và biểu diễn qua một vài số hạng đầu và có thể áp dụng phương pháp quy nạp để chứng minh. Bài tập 2.1.3. Cho dãy số (un) xác định như sau:    u1 = 3, u2 = 14 nun+2 − (5n + 1)un+1 + 4(n + 1)un = 1 (n ∈ N∗ ) Đặt S = 3 2011 n=1 un − 4(24022 − 1). Chứng minh rằng S chia hết cho 2011. Lời giải. Ta có nun+2 − (5n + 1)un+1 + 4(n + 1)un = 1 ⇒ n[un+2 − 4un+1 − (n + 1) + 1] = (n + 1)(un+1 − 4un − n + 1) 21
  • 24. Do đó un+2 − 4un+1 − (n + 1) + 1 = n + 1 n (un+1 − 4un − n + 1) = n + 1 n · n n − 1 (un − 4un−1 − (n − 1) + 1) = n + 1 n · n n − 1 · · · 2 1 (u2 − 4u1 − 1 + 1) = 2(n + 1) Suy ra un+2 − 4un+1 = 3n + 2 ⇒ un+2 + (n + 2) = 4[un+1 + (n + 1)] = 42 (un + n) = ... = 4n (u2 + 2) = 4n+2 nên un = 4n − n đúng với mọi n ∈ N∗ . Từ đó S = 3 2011 n=1 un − 4(24022 − 1) = 3 2011 n=1 (4n − n) − 4(24022 − 1) = 3 2011 n=1 4n − 3 2011 n=1 n − 4(42011 − 1) = 4(42011 − 1) − 3 · 2011 · 1006 − 4(42011 − 1) = −3 · 2011 · 1006 ... 2011 Một số dạng toán liên quan đến tính chia hết có thể cho ở dạng tìm số dư trong phép chia các số hạng của dãy số cho một số nào đó, ta thường sử dụng các kiến thức về đồng dư để giải toán. Bài tập 2.1.4. Cho a1 = 19, a2 = 98. Với mỗi số nguyên n ≥ 1, ta xác định an+2 bằng số dư của phép chia an + an+1 cho 100. Tìm số dư trong phép chia a2 1 + a2 2 + · · · + a2 1998 cho 8. Lời giải. Gọi rn là số dư của phép chia an cho 4. Theo giả thiết, ta có an + an+1 ≡ an+2(mod100) 22
  • 25. nên an + an+1 ≡ an+2(mod4). Mặt khác rn ∈ {0, 1, 2, 3}, tức là dãy {rn} bị chặn và do đó dãy {rn} tuần hoàn. Ta tính được r1 = 3, r2 = 2, r3 = 1, r4 = 3, r5 = 0, r6 = 3, r7 = 3, r8 = 2, r9 = 1. Dễ kiểm tra {rn} có chu kỳ 6, nghĩa là rn = rn+6 với mọi n ≥ 1. Lại có a2 n − r2 n = (an − rn)(an + rn), do (an − rn) ... 4 nên an, rn đồng tính chẵn lẻ. Từ đó suy ra (an + rn) ... 2, suy ra (a2 n − r2 n) ... 8 hay a2 n ≡ r2 n(mod8). Vậy a2 1 + a2 2 + · · · + a2 1998 ≡ r2 1 + r2 2 + · · · + r2 1998(mod8). Mà r2 1 + r2 2 + · · · + r2 1998 = 333(r2 1 + r2 2 + r2 3 + r2 4 + r2 5 + r2 6) = 333(9 + 4 + 1 + 9 + 0 + 9) ... 8. Do đó a2 1 + a2 2 + · · · + a2 1998 chia hết cho 8 hay số dư bằng 0. Bài tập 2.1.5. Cho λ là nghiệm dương của phương trình t2 − 1998t − 1 = 0 và dãy (xn) được xác định như sau:    x0 = 1 xn+1 = [λxn], với mọi n ≥ 0. Trong đó [x] là số nguyên lớn nhất không vượt quá x. Tìm số dư của phép chia x1998 cho 1998. Lời giải. Ta có 1998 < λ = 999+ √ 9992 + 1 < 1999 nên x1 = 1998, x2 2 = 19982 . Vì λ là nghiệm dương của phương trình t2 − 1998t − 1 = 0 nên λ là một số vô tỷ và λ2 = 1998λ + 1 ⇒ λ = 1998 + 1 λ ⇒ λxn = 1998xn + xn λ . Dễ thấy xn là số nguyên dương, do đó [λxn] = 1998xn + xn λ = 1998xn+ xn λ . 23
  • 26. Vì xn = [λxn−1] nên xn < λxn−1 < xn +1 (do λ là số vô tỷ nên λxn−1 /∈ Z), suy ra xn λ < xn−1 < xn λ + 1 λ . Do λ > 1998 nên xn λ = xn−1 − 1 nên xn+1 = [λxn] = 1998xn + xn λ = 1998xn + xn−1 − 1. Suy ra xn+1 ≡ xn−1 − 1(mod1998) ⇒ x1998 ≡ x0 − 999 ≡ 1000(mod1998). Bài toán trên có thể tổng quát hóa thành bài toán sau: Cho m, p là hai số nguyên dương sao cho m2 + 4p không là số chính phương và m ≥ p. Gọi λ là nghiệm dương của phương trình x2 − mx − p = 0, dãy (xn) được xác định như sau:    x0 = a xn+1 = [λxn], với mọi n ≥ 0, với a nguyên dương. Tìm số dư của xn khi chia cho m. Lời giải. Vì m2 +4p không là số chính phương nên λ là số vô tỷ. Ta có λ = m + p λ nên λxn = mxn + pxn λ . Dễ thấy dãy đã cho là dãy số nguyên nên xn+1 = [λxn] = mxn + pxn λ = mxn+ pxn λ . Vì xn = [λxn−1] nên xn < λxn−1 < xn + 1 ⇒ xn λ < xn−1 < xn λ + 1 λ ⇒ xn−1 − 1 λ < xn λ < xn−1 ⇒ pxn−1 − p λ < pxn λ < pxn−1. Dễ thấy phương trình x2 − mx − p = 0 luôn có hai nghiệm trái dấu và tổng của chúng bằng m nên nghiệm dương λ > m suy ra λ > p hay p λ < 1. Nên pxn−1 − 1 < pxn−1 − p λ ⇒ pxn−1 − 1 < pxn λ < pxn−1. Khi đó pxn λ = pxn−1 − 1. 24
  • 27. Vậy xn+1 = [λxn] = mxn + pxn λ = mxn+ pxn λ = mxn + pxn−1 − 1. Do đó xn+1 ≡ pxn−1 − 1(modm) ≡ p(pxn−3 − 1) − 1 = p2 xn−3 − (p + 1)(modm) ......................... ≡ pk+1 xn−(2k+1) − (pk + · · · + p + 1)(modm). Nếu n lẻ: n = 2k + 1 ta có xn+1 = x2k+2 ≡ pk+1 · x0 − (pk + · · · + p + 1) = pk+1 · x0 − pk+1 − 1 p − 1 (modm). Nếu n chẵn: n = 2k + 2 ta có x2k+3 ≡ pk+1 · x1 − (pk + · · · + p + 1) = pk+1 · x1 − pk+1 − 1 p − 1 (modm) trong đó x1 = [λx0]. Nhận xét. Bài toán này có thể đặc biệt hóa với các giá trị cụ thể của m, p khác nhau thỏa mãn m2 +4p không là số chính phương và m ≥ p, từ đó ta được nhiều bài toán cụ thể tương tự bài 2.1.5. Chẳng hạn chọn m = 1, p = 1 hoặc m = 1000, p=3. Bài tập 2.1.6. Cho dãy (an) xác định bởi:    a0 = 9 an+1 = 27a28 n + 28a27 n , với mọi n = 0, 1, 2, ... Chứng minh rằng số a11 có tận cùng nhiều hơn 2000 chữ số 9 (viết trong hệ thập phân). Lời giải. Ta có an+1 + 1 = 27a28 n + 28a27 n + 1 = 27a27 n (an + 1) + a27 n + 1 = (an + 1)(27a27 n + a26 n − a25 n + a24 n − ... + a2 n − an + 1) = (an + 1)[27(a27 n + 1) + (a26 n − 1) − (a25 n − 1) + · · · + (a2 n − 1) − (an + 1)]. 25
  • 28. Do (a27 n + 1) ... (an + 1); (a26 n − 1) ... (an + 1); (a25 n + 1) ... (an + 1) ... nên (an+1 + 1) ... (an + 1)2 . (1) Với a0 = 9 và (1) cho n lấy giá trị từ 1 đến 10, ta suy ra (a11 + 1) ... 10211 hay (a11 + 1) ... 102048 , từ đó suy ra a11 có tận cùng nhiều hơn 2000 chữ số 9. Một số bài toán về dãy số liên quan đến việc tìm điều kiện cho các chỉ số để số hạng nào đó của dãy số chia hết cho một số cho trước, với dạng toán này ta thường phải tìm số hạng tổng quát. Bài tập 2.1.7. Dãy số (un) được xác định như sau:    u1 = 1, u2 = 2, u3 = 40 un = 10u2 n−1 · un−3 − 24un−1 · u2 n−2 un−2 · un−3 , n = 4, 5, ... Tìm số n nhỏ nhất để un ... 2048. Lời giải. Nhận xét: Công thức truy hồi của dãy số rất phức tạp, tuy nhiên nếu đặt dãy số phụ ta sẽ đưa được về dạng tuyến tính cấp hai. Từ công thức truy hồi của dãy, ta có un un−1 = 10un−1un−3 − 24u2 n−2 un−2un−3 = 10un−1 un−2 − 24un−2 un−3 . Do vậy, ta đặt vn = un un−1 thì dãy (vn) được xác định như sau:    v2 = 2, v3 = 20 vn = 10vn−1 − 24vn−2, n = 4, 5, ... Phương trình đặc trưng x2 − 10x + 24 = 0 có hai nghiệm là x1 = 6, x2 = 4 nên vn = C1 · 6n + C2 · 4n . 26
  • 29. Cho n = 2, n = 3 ta được C1 = 1 6 , C2 = − 1 4 . Do đó vn = 6n−1 − 4n−1 . Vậy un = vn · vn−1 · · · v2 = (6n−1 − 4n−1 )(6n−2 − 4n−2 ) · · · (6 − 4) = 2n−1 · 2n−2 · · · 2 · (3n−1 − 2n−1 )(3n−2 − 2n−2 ) · · · (3 − 2) = 2 (n−1)n 2 · (3n−1 − 2n−1 )(3n−2 − 2n−2 ) · · · (3 − 2). Do (3n−1 − 2n−1 )(3n−2 − 2n−2 ) · · · (3 − 2) là số lể nên để un ... 2048 thì 2 (n−1)n 2 ... 2048 hay 2 (n−1)n 2 ... 211 . Do đó (n − 1)n 2 ≥ 11 ⇒ n ≥ 6. Vậy n = 6 là giá trị cần tìm. Bài tập 2.1.8. Cho dãy số nguyên (an) thỏa mãn: (n − 1)an+1 = (n + 1)an − 2(n − 1) với mọi n ∈ N∗ . Biết a1999 ... 2000. Tìm số n nhỏ nhất sao cho an (n ≥ 2) chia hết cho 2000. Lời giải. Chia hai vế của đẳng thức cho (n − 1)(n + 1)n ta có an−1 n(n + 1) = an (n − 1)n − 2 n(n + 1) . (1) Đặt an (n − 1)n = bn thì (1) có dạng bn+1 = bn − 2 n(n + 1) ⇔ bn+1 − 2 n + 1 = bn − 2 n với mọi n ∈ N∗ . Do đó bn − 2 n = b2 − 2 2 = b2 − 1 hay bn = b2 − 1 + 2 n nên an = 2 n + b2 − 1 n(n − 1) = 2(n − 1)+ a2 2 − 1 n(n − 1). Theo giả thiết 2000 là ước của a1999 = 1998 2 + 1999 a2 2 − 1 . Tức là 1000 là ước của 2 + 1999 a2 2 − 1 do đó a2 2 ∈ Z. Hay 1000 là ước của 2 + 2000 a2 2 − 1 − a2 2 − 1 ⇔ 1000 là ước của 2− a2 2 − 1 . 27
  • 30. Vì thế a2 2 − 1 = 1000m + 2, m ∈ Z. Suy ra an = 2(n − 1) + (1000m + 2)n(n − 1). Vì n(n − 1) là số chẵn nên an chia hết cho 2000 khi và chỉ khi (n − 1) + n(n − 1) = M · 1000 ⇔ (n − 1)(n + 1) = 1000M ⇒ n = 2k + 1. Vậy k(k + 1) = 250M. Nên số k nhỏ nhất là 124, do đó n nhỏ nhất là 249. Bài tập 2.1.9 (HSG QG 1998). Cho dãy số nguyên dương (an), n = 0, 1, 2, ... được xác định bởi:    a0 = 20, a1 = 100 an+2 = 4an+1 + 5an + 20 (1) với n = 0, 1, 2, ... Tìm số nguyên dương h nhỏ nhất có tính chất an+h − an chia hết cho 1998 với mọi n ∈ N. Lời giải. Trước hết ta tìm số hạng tổng quát của dãy. Đặt an = bn + t với mọi n. Thay vào (1) ta được bn+2 = 4bn+1 + 5bn + 8t + 20. Ta chọn t để 8t + 20 = 0 ⇔ t = − 5 2 . Từ đó an = bn − 5 2 và bn+2 − 4bn+1 − 5bn = 0. Phương trình đặc trưng x2 − 4x − 5 = 0 có hai nghiệm là 5 và −1, từ đó có công thức tổng quát bn = c1 · 5n + c2 · (−1)n . Cho n = 0, n = 1 ta tính được c1 = 125 6 , c2 = 5 3 . Vậy an = 125 6 · 5n + 5 3 · (−1)n − 5 2 . (2) Giả sử h là số nguyên dương thỏa mãn an+h ≡ an(mod1998) (với mọi n ∈ N). (3) 28
  • 31. Từ a0 = 20, a1 = 100 suy ra h ≥ 2. Ta sẽ chứng minh h thỏa mãn (3) khi và chỉ khi h chẵn, h ≥ 2 và ah−1 ≡ 0(mod1998) (4) Điều kiện cần. ah = a0+h ≡ a0 = 20(mod1998). (5) Và ah+1 ≡ a1 = 100(mod1998). Từ 5ah−1 = ah+1 − 4ah − 20 ≡ 0(mod 1998) suy ra ah−1 ≡ 0(mod1998) vì (5, 1998) = 1. Nếu h lẻ thì h − 1 chẵn, từ (2) ta có ah = 5ah−1 ≡ 0(mod1998) mẫu thuẫn với ah ≡ 20(mod1998) theo (5). Vậy h phải chẵn, h ≥ 2 và ah−1 ≡ 0(mod1998). Điều kiện đủ. Giả sử h ≥ 2 và ah−1 ≡ 0(mod1998). Khi đó theo (2) có 5ah−2 = ah−1 ≡ 0(mod1998). Từ (1) có ah = 4ah−1 + 5ah−2 + 20 ≡ 20a0(mod1998) ah+1 = 4ah + 5ah−1 + 20 ≡ a1(mod1998). Bằng chứng minh quy nạp dễ dàng suy ra an+h = an(mod1998). Từ (3) ⇔ (4) có h chẵn, h ≥ 2 và ah−1 ≡ 0(mod1998). Vì h − 1 lẻ theo (2) ta có ah−1 = 52 6 (5h − 1) ≡ 0(mod1998) suy ra (5h − 1) ≡ 0(mod1998) vì (5, 1998) = 0 (4) ⇔    5h ≡ 1(mod6 · 1998) h chẵn 1998 = 2 · 33 · 37 ⇒ 6 · 1998 = 22 · 34 · 37 nên áp dụng định lý Euler có 5h ≡ 1(mod22 ) thỏa mãn với mọi h 5h ≡ 1(mod34 ) ⇔ h ... 2 · 33 = 54 5h ≡ 1(mod37) ⇔ h ... 36 29
  • 32. (4) ⇔ h chia hết cho [54, 36] = 108. Vậy giá trị nhỏ nhất của h thỏa mãn là 108. Các bài toán chứng minh dãy số có vô hạn số hạng chia hết cho một số cho trước thường được chứng minh số dư trong phép chia là hữu hạn và do đó tuần hoàn dẫn đến có vô hạn số hạng chia hết cho số đã cho. Bài tập 2.1.10. Dãy số (xn) (n = 0, 1, 2, ...) được xác định như sau:    x0 = a, x1 = b xn+1 = 5x2 n − 3xn−1 với mọi n ≥ 2. Chứng minh rằng với mọi cách chọn các số nguyên a, b tùy ý thì dãy trên hoặc không có số hạng nào chia hết cho 1997, hoặc có vô hạn số hạng chia hết cho 1997. Lời giải. Với mỗi i ∈ N gọi ri là số dư phép chia xi cho 1997 (0 ≤ ri < 1997). Xét dãy các cặp số (r0, r1), (r1, r2), ...Vì tập giá trị của dãy này hữu hạn nên tồn tại cặp số (i, j) với i, j ∈ N∗ , sao cho ri = rj, ri+1 = rj+1 hay xi ≡ xj(mod1997), xi+1 ≡ xj+1(mod1997). Từ cách xác định xn suy ra với i, j như thế thì xi+k ≡ xj+k(mod1997) với mọi k ∈ N. (1) Mặt khác cũng từ công thức truy hồi suy ra 3xi−1 ≡ 3xj−1(mod1997) suy ra xi−1 ≡ xj−1(mod1997). Tiếp tục như vậy ta được xi−k ≡ xj−k(mod1997) với mọi k mà 0 ≤ k ≤ i. (2) Từ (1) và (2) và xj+k = xi+k+j−i với mọi k ≥ 0 suy ra xn ≡ xn+T (mod1997) với mọi n ∈ N, T = j − i. (3) • Nếu tồn tại 0 ≤ r ≤ T − 1 để xr ≡ 0(mod1997) thì xn ≡ 0(mod1997) với mọi n = mT + r (m ∈ N). • Nếu trong tập {x0, x1, ..., xT −1} không có số nào chia hết cho 1997 thì mọi n = mT + r với 0 ≤ r ≤ T − 1 ta có xn không chia hết cho 1997. 30
  • 33. Bài toán này có thể tổng quát: Cho trước A ∈ N∗ và dãy (xn) (n = 0, 1, 2, ...) được xác định bởi x0 = a0, ..., xm = am và thỏa mãn quan hệ b0xn+1 = b1xk1 n + b2xk2 n−1 + · · · + bmxkm n−m+1 + bm+1xn−m trong đó ai ∈ Z, bi ∈ Z, ki ∈ N mà (bi, A) = 1 với i ∈ {0, 1, ..., m} và n ≥ m. Chứng minh rằng khi đó hoặc trong dãy không có số nào chia hết cho A, hoặc có vô hạn số chia hết cho A. Bài tập 2.1.11. Cho dãy số (an) xác định bởi: a1 = 824, a2 = 67 an+1 = a2 n − 3an−1, với mọi n ≥ 2. Chứng minh rằng tồn tại vô hạn số hạng của dãy chia hết cho 2017. Lời giải. Ta chứng minh dãy số dư của dãy (an) theo mod2017 là dãy tuần hoàn. Thật vậy, ta nói bộ (x, y) ≡ (a, b)(mod2017) khi và chỉ khi:    x ≡ a(mod2017) y ≡ b(mod2017). Rõ ràng một bộ (x, y) bất kỳ chỉ có thể đồng dư với một bộ trong tập {(i, j) : 0 ≤ i, j ≤ 2016}. Ta xét 20172 + 1 bộ: (a1, a2), (a2, a3), ..., (a20172+1, a20172+2). Theo nguyên lý Dirichlet, tồn tại hai trong số những bộ này đồng dư với nhau, nghĩa là có i khác j trong N∗ sao cho: (ai, ai+1) ≡ (aj, aj+1). Ta gọi tập hợp những bộ số (i, j) như vậy là tập A. Ta tìm bộ số thuộc A có chỉ số i nhỏ nhất là (i0, j0), ta chứng minh i0 = 1. Thật vậy, giả sử i0 ≥ 2, ta có 3ai0−1 = a2 i0 − ai0+1 ≡ a2 j0 − aj0+1 = 3aj0−1. Vậy (ai0−1, ai0 ) ≡ (aj0−1, aj0 ) nên suy ra bộ (i0 − 1, j0 − 1) cũng thuộc A. 31
  • 34. Điều này mâu thuẫn cách chọn bộ (i0, j0). Vậy tồn tại T ∈ N∗ sao cho    a1 ≡ a1+T (mod2011) a2 ≡ a2+T (mod2011). Suy ra a3 = a2 2 − 3a1 ≡ a2 2+T − 3a1+T = a3+T a4 = a2 3 − 3a2 ≡ a2 3+T − 3a2+T = a4+T ...................... an = a2 n−1 − 3an−2 ≡ a2 n−1+T − 3an−2+T = an+T , ∀ n ≥ 3. Vậy ta có an ≡ an+T với mọi n ∈ N∗ . Mặt khác, ta lại có a3 = 672 − 3 · 824 = 2017 ≡ 0(mod2017) nên suy ra a3+kT ≡ a3 ≡ 0(mod2017). Vậy trong dãy đã cho có vô số phần tử chia hết cho 2017. Bài tập 2.1.12 (HSG QG 1989). Xét dãy số Fibonaxi 1, 1, 2, 3, 5, 8, ... Đặt f(n) = 1985n2 + 1956n + 1960. a) Chứng minh rằng tồn tại vô hạn số F của dãy trên sao cho f(F) chia hết cho 1989. b) Tồn tại hay không một số G của dãy trên sao cho f(G) + 2 chia hết cho 1989? Lời giải. a) Đặt g(n) = 4n2 + 33n + 29. Ta có g(n) = 1989(n2 + n + 1) − f(n). Do đó f(n) ... 1989 ⇔ g(n) ... 1989. Xét dãy số sau đây: −1, 1, 0, 1, 1, 2, ... 32
  • 35. F0 = −1, F1 = 1, ..., Fn+1 = Fn + Fn−1 với mọi n ≥ 1 (thêm số −1, 1, 0 vào trước dãy Fibonaxi). Gọi ri là số dư của Fi khi chia cho 1989 (0 ≤ ri ≤ 1988). Xét dãy các cặp số sau: (r0, r1), (r1, r2), (r2, r3), ...Theo nguyên lý Dirichlet trong 19892 + 1 cặp đầu tiên, ít nhất phải có hai cặp trùng nhau. Giả sử (rp, rp+1) = (rp+α, rp+α+1) (p, α ∈ N). Tức là ta có rp = rp+α, rp+1 = rp+α+1. Từ công thức Fn−1 = Fn+1 − Fn, ta suy ra rp−1 = rp+α−1, rp−2 = rp+α−2, ..., r2 = rα+2, r1 = rα+1, r0 = rα. Từ r0 = rα, r1 = rα+1 và Fn+1 = Fn + Fn−1, ta suy ra ri = rα với mọi i = 0, 1, 2... Do đó ta có r0 = rα = r2α = r3α = ... = rkα, ∀ k ≥ 1. Từ đây ta có g(Fkα) = 1989A = g(−1) = 1989A. Mặt khác Fkα với k = 1, 2, 3 đều là các số Fibonaxi, nên ta suy ra có vô hạn số Fibonaxi F thỏa mãn f(F) chia hết cho 1989. b) Bây giờ ta chứng minh với mọi n nguyên thì số 4n2 + 7n + 1 không chia hết cho 13. Thật vậy, ta có: 16(4n2 + 7n + 1) = (8n + 7)2 − 7 − 13 · 2. Đặt 8n + 7 = 13t ± r (0 ≤ r ≤ 6), t và r là các số nguyên, ta có: (8n + 7)2 = 13(13t2 ± 2tr) + r2 do đó 16(4n2 + 7n + 1) = r2 − 7 + 13m (m là số nguyên nào đó). Thử trực tiếp các giá trị của r = 0, 1, 2, 3, 4, 5, 6 ta đều thấy r2 − 7 không chia hết cho 13. Vậy 4n2 + 7n + 1 không chia hết cho 13 với mọi n nguyên. Mặt khác f(n) + 2 = 1989(n2 + n + 1) − 26(n + 1) − (4n2 + 7n + 1). Do 1989 ... 13 nên f(n) + 2 không chia hết cho 13 với mọi n. Vì vậy không tồn tại số Fibonaxi F nào để f(F) + 2 chia hết cho 1989. 33
  • 36. Bài tập 2.1.13. Cho dãy số (bn) (n = 1, 2, ...) xác định bởi:    b1 = 0, b2 = 14, b3 = −18 bn+1 = 7bn−1 − 6bn−2, với mọi n ≥ 3. Chứng minh rằng với mọi số nguyên tố p, ta luôn có bp chia hết cho p. Lời giải. Ta có phương trình đặc trưng x3 − 7x + 6 = 0 có nghiệm x = 1, x = 2 và x = −3. Từ đó suy ra bn = a · 1n + b · 2n + c · (−3)n (1) với mọi n ≥ 1. Lần lượt cho n = 1, 2, 3 theo giả thiết ta có    a + 2b − 3c = 0 a + 4b + 9c = 14 a + 8b − 27c = −18 ⇔ a = b = c = 1. Vậy bn = 1 + 2n + (−3)n . (2) Với p nguyên tố, theo định lý nhỏ Fermat, ta có 2p ≡ 2(modp) và (−3)p ≡ −3(modp). Do đó từ (2) ta có bp ≡ 0(modp) với mọi số nguyên tố p hay bp ... p với mọi số nguyên tố p. Bài tập 2.1.14. Cho dãy số (an) xác định bởi:    a1 = a2 = 1 an+2 = an+1 + an, n ≥ 1. Tìm tất cả các số nguyên dương a, b với a < b thỏa mãn điều kiện an − 2nan chia hết cho b với mọi n ≥ 1. 34
  • 37. Lời giải. Ta có a3 = 2. Do an ≡ 2nan (modb) nên    a1 ≡ 2a(modb) a3 ≡ 6a3 (modb) ⇒    2a ≡ 1(modb) 6a3 ≡ 2(modb) ⇒    2a ≡ 1(modb) 3(2a)3 ≡ 8(modb) ⇒    2a ≡ 1(modb) 5 ≡ 0(modb) ⇒    2a ≡ 1(modb) b = 5 ⇒    a = 3 b = 5. Bây giờ ta chứng minh cặp (a; b) = (3; 5) thỏa mãn điều kiện đề bài. Đặt bn = an − 2n3n . Do an+2 = an+1 + an nên bn+2 = bn+1 + bn − 10 · 3n · (n + 3) với mọi n ≥ 1, suy ra bn+2 = bn+1 + bn(mod5). Mà b1 = −5 ≡ 0(mod5) với mọi n ≥ 1, b2 = −35 ≡ 0(mod5). Nên từ hệ thức trên suy ra bn ≡ 0(mod5) với mọi n ≥ 1. Hay an − 2nan chia hết cho 5 với mọi n ≥ 1. Bài tập 2.1.15. Cho dãy số (an) (n = 0, 1, 2, ...) được xác định bởi:    a0 = 29, a1 = 105, a2 = 381 an+3 = 3an+2 + 2an+1 + an với mọi n = 0, 1, 2, ... Chứng minh rằng với mỗi số nguyên dương m luôn tồn tại số tự nhiên n sao cho các số an, an+1 − 1, an+2 − 2 đều chia hết cho m. Lời giải. Từ công thức truy hồi ta tìm được a−1 = 8, a−2 = 2, a−3 = 1, a−4 = 0. Giả sử an ≡ rn(modm) trong đó 0 ≤ rn < m. Xét các bộ ba (rn, rn+1, rn+2) (n ∈ Z). Vì các bộ ba là vô hạn mà tập giá trị này là hữu hạn (không vượt quá m3 ) nên tồn tại p, q mà p < q ∈ Z sao cho (rp, rp+1, rp+2) = (rq, rq+1, rq+2). Tức là    rp = rq rp+1 = rq+1 rp+2 = rq+2 ⇔    ap ≡ aq(modm) ap+1 ≡ aq+1(modm) ap+2 ≡ aq+2(modm). 35
  • 38. Từ công thức truy hồi và hệ đồng dư trên ta dễ dàng suy ra aq+k ≡ ap+k(modm) ∀k ∈ Z ⇔ ak ≡ ap−q+k(modm) ∀k ∈ Z ⇔ ak ≡ at+k(modm), t = q − p ∈ N∗ . Từ đó ak ≡ aet+k(modm) với mọi e ∈ N∗ . Nói riêng    alt−4 ≡ a−4 ≡ 0(modm) alt−3 ≡ a−3 ≡ 1(modm) alt−2 ≡ a−2 ≡ 2(modm). Cho n đủ lớn ta có lt − 4 ∈ N. Do đó tồn tại n = lt − 4 ∈ N để an ≡ 0(modm), an+1 ≡ 1(modm) và an+2 ≡ 2(modm). 2.2. Tính chất số nguyên Các bài toán chứng minh dãy số gồm toàn các số nguyên được đưa về công thức truy hồi tuyến tính sau đó chứng minh bằng phương pháp quy nạp với một vài số hạng đầu là số nguyên. Bài tập 2.2.1. Cho dãy số (an) được xác định bởi:    a1 = 2 an+1 = 3an + 4 + 8a2 n Chứng minh rằng dãy số gồm toàn các số nguyên. Lời giải. Từ giả thiết, ta có a2 = 12, an+1 > 3an và (an+1 − 3an)2 = 4 + 8a2 n, an > 0 với mọi n. Suy ra a2 n+1 − 6an+1an + a2 n = 4. (1) 36
  • 39. Thay n bởi n + 1 ta được a2 n+2 − 6an+2an+1 + a2 n+1 = 4. (2) Trừ hai vế của (1) và (2) ta được: a2 n+2 − a2 n − 6an+1(an+2 − an) = 0 ⇔ (an+2 − an)(an+2 − 6an+1 + an) = 0. Vì an+2 > an nên an+2 − 6an+1 + an = 0 (∀ n ∈ N∗ ). (3) Vì a1, a2 ∈ Z và do (3) nên an ∈ Z với mọi n ∈ N∗ . (Chứng minh bằng phương pháp quy nạp) Bài tập 2.2.2. Cho ba số nguyên a, b, c thỏa mãn điều kiện a2 = b + 1. Dãy số (un) được xác định như sau:    u0 = 0 un+1 = aun + bu2 n + c2, n = 0, 1, 2, ... Chứng minh rằng mọi số hạng của dãy số trên đều là số nguyên. Lời giải. Từ giả thiết ta có un+1 − aun = bu2 n + c2, n = 0, 1, 2, ...Suy ra u2 n+1 − 2aunun+1 + a2 u2 n = bu2 n + c2 . (1) Với giả thiết a2 = b + 1 thì (1) suy ra (a2 − b)u2 n+1 − 2aunun+1 + a2 u2 n = (a2 − 1)u2 n + c2 hay u2 n + 2aunun+1 + a2 u2 n+1 = bu2 n+1 + c2 . (2) Từ giả thiết ta có bu2 n+1 + c2 = (un+2 − aun+1)2 nên (2) suy ra (un − aun+1)2 = (un+2 − aun+1)2 . (3) 37
  • 40. Do un+2 − aun+1 ≥ 0 nên (3) suy ra un+2 − aun+1 = |un − an+1| ⇒ un+2 = aun+1 + |un − an+1|. (4) Do u0 = 0 nên suy ra u1 = au0 + bu2 0 + c2 = √ c2 = |c|. Do đó u0, u1 ∈ Z. Vậy từ (4) suy ra un ∈ Z với mọi n ∈ N. Nhận xét. Ta cũng có thể giải bài toán này bằng cách khác như sau: Ta có u2 n+1 − 2aunun+1 + u2 n(a2 − b) − c2 = 0 hay u2 n+1 − 2aunun+1 + u2 n − c2 = 0. (5) Trong (5) thay n bởi n + 1 ta có u2 n+2 − 2aun+1un+2 + u2 n+1 − c2 = 0. (6) Trừ từng vế của (6) cho (5) ta được u2 n+2 − u2 n − 2aun+1un+2 + 2aunun+1 = 0 hay (un+2 − un)(un+2 + un − 2aun+1) = 0. (7) Từ (7) suy ra un+2 = un hoặc un+2 = 2aun+1 − un. Từ đó do u0, u1 ∈ Z nên un ∈ Z với mọi n = 1, 2, ... Từ bài toán này có thể cho nhiều bài toán với các giá trị a, b, c cụ thể. Chẳng hạn, chứng minh rằng mọi số hạng của các dãy số sau đều là số nguyên. 1)    u0 = 0 un+1 = 5un + 24u2 n + 9 2)    u0 = 0 un+1 = 4un + 15u2 n + 36 Ta cũng có thể dựa vào cách chứng minh để đưa ra các bài toán sau: 38
  • 41. 3)    u0 = 1 un+1 = 5un + 24u2 n + 25 4)    u0 = 2 un+1 = 3un + 8u2 n + 9 Bài tập 2.2.3. Tìm số nguyên dương k sao cho dãy số sau gồm toàn các số nguyên:    a1 = 1 an+1 = 5an + ka2 n − 8 với mọi n = 1, 2, 3, ... (1) Lời giải. Từ giả thiết ta có a2 = 5 + √ k − 8. Đặt √ k − 8 = t (t ∈ N) khi đó a2 = 5 + t, theo (1) ta có a3 = 5(5 + t) + (t2 + 8)(5 + t)2 − 8. Để a3 ∈ Z thì ta phải có f(t) = (t2 + 8)(5 + t)2 − 8 = q2 (q ∈ N). Ta có f(t) = t4 + 10t3 + 33t2 + 80t + 192. Do (t2 +5t+4)2 < f(t) < (t2 +5t+14)2 và f(t) ... 2 nên suy ra q = t2 +5t+v (5 ≤ v ≤ 13, v ∈ N). Từ đó v ∈ {6, 8, 10, 12}. Thử trực tiếp ta được v = 8 và q = t2 + 5t + 8 ⇒ t = 4 ⇒ k = 24. Ngược lại với k = 24 thì an+1 = 5an + 24a2 n − 8 ⇒ a2 n+1 − 10anan+1 + a2 n + 8 = 0. (2) Thay n bởi n + 1 ta có a2 n+2 − 10an+1an+2 + a2 n+1 + 8 = 0. (3) 39
  • 42. Trừ từng vế (2) và (3) ta được (an − an+2)(an+2 + an − 10an+1) = 0. Do (an) là dãy tăng thực sự nên an < an+2. Do đó an+2 = 10an+1 − an, trong đó a1 = 1, a2 = 9. Vậy dãy (an) gồm toàn các số nguyên khi và chỉ khi k = 24. Bài tập 2.2.4 (Olympic Bungari 1978). Cho dãy số (an):    a1, a2 ∈ Z; a2 1 + a2 2 + a a1a2 ∈ Z an+2 = a2 n+1 + a an , với mọi n ∈ N∗ , an = 0 (a là số cho trước). Chứng minh rằng dãy số gồm toàn các số nguyên. Lời giải. Từ giả thiết ta có anan+2 = a2 n+1 + a với mọi n, suy ra an+1an−1 = a2 n + a với mọi n ≥ 2. Trừ hai đẳng thức này vế theo vế ta có anan+2 + a2 n = an+1an−1 + a2 n+1. Hay an+2 + an an+1 = an+1 + an−1 an ⇒ an+1 + an−1 an = t (hằng số với mọi n ≥ 2). Hay an+1 = tan − an−1. Mặt khác a3 = a2 2 + a a1 ⇒ t = a3 + a1 a2 = a2 1 + a2 2 + a a1a2 ∈ Z. Vì a1, a2, t ∈ Z suy ra an ∈ Z với mọi n ∈ N∗ (Chứng minh bằng quy nạp). Bài tập 2.2.5. Xét các dãy số (an), n ≥ 1 xác định bởi:    a1 = a2 = 1, a3 = 199 an+1 = 1989 + anan−1 an−2 , (n ≥ 3). Chứng minh tất cả các số hạng của dãy đều là số nguyên dương. Lời giải. Giả thiết suy ra an+1an−2 = 1989 + anan−1. 40
  • 43. Thay n bởi n − 1 ta được anan−3 = 1989 + an−1an−2. Trừ hai đẳng thức trên vế theo vế ta được an+1an−2 − anan−1 = anan−3 − an−1an−2 suy ra an+1 + an−1 an = an−1 + an−3 an−2 với n ≥ 4. • Nếu n chẵn an+1 + an−1 an = an−1 + an−3 an−2 = ... = a3 + a1 a2 = 200. • Nếu n lẻ an+1 + an−1 an = an−1 + an−3 an−2 = ... = a4 + a2 a3 = 11. Suy ra an+1 =    200an − an−1 nếu n chẵn 11an − an−1 nếu n lẻ. Từ công thức trên, bằng phương pháp chứng minh quy nạp với a1, a2, a3 là các số nguyên dương suy ra an+1 là số nguyên dương với mọi n ≥ 1. Bài tập 2.2.6. Cho dãy (un) được xác định bởi:    u1 = u2 = 0 un+1 = u2 n + 10un − 5un−1 + 50 un−1 + 5 (n ≥ 2). Chứng minh rằng un là số nguyên chia hết cho 5 với mọi n = 0, 1, 2, ... Lời giải. un+1 = u2 n + 10un − 5un−1 + 50 un−1 + 5 ⇔ un+1 + 5 = (un + 5)2 + 50 un−1 + 5 ⇔ 1 5 un+1 + 1 = 1 5 un + 1 2 + 2 1 5 un−1 + 1 . 41
  • 44. Đặt xn = 1 5 un + 1, dãy đã cho được viết lại    x1 = x2 = 1 xn+1 = x2 n + 2 xn−1 , (n ≥ 2). Ta chứng minh dãy (xn) nguyên. +/ Với n = 2, x3 = 3 ∈ Z. +/ Giả sử mệnh đề đúng với mọi n = k ≥ 2, tức là xk+1 ∈ Z. Ta chứng minh xk+2 ∈ Z. Ta có xk+1xk−1 = x2 k + 2, xk+2xk = x2 k+1 + 2 ⇒ xk+2xk − xk+1xk−1 = x2 k+1 − x2 k ⇒ xk xk+1 + xk−1 = xk+1 xk+2 + xk . Tương tự, ta được: xk xk+1 + xk−1 = xk+1 xk+2 + xk = ... = x2 x3 + x1 = 1 4 . Từ đó suy ra xk+1 xk+2 + xk = 1 4 ⇒ xk+2 = 4xk+1 − xk. Từ giả thiết quy nạp xk, xk+1 ∈ Z nên xk+2 ∈ Z, ta có điều phải chứng minh. Vậy ta có 1 5 un ∈ Z nên un là số nguyên chia hết cho 5. Bài tập 2.2.7. Giả sử dãy số nguyên (an) thỏa mãn a4 = 4 và 1 a1a2a3 + 1 a2a3a4 + · · · + 1 anan+1an+2 = (n + 3)an 4an+1an+2 với mọi n nguyên, n ≥ 2. Chứng minh rằng an = n với mọi n nguyên dương. Lời giải. Ta viết lại hệ thức truy hồi dưới dạng (n + 2)an−1 4anan+1 + 1 anan+1an+2 = (n + 3)an 4an+1an+2 ⇔ (n + 2)an+2 = (n + 3)a2 n − 4 an−1 với mọi n ≥ 3. Cho n = 2 vào đẳng thức ban đầu, ta được 4(a1 +4) = 5a1a2 2 suy ra rằng a1 là ước của 16 và 5 là ước của a1 + 4. Từ đó suy ra a1 = 16, a2 = 1 hoặc a1 = 1, a2 = 2. Trường hợp 1. Nếu a1 = 16, a2 = 1 thì 5a5 = 6a2 3 − 4, a3a6 = 18 và 7a7 = 2a2 5 − 1 khi cho n = 3, 4, 5 tương ứng. Vì a3 ≡ ±2(mod5) và a3 là ước số của 18 nên a3 = 3 hoặc a3 = 18. Kiểm tra trực tiếp cho thấy cả hai đều không phải 42
  • 45. là nghiệm của bài toán. Trường hợp 2. Nếu a1 = 1, a2 = 2. Khi đó thay n = 3, 4 ta được 5a5 +2 = 3a2 3, a3a6 = 18. Một lần nữa ta lại có a3 ≡ 2(mod5) và a3 là ước của 18 nên suy ra hai trường hượp a3 = 3, a6 = 6 hoặc a3 = 18, a6 = 1. Trong trường hợp thứ hai ta có a5 = 194, mâu thuẫn với đẳng thức 8a8 = 9a2 6 − 4 a5 . Trường hợp thứ hai ta suy ra a5 = 5 và như vậy ai = i với mọi i = 1, 2, 3, 4, 5, 6. Bây giờ bằng phương pháp quy nạp dễ dàng chứng minh rằng an = n với mọi n nguyên dương. Bình luận. Bài này là đề thi chọn đội tuyển Bulgaria năm 2006. Đây là dạng toán về dãy số nguyên. Có thể phát biểu bài toán dưới dạng khác: Tìm tất cả các giá trị a1, a2, a3 để dãy số cho bởi công thức truy hồi ban đầu và điều kiện a4 = 4 chứa gồm toàn các số nguyên. Dưới đây là một số bài toán tương tự. 1. (VMO 1996) Hãy xác định tất cả các hàm số f : N∗ → N∗ thỏa mãn đẳng thức: f(n) + f(n + 1) = f(n + 2)f(n + 3) − 1996 với mọi n ∈ N∗ . 2. (VMO 1997) Hỏi có bao nhiêu hàm số f : N∗ → N∗ thỏa mãn điều kiện sau: f(n)f(n + 2) = [f(n + 1)]2 + 1997 với mọi n ∈ N∗ . Các bài toán tiếp theo sẽ liên quan đến việc biểu diễn các số hạng của dãy số theo tổng lũy thừa các số nguyên. Phương pháp thường dùng là sử dụng định lý Fermat để chỉ ra các phần dư. Bài tập 2.2.8. Cho dãy số (xn) xác định bởi:    x1 = 5, x2 = 7 xn+1 = x3 n + 6xn−1 + 3 · 22008 . 43
  • 46. Chứng minh rằng xn không thể biểu diễn được dưới dạng tổng của các lũy thừa bậc 6 của ba số nguyên dương. Lời giải. Xét A = a6 + b6 + c6 . Theo định lý Fermat x13 ≡ x(mod13) ⇔ x(x6 − 1)(x6 + 1) ≡ 0(mod13) ⇔      x6 ≡ 0 (mod13) x6 ≡ 1 (mod13) x6 ≡ −1 (mod13) Vậy bộ thặng dư của A mod 13 là: S = {0, ±1, ±2, ±3}. Ta có 212 ≡ 1(mod13) ⇒ 22004 ≡ 1(mod13) (do 2004 ... 12). Nên 22008 = 24 · 22004 ≡ 24 (mod13) ≡ 3(mod13) ⇒ 3 · 22008 ≡ 9(mod13). Nên xn+1 ≡ x3 n + 6xn−1 + 9(mod13). Ta có x3 ≡ 382 ≡ 5(mod13), x4 ≡ 176 ≡ 7(mod13) nên số dư của xn khi chia cho 13 tuần hoàn với chu kỳ 2. Từ đó suy ra xn ≡ xn−2(mod13). Do đó    x2n ≡ x2 ≡ 7(mod13) x2n+1 ≡ x1 ≡ 5(mod13) ⇒   xn ≡ 5 (mod13) xn ≡ 7 (mod13) Mà 5, 7 /∈ S. Vậy xn không thể biểu diễn được dưới dạng tổng của lũy thừa bậc 6 của ba số nguyên dương. Bài tập 2.2.9. Cho dãy (an), (n = 0, 1, 2, ...) được xác định bởi:    a0 = 2 an+1 = 4an + 15a2 n − 60. Hãy xác định số hạng tổng quát của an, chứng minh rằng số 1 5 (a2n + 8) có thể biểu diễn thành tổng bình phương của ba số nguyên liên tiếp với mọi n ≥ 1. Lời giải. Theo bài ta có a2 n+1 − 8anan+1 + a2 n + 60 = 0. (1) 44
  • 47. Thay n bởi n − 1 ta được a2 n − 8an−1an + a2 n−1 + 60 = 0. (2) Trừ theo từng vế (1) cho (2) được a2 n+1 − a2 n−1 + 8an−1an − 8anan+1 = 0 hay (an+1 − an)(an+1 − 8an + an−1) = 0. Từ giả thiết, ta có an+1 > 4an > 16an−1 suy ra an+1 − an > 0. Do đó an+1 − 8an + an−1 = 0. Ta có phương trình đặc trưng t2 − 8t + 1 = 0 có hai nghiệm là t1 = 4 + √ 15, t2 = 4 − √ 15. Từ đó an = (4 + √ 15)n + (4 − √ 15)n . Bây giờ ta chứng minh 1 5 (a2n + 8) có thể biểu diễn thành tổng của ba số nguyên liên tiếp với mọi n ≥ 1. Thật vậy, với mỗi n ≥ 1, thì tồn tại k ∈ N để: (4 + √ 15)n − (4 − √ 15)n = √ 15k ⇒ [(4 + √ 15)n − (4 − √ 15)n ]2 = 15k2 hay (4 + √ 15)2n + (4 − √ 15)2n = 15k2 + 2. Nên 1 5 (a2n + 8) = 1 5 [(4 + √ 15)2n + (4 − √ 15)2n + 8] = 3k2 + 2 = (k − 1)2 + k2 + (k + 1)2 . Vậy ta có điều phải chứng minh. Bài tập 2.2.10 (IMO 2005). Xét dãy số a1, a2, ... được xác định như sau: an = 2n + 3n + 6n − 1, (n = 1, 2, ...). Tìm tất cả các số nguyên dương nguyên tố cùng nhau với mọi số hạng của dãy số trên. Lời giải. Ta khẳng định số 1 là số cần tìm duy nhất. Ta chỉ cần chứng minh 45
  • 48. rằng mỗi số nguyên tố p là ước của một số hạng an nào đó. Khẳng định này đúng với p = 2, p = 3 vì a2 = 48. Bây giờ, ta xét số nguyên tố p > 3 bất kỳ. Theo định lý Fermat nhỏ, số dư của phép chia 2p−1 , 3p−1 , 6p−1 trong phép chia cho p đều là 1. Suy ra số dư trong phép chia của tổng 3 · 2p−1 + 2 · 3p−1 + 6p−1 trong phép chia cho p là 3 + 2 + 1 = 6. Do đó, tổng 3 · 2p−1 + 2 · 3p−1 + 6p−1 = 6 · 2p−2 + 6 · 3p−2 + 6 · 6p−2 có số dư 6 trong phép chia cho p. Suy ra ap−2 = 2p−2 + 3p−2 + 6p−2 − 1 chia hết cho p. Ta có điều phải chứng minh. 2.3. Tính chính phương Với tính chất này ta thường tìm số hạng tổng quát của dãy số, đưa biểu thức cần chứng minh về bình phương đủ của một số nguyên. Với một số bài toán tổng quát ta có thể đặc biệt hóa để có bài toán mới, ngược lại với một bài toán cụ thể ta có thể tổng quát hóa để được một dạng toán. Bài tập 2.3.1. Cho dãy số (an):    a0 = 0, a1 = 1 (1) an+1 = 2an − an−1 + 1 (2) Chứng minh rằng 4an+2an + 1 là số chính phương (n ≥ 1). Lời giải. Cách 1. Xét phương trình đặc trưng λ2 −2λ+1 = 0 ⇔ λ = 1 (nghiệm kép). Ta tìm g(n) = an2 sao cho g(n+1)−2g(n)+g(n−1) = 1 với mọi n ∈ N∗ . Giải ra ta có g(n) = n2 2 hay a∗ n = n2 2 là một nghiệm riêng của phương trình (2). Do đó (2) có nghiệm tổng quát là: an = C1 + nC2 + n2 2 . 46
  • 49. Theo (1), a0 = 0 suy ra C1 = 0, a1 = 1 nên C2 + 1 2 = 1 ⇒ C2 = 1 2 . Vậy an = 1 2 · n + n2 2 = n(n + 1) 2 . Do đó 4an+2an + 1 = 4 · (n + 2)(n + 3) 2 · n(n + 1) 2 + 1 = n(n + 1)(n + 2)(n + 3) + 1 = (n2 + 3n)(n2 + 3n + 2) + 1 = (n2 + 3n + 1)2 (đpcm). Cách 2. Từ công thức truy hồi của dãy ta thay n + 1 bởi n ta được an = 2an−1 − an−2 + 1. (3) Trừ vế theo vế đẳng thức (3) và (2) ta được an−1 − 3an + 3an−1 − an−2 = 0. Xét phương trình đặc trưng λ3 − 3λ2 + 3λ − 1 = 0 ⇔ λ = 1. Suy ra an = α+βn+γn2 . Do a0 = 0, a1 = 1, a2 = 3 nên ta tìm được α = 0, β = γ = 1 2 ⇒ an = n(n + 1) 2 . Do đó 4an+2an + 1 = 4 · (n + 2)(n + 3) 2 · n(n + 1) 2 + 1 = n(n + 1)(n + 2)(n + 3) + 1 = (n2 + 3n)(n2 + 3n + 2) + 1 = (n2 + 3n + 1)2 (đpcm). Nhận xét. Ta có thể tìm số hạng tổng quát mà không cần phương pháp sai phân, cách làm này sẽ gần gũi hơn với chương trình học phổ thông ban cơ bản. Đặt bn = an+1 − an. Từ giả thiết ta có an+1 − an = an − an−1 + 1. Do đó 47
  • 50. bn = bn−1 + 1. Từ đó tìm được bn = 1 + n (do (bn) là cấp số cộng với công sai bằng 1, b0 = 1). Ta có an = n−1 k=0 (ak+1 − ak) + a0 = n−1 k=0 bk = n + n−1 k=0 k = n + n(n − 1) 2 = n(n + 1) 2 . Bài tập 2.3.2. Cho dãy số (an) xác định bởi:    a1 = 3 n(2n − 1)an+1 = (2n + 1)(n + 1)an − (2n − 1)(2n + 1) với mọi n ≥ 1. Chứng minh rằng an + 1 là số chính phương với mọi n = 1, 2, 3, ... Lời giải. Nhận xét: Dãy số được cho bởi công thức truy hồi khá phức tạp, song nếu dùng phương pháp đổi sang dãy số phụ thì việc tìm số hạng tổng quát sẽ dễ dàng. Từ giả thiết, ta có 2n(2n − 1)an+1 = (2n + 1)(2n + 2)an − 2(2n − 1)(2n + 1). Chia hai vế đẳng thức cho (2n − 1)2n(2n + 1)(2n + 2) ta được an+1 (2n + 1)(2n + 2) = an (2n − 1)2n − 2 2n(2n + 2) . Đặt an (2n − 1)2n = bn thì ta có bn+1 = bn − 2 2n(2n + 2) ⇔ bn+1 − 1 2n + 2 = bn − 1 2n , ∀ n ∈ N∗ . Do đó bn − 1 2n = bn−1 − 1 2(n − 1) = ... = b1 − 1 2 . Suy ra bn = 1 2n + b1 − 1 2 . Từ đó an = 2n − 1 + (2n − 1)2n a1 2 − 1 2 = 4n2 − 1. Vậy an + 1 = 4n2 là số chính phương. Nhận xét: Thực chất bài toán này chỉ là tìm số hạng tổng quát dựa vào 48
  • 51. dãy số phụ, việc chứng minh số chính phương là khá rõ ràng. Nếu chọn các giá trị khác nhau của a1 ta được các bài toán khác. Chẳng hạn • a1 = 1 thì an = 2n − 1. Bài toán có thể yêu cầu chứng minh dãy gồm toàn các số tự nhiên lẻ. • a1 = 2 thì an = (2n − 1)(n + 1). Bài toán có thể yêu cầu chứng minh an chẵn khi n lẻ và an lẻ khi n chẵn. Bài tập 2.3.3. Cho dãy số nguyên (an) (n = 0, 1, 2, ...) thỏa mãn: an+2 + an−1 = 2(an+1 + an) với n = 1, 2, ... Chứng tỏ rằng tồn tại số nguyên M không phụ thuộc n sao cho M + 4an+1an là số chính phương với mọi n ∈ N. Lời giải. Đặt un = an+2 − an+1 − an với mọi n ≥ 0. Từ giả thiết suy ra un = un−1 + 2an ⇒ u2 n = (un−1 + 2an)2 = u2 n−1 + 4un−1an + 4a2 n = u2 n−1 + 4(an+1 − an − an−1)an + 4a2 n = u2 n−1 + 4an+1an − 4anan−1. Suy ra u2 n − 4an+1an = u2 n−1 − 4anan−1 với mọi n ≥ 1. Từ đó u2 n − 4an+1an = (a2 − a1 − a0)2 − 4a1a0 là hằng số. Gọi hằng số đó là M. Khi đó M + 4an+1an = u2 n với mọi n ≥ 0. Nhận xét. Ta có thể cho a0, a1, a2 các giá trị cụ thể để được các bài toán mới. Chẳng hạn: 1) Cho dãy số nguyên (an) thỏa mãn:    a0 = 0, a1 = 1, a2 = 1 an+2 + an−1 = 2(an+1 + an). Chứng minh rằng an+1an là số chính phương. 49
  • 52. 2) Cho dãy số nguyên (an) thỏa mãn:    a0 = 0, a1 = 1, a2 = 5 an+2 + an−1 = 2(an+1 + an). Chứng minh rằng 5 + 4an+1an là số chính phương. Bài tập 2.3.4. Cho dãy số (an) (n = 1, 2, ...) được xác định bởi:    a1 = 1, a2 = 3 an+1 = (n + 2)an − (n + 1)an−1 với mọi n ≥ 2. Tìm các giá trị của n để an là số chính phương. Lời giải. Ta sẽ giải bài toán tổng quát: Cho số nguyên p ≥ 2. Cho dãy số (an) (n = 1, 2, ...) được xác định bởi a1 = 1, a2 = 3, an+1 = (n + 2)an − (n + 1)an−1 với mọi n ≥ 2. Hãy xác định các giá trị của n để an là lũy thừa p của số tự nhiên. Lời giải. Với mỗi n ≥ 2, đặt bn = an −an−1. Khi đó từ công thức xác định dãy (an), ta có bn = n · bn−1 với mọi n ≥ 3. Kết hợp với b2 = a2 − a1 = 3 − 1 = 2!. Ta được bn = n! với mọi n ≥ 2. Ta có an = n k=2 (ak − ak−1) + a1 = n k=2 bk + 1 = n k=1 k!. Kết hợp với a1 = 1 = 1!. Ta được an = n k=1 k! với mọi n ≥ 1. (1) Xét p = 2. Từ (1) ta có an ≡ 3(mod10) với mọi n ≥ 5 nên an khác a2 với mọi n ≥ 5 (vì các số chính phương chỉ có thể có tận cùng bởi 0, 1, 4, 5, 6, 9). Với n = 1, 2, 3, 4 bằng cách thử trực tiếp ta thấy an là số chính phương khi và 50
  • 53. chỉ khi n = 1, n = 3. Xét p > 2. Khi đó an ≡ 0(mod3) với mọi n ≥ 2 (suy ra từ (1)) nên điều kiện cần để tồn tại a ∈ N sao cho an = ap là an ≡ 0(mod27) hoặc an = 1. Từ (1) ta có an > 1 với mọi n ≥ 2 và an = a8 + n k=9 k! với mọi n ≥ 9. Suy ra an ≡ a8(mod27) với mọi n ≥ 0. Mà a8 = 46233 ≡ 1(mod27) nên an ≡ 1(mod27) với mọi n ≥ 8. Như vậy với mọi n ≥ 8 đều không tồn tại a ∈ N sao cho an = ap . Với 1 ≤ n ≤ 7 bằng cách thử trực tiếp ta thấy chỉ có duy nhất giá trị n = 1 là giá trị cần tìm. Nhận xét. Bài đã ra là trường hợp đặc biệt của bài toán khái quát khi p = 2. Theo đó tất cả các giá trị n thỏa mãn là n = 1, n = 3. Bài tập 2.3.5 (HSG QG bảng B 1998). Cho các số nguyên a, b. Xét dãy số nguyên (an), (n = 0, 1, 2, ...) được xác định bởi:    a0 = a, a1 = b, a2 = 2b − a + 2 an+3 = 3an+2 − 3an+1 + an với n = 0, 1, 2, ... a) Tìm số hạng tổng quát an. b) Tìm tất cả các số nguyên a, b để an là số chính phương với mọi n ≥ 1998. Lời giải. a) Từ điều kiện đề bài ta có an+3 − an+2 = 2(an+2 − an+1) − (an+1 − an). (1) Đặt an+1 − an = bn với mọi n ≥ 0 thì (1) trở thành bn+2 = 2bn+1 − bn ⇒ bn+2 − bn+1 = bn+1 − bn = ... = b1 − b0 = a2 − 2a1 + a0 = 2 suy ra bn+1 = bn + 2 với mọi n. Vậy công thức tổng quát của bn là bn = b0 + 2n = 2n + b − a. 51
  • 54. Từ đó an − a0 = n−1 k=0 (ak+1 − ak) = n−1 k=0 bk = 2 n−1 k=0 k + n(b − a) = n(n − 1) + n(b − a). Vậy an = n2 + n(b − a − 1) + a. b) Giả sử an = n2 + n(b − a − 1) + a = u2 (n ≥ 1998, u > 0). Ta có 4u2 = 4n2 + 4n(b − a − 1) + 4a = [2n + (b − a − 1)]2 + 4a − (b − a − 1)2 . Đặt v = 2n + b − a − 1 và d = 4a − (b − a − 1)2 , ta có d = 4u2 − v2 = (2u + v)(2u − v). Với n đủ lớn thì v = 2n + b − a − 1 > 0. Nếu d khác 0 thì 2u − v khác 0 và |d| = |(2u − v)(2u + v)| ≥ |2u + v| ≥ 2u + v ≥ v = 2n + b − a − 1. Vì d là hằng số nên điều này không thể xảy ra khi n đủ lớn. Vậy d = 0. Suy ra 4a = (b − a − 1)2 . Đặt b − a − 1 = 2t thì a = t2 và b = a + 1 + 2t = (t + 1)2 . Đảo lại nếu a = t2 và b = (t + 1)2 thì an = n2 + n(b − a − 1) + a = n2 + 2tn + t2 = (n + t)2 . Kết luận an = u2 ⇔ a = t2 và b = (t + 1)2 . Ta có thể tìm số hạng tổng quát bằng phương pháp sai phân nhờ việc xét phương trình đặc trưng: λ3 − 3λ2 + 3λ − 1 = 0. Từ kết quả bài toán ta cũng có thể cho t các giá trị khác nhau để được bài toán mới. Chẳng hạn: Xét dãy số nguyên (an), (n = 0, 1, 2, ...) được xác định 52
  • 55. bởi:    a0 = 4, a1 = 9, a2 = 16 an+3 = 3an+2 − 3an+1 + an với n = 0, 1, 2, ... Chứng minh rằng an là số chính phương với mọi n ≥ 1998. Bài tập 2.3.6 (chọn đội tuyển QG 2006). Cho dãy số (an) được xác định bởi: a0 = 1, an+1 = 1 2 an + 1 3an với mọi n = 1, 2, 3, ... Chứng minh rằng với mọi số nguyên n, số An = 3 3a2 n − 1 là một số chính phương. Lời giải. Trước hết ta sẽ chứng minh rằng 3An+1 = 4An(An + 3) với mọi n nguyên dương. (*) Thật vậy, ta có An + 3 = 3 3a2 n − 1 + 3 = 9a2 n 3a2 n − 1 nên 4An(An + 3) = 4An · 9a2 n 3a2 n − 1 = 108a2 n (3a2 n − 1)2 . Mặt khác 3An+1 = 9 3a2 n+1 − 1 = 9 3 · 1 22 an + 1 3an 2 − 1 = 108a2 n (3a2 n − 1)2 . Do đó (∗) được chứng minh, tức là 3An+1 = 4An(An + 3) với mọi n nguyên dương. Hơn nữa, ta tính được A1 = 9 nên dễ dàng thấy An chia hết cho 3 với mọi n. Tiếp theo, ta chứng minh bằng quy nạp rằng An 3 + 1 là số chính phương với mọi n. (**) + Với n = 1 thì An 3 + 1 = 4 là số chính phương nên (**) đúng. + Giả sử (**) đúng với n = k, tức là tồn tại số tự nhiên p sao cho Ak 3 + 1 = p2 . Ta có Ak+1 3 + 1 = 4Ak(Ak + 3) 9 + 1 = 4 · Ak 3 · 1 + Ak 3 = 4p2 (p2 − 1) + 1 = (2p2 − 1)2 53
  • 56. cũng là một số chính phương nên (**) cũng đúng với n = k + 1. Do đó (**) được chứng minh. Từ 3An+1 = 4An(An + 3) ⇒ An+1 = 4An An 3 + 1 và (**) ta dễ dàng chứng minh bằng quy nạp An là số chính phương với mọi n. Bài tập 2.3.7. Dãy số (un) được xác định bởi:    u0 = 9, u1 = 161 un = 18un−1 − un−2 với n = 2, 3, ... Chứng minh rằng u2 n − 1 5 là số chính phương với mọi số tự nhiên n. Lời giải. Nhận xét rằng mọi số hạng của dãy đều là số nguyên. Với n = 0 ta có u2 0 − 1 5 = 92 − 1 5 = 16 = 42 . Với n = 1 ta có u2 1 − 1 5 = 1612 − 1 5 = 5184 = 722 . Vậy bài toán đúng với n = 0, 1. Xét với n ≥ 2 ta có un = 18un−1 − un−2 ⇔ un − 9un−1 = 9un−1 − un−2 ⇔ (un − 9un−1)2 = (9un−1 − un−2)2 hay u2 n − 18unun−1 = u2 n−2 − 18un−1un−2. (1) Lần lượt thay n = 2, 3, ... vào (1) ta được    u2 2 − 18u2u1 = u2 0 − 18u1u0 u2 3 − 18u3u2 = u2 1 − 18u2u1 .................................... u2 n − 18unun−1 = u2 n−2 − 18un−1un−2 Cộng theo vế các đẳng thức trên ta thu được u2 n + u2 n−1 − 18unun−1 = u2 1 + u2 0 − 18u1u0. (2) 54
  • 57. Thay u0 = 9, u1 = 161 vào (2) ta được u2 n + u2 n−1 − 18unun−1 = −80 hay (9un − un−1)2 80 = u2 n − 1. Do các số hạng của dãy đều nguyên nên (9un − un−1)2 ... 42 · 5 nên (9un − un−1) ... 4 · 5. Vì thế 9un − un−1 = 20a (a ∈ Z). Suy ra u2 n − 1 5 = a2 (đpcm). Cách khác. Sử dụng phương trình đặc trưng t2 − 18t + 1 = 0 để tìm số hạng tổng quát un = 1 2 [(9 + √ 80)n+1 + (9 − √ 80)n+1 ]. Từ đó suy ra u2 n − 1 5 = 1 20 [(9 + √ 80)n+1 − (9 − √ 80)n+1 ]2 . Tiếp tục sử dụng khai triển Newton ta thu được điều phải chứng minh. Bài tập 2.3.8. Cho dãy số nguyên dương (xn) xác định như sau:    x0 = 2, x1 = 16 xn = 6xn−1 − xn−2 + 4, n = 2, 3, ... Chứng minh rằng 1 2 x2 n + xn là số chính phương với mọi n = 0, 1, 2, ... Lời giải. Đặt xn = un − 1. Ta có    u0 = 3, u1 = 17 un = 6un−1 − un−2, n = 2, 3, ... Ta chứng minh u2 n − 1 chia hết cho 2 và thương số là số chính phương với mọi n = 0, 1, 2, .... Ta có u2 0 − 1 = 8, u2 1 − 1 = 288. Rõ ràng kết luận đúng khi n = 0, n = 1. Xét với n ≥ 2. Ta có un − 3un−1 = 3un−1 − un−2. Bình phương hai vế đẳng thức trên ta có u2 n + 9u2 n−1 − 6unun−1 = 9u2 n−1 − 6un−1un−2 + u2 n−2 hay u2 n − 6unun−1 = −6un−1un−2 + u2 n−2 ∀ n ≥ 2. (1) 55
  • 58. Lần lượt thay n = 2, 3, ..., k ta có    u2 2 − 6u2u1 = −6u1u0 = u2 0 u2 3 − 6u3u2 = −6u2u1 + u2 1 ............................ u2 k − 6ukuk−1 = −6uk−1uk−2 + u2 k−2. Cộng từng vế k − 1 đẳng thức trên ta có u2 k + u2 k−1 − 6ukuk−1 = −6u1u0 + u2 1 + u2 0. (2) Do u0 = 3, u1 = 17 nên từ (2) ta có u2 k + u2 k−1 − 6ukuk−1 = −8 ⇒ 8(u2 k − 1) = (3uk − uk−1)2 ⇒ u2 k − 1 = (3uk − uk−1)2 8 , k = 2, 3, ... (3) Do u0, u1 ∈ Z và un = 6un−1 − un−2 suy ra uk ∈ Z với mọi k = 0, 1, 2, ... Vì vậy từ (3) suy ra (3uk −uk−1)2 ... 8 suy ra 3uk −uk−1 ... 4. Suy ra 3uk −uk−1 = 4ak (với ak ∈ Z). Thay lại vào (3) suy ra u2 k − 1 = 2a2 k với ak ∈ Z. Vì ak ∈ Z nên suy ra điều phải chứng minh. Bài tập 2.3.9. Cho dãy số (un) xác định như sau: u1 = 1995, u2 = 1996, un+1 = un(un − 1) + 2 với mọi n = 2, 3, .... Chứng minh rằng (u2 1 + 1)(u2 2 + 1) · · · (u2 1996 + 1) − 1 là số chính phương. Lời giải. Đặt Sk = (u2 1 + 1)(u2 2 + 1) · · · (u2 k + 1) − 1. Ta chứng minh Sk = (uk+1 − 1)2 . (1) Thật vậy với k = 1 ta có S1 = (u2 1 + 1) − 1 = 19952 = (1996 − 1)2 = (u2 − 1)2 . Vậy (1) đúng với k = 1. Giả sử (1) đúng với n = k, tức là Sn = (un+1 − 1)2 . (2) 56
  • 59. Ta có Sn+1 = (u2 1 + 1)(u2 2 + 1) · · · (u2 n+1 + 1) − 1 = (Sn + 1)(u2 n+1 + 1) − 1. (3) Do giả thiết quy nạp ta có Sn+1 = [(un+1 − 1)2 + 1](u2 n+1 + 1) − 1 = [(u2 n+1 + 1) − 2un+1 + 1](u2 n+1 + 1) − 1 = (u2 n+1 + 1)2 − 2un+1(u2 n+1 + 1) + u2 n+1 = (u2 n+1 + 1 − un+1)2 = (un+2 − 1)2 . (4) Từ (4) suy ra (1) đúng khi k = n + 1. Vậy (1) đúng với mọi k. Do đó S1996 = (u2 1 + 1)(u2 2 + 1) · · · (u2 1996 + 1) − 1 = (u1997 − 1)2 . (5) Vì u1, u2 nguyên nên un+1 = un(un − 1) + 2 nguyên với mọi n. Do đó S1996 là số chính phương. 2.4. Bài tập Bài tập 2.4.1. Cho dãy số (an) xác định bởi a1 = a2 = 12 √ 2, an+1 = −an a2 n−1 + 1 + an−1 a2 n + 1 với mọi n ≥ 2. Chứng minh rằng 2 + 2 a2 n + 1 là số nguyên dương với mọi n ≥ 1. Bài tập 2.4.2. Chứng minh rằng mọi số hạng của dãy số {an} xác định bởi: a0 = 1, an+1 = 2an + 3a2 n − 2 đều nguyên. Bài tập 2.4.3. Dãy số (un) được xác định bởi:    u0 = 2, u1 = 5 un+1un−1 − u2 n = 6n−1 với mọi n = 1, 2, 3, ... Chứng minh rằng mọi số hạng của dãy đều là các số nguyên. 57
  • 60. Bài tập 2.4.4. Cho dãy số (xn) được xác định bởi:    x0 = 1 xn+1 = 3xn + 5x2 n − 4 2 với mọi n = 0, 1, 2, ... Chứng minh rằng mọi số hạng của dãy đều là số nguyên. Bài tập 2.4.5. Dãy số không âm (un), n = 0, 1, 2, ... thỏa mãn điều kiện sau:    u1 = 1 um+n + um−n = 1 2 (u2m + u2n) với mọi m ≥ n; m, n ∈ N Chứng minh rằng mọi phần tử của dãy là số chính phương. Bài tập 2.4.6. Dãy số (un) xác định bởi:    u1 = 1, u2 = 3 un = (n + 1)un−1 − nun−2 với mọi n = 3, 4, 5, ... Tìm các số hạng của dãy chia hết cho 11. Bài tập 2.4.7. Cho p, q là hai số nguyên dương cho trước. Dãy số (un) được xác định bởi:    u1 = p + q − 1 un+1 = p + q − pq un n = 1, 2, ... Đặt Pn = u1u2 · · · un. Chứng minh rằng với mọi n = 1, 2, ... thì Pn là số nguyên. Bài tập 2.4.8. Cho dãy số (un) được xác định bởi u0 = 3, u1 = 11, un+2 = 2un+1 + 7un với n = 0, 1, 2, ... Tìm các số nguyên dương lẻ a sao cho với mọi số nguyên dương m, n tùy ý luôn tìm được số nguyên dương k thỏa mãn uk n − a chia hết cho 2m . 58
  • 61. Bài tập 2.4.9. Cho dãy số (un) thỏa mãn: un+2 = unun+1 2un − un+1 (n = 1, 2, ...) Hãy tìm điều kiện cần và đủ đối với u1, u2 để trong dãy đã cho có vô hạn số hạng là số nguyên. Bài tập 2.4.10. Cho dãy số (un) xác định như sau: u1 = 20, u2 = 100 và un+1 = 4un + un−1 − 1976 n = 2, 3, ... Chứng minh rằng tồn tại ít nhất một số của dãy số chia hết cho 1996. 59
  • 62. Chương 3 Giới hạn của dãy số 3.1. Giới hạn của tổng Các bài toán về tìm giới hạn của tổng ta thu gọn tổng đó bằng cách phân tích hạng tử tổng quát thành hiệu các hạng tử nối tiếp nhau để các hạng tử có thể triệt tiêu, cuối cùng đưa tổng đó về biểu thức chỉ còn chứa xn, sau đó tìm lim xn. Bài tập 3.1.1. Cho dãy số (xn) (n = 1, 2, ...) được xác định như sau: x1 = 1 và xn+1 = xn(xn + 1)(xn + 2)(xn + 3) + 1 với n = 1, 2, ... Đặt yn = n i=1 1 xi + 2 , (n = 1, 2, ...). Tìm lim n→+∞ yn. Lời giải. Ta có x2 = 5 và xn > 0 với mọi n = 1, 2, ... xn+1 = xn(xn + 1)(xn + 2)(xn + 3) + 1 = (x2 n + 3xn)(x2 n + 3xn + 2) + 1 = x2 n + 3xn + 1. (1) 60
  • 63. Từ đó suy ra xn+1 + 1 = x2 n + 3x2 n + 2 = (xn + 1)(xn + 2) ⇒ 1 xn+1 + 1 = 1 (xn + 1)(xn + 2) = 1 xn + 1 − 1 xn + 2 ⇒ 1 xn + 2 = 1 xn + 1 − 1 xn+1 + 1 . Do đó yn = n i=1 1 xi + 2 = n i=1 1 xi + 1 − 1 xi+1 + 1 = 1 x1 + 1 − 1 xn+1 + 1 = 1 2 − 1 xn+1 + 1 . Từ (1) xk+1 = x2 k + 3xk + 1 > 3xk ≥ 3 · 3k−1 = 3k . Ta dễ dàng chứng minh bằng quy nạp rằng xn > 3n−1 . (2) Nên lim n→+∞ yn = 1 2 (vì do (2) xn+1 > 3n ). . Ta có thể chứng minh lim xn = +∞ với cách khác: Dễ thấy (xn) là dãy tăng, giả sử rằng lim xn = a (a ≥ 1). Nên ta có a = a(a + 1)(a + 2)(a + 3) + 1. Suy ra a2 = a(a + 1)(a + 2)(a + 3) + 1 hay a4 + 6a3 + 10a2 + 6a + 1 = 0. Rõ ràng phương trình này không có nghiệm nào thỏa mãn a ≥ 1. Vậy lim xn = +∞. Bài tập 3.1.2 (HSG QG năm 2009). Cho dãy số (xn), n = 1, 2, ... xác định bởi:    x1 = 1 2 xn = x2 n−1 + 4xn−1 + xn−1 2 , (n = 2, 3, ...) Chứng minh rằng dãy (yn) (n = 1, 2, ...) với yn = n i=1 1 x2 i có giới hạn hữu hạn và tìm giới hạn đó. 61
  • 64. Lời giải. Từ giả thiết ta có xn > 0 với mọi n ≥ 1. Ta có xn − xn−1 = x2 n−1 + 4xn−1 + xn−1 2 − xn−1 = x2 n−1 + 4xn−1 − xn−1 2 > 0 với mọi n ≥ 2. Do đó dãy (xn) tăng. Giả sử lim xn = a thì a > 0 và a = √ a2 + 4a + a 2 ⇔ a = 0 (vô lý). Vậy lim xn = +∞. Từ xn = x2 n−1 + 4xn−1 + xn−1 2 với mọi n ≥ 2 suy ra x2 n = (xn + 1)xn−1 ⇒ 1 x2 n = 1 xn−1 − 1 xn với mọi n ≥ 2. Do đó yn = n i=1 1 x2 i = 1 x2 1 + 1 x1 − 1 x2 + 1 x2 − 1 x3 + · · · + 1 xn−1 − 1 xn = 1 x2 1 + 1 x1 − 1 xn = 6 − 1 xn với mọi n ≥ 2. Suy ra yn < 6 với mọi n ≥ 1 và dãy (yn) tăng vì yn = yn−1 + 1 xn > yn−1. Vậy (yn) có giới hạn hữu hạn và lim n→+∞ yn = lim n→+∞ 6 − 1 xn = 6. Bài tập 3.1.3. Xét dãy số (xn) (n = 1, 2, 3, ...) xác định bởi: x1 = 2 và xn+1 = 1 2 (x2 n + 1) với mọi n = 1, 2, 3, ... Đặt Sn = 1 1 + x1 + 1 1 + x2 + · · · + 1 1 + xn . Tìm lim n→+∞ Sn. Lời giải. Ta có thể tổng quát hóa bài toán như sau: Cho dãy (un) thỏa mãn    u1 = a un+1 = u2 n + (b + c)un + c2 b − c . 62
  • 65. Ta chứng minh Sn = n i=1 1 ui + b = 1 u1 + c − 1 un+1 + c . Thật vậy ta có un+1 = u2 n + (b + c)un + c2 b − c ⇒ un+1 + c = u2 n + (b + c)un + bc b − c = (un + b)(un + c) b − c . Từ đó 1 un+1 + c = 1 un + c − 1 un + b ⇒ 1 un + b = 1 un + c − 1 un+1 + c . Khai triển và ước lượng được 1 u1 + b = 1 u1 + c − 1 u2 + c 1 u2 + b = 1 u2 + c − 1 u3 + c ............................ 1 un + b = 1 un + c − 1 un+1 + c . Do đó Sn = 1 u1 + c − 1 un+1 + c . Từ đó vận dụng vào bài toán trên với b = 1, c = −1 ta có Sn = 1 x1 − 1 − 1 xn+1 − 1 = 1 − 1 xn+1 − 1 . Mà xn+1 − xn = 1 2 (xn − 1)2 > 0 với mọi n ∈ N∗ nên dãy (xn) là dãy tăng. Giả sử lim n→+∞ xn = a (a > 2) thì 2a = a2 + 1 ⇒ a = 1 (vô lý). Vậy lim n→+∞ xn = +∞. Do đó lim n→+∞ Sn = 1. Bài tập 3.1.4. Cho dãy số (xn) được xác định bởi: x1 = 1, xn+1 = (2xn + 1)2012 2012 + xn với n là số nguyên dương. Đặt un = (2x1 + 1)2011 2x2 + 1 + (2x2 + 1)2011 2x3 + 1 + (2x3 + 1)2011 2x4 + 1 + · · · + (2xn + 1)2011 2xn+1 + 1 . 63
  • 66. Tính lim un. Lời giải. Ta có xn+1 − xn = (2xn + 1)2012 2012 với mọi n ≥ 1. Suy ra 1 2xn + 1 − 1 2xn+1 + 1 = 2(xn+1 − xn) (2xn + 1)(2xn+1 + 1) = (2xn + 1)2011 1006(2xn+1 + 1) ⇒ n i=1 (2xi + 1)2011 2xi+1 + 1 = 1006 n i=1 1 2xi + 1 − 1 2xi+1 + 1 = 1006 1 2x1 + 1 − 1 2xn+1 + 1 . Mặt khác xn+1 − xn ≥ 0 nên (xn) là dãy số tăng với mọi n ≥ 1. Nếu (xn) bị chặn thì lim xn tồn tại. Đặt lim xn = a ⇒ a ≥ 1 và a = (2a + 1)2012 2012 + a (vô lý). Suy ra (xn) không bị chặn, hay lim xn = +∞, suy ra lim n→+∞ 1 2xn+1 + 1 = 0. Vậy lim n→+∞ un = 1006 3 . Bài tập 3.1.5. Cho dãy số (xn) với n = 1, 2, ... được xác định bởi:    x1 = a, (a > 1), x2 = 1 xn+2 = xn − ln xn (n ∈ N∗ ). Đặt Sn = n−1 k=1 (n − k) ln √ x2k−1 (n ≥ 2). Tìm lim n→+∞ Sn n . Lời giải. Nhận xét rằng x2n = 1, n = 1, 2, ...do ln 1 = 0 suy ra lim n→+∞ x2n = 1. Tiếp theo, ta chứng minh dãy (x2n+1) cũng có giới hạn là 1. Xét hàm số f(x) = x − ln x liên tục và đồng biến trong (1; +∞) vì f (x) = 1 − 1 x > 0 với mọi x > 1. Trước hết ta chứng minh bằng phương pháp quy nạp, dãy (x2n+1) bị chặn 64
  • 67. dưới bởi 1. Theo giả thiết thì x1 = a > 1, giả sử x2k+1 > 1 thì f(x2k+1) > f(1) > 1 nên hiển nhiên x2k+3 > 1 tức dãy (x2n+1) bị chặn dưới bởi 1. Tiếp theo ta chứng minh dãy (x2n+1) là dãy giảm. Thật vậy do x2n+1 > 1 nên ln x2n+1 > 0. Vì thế x2n+3 − x2n+1 = − ln x2n+1 < 0, tức là dãy (x2n+1) là dãy giảm. Từ đó suy ra (x2n+1) có giới hạn c = lim n→+∞ x2n+1. Chuyển qua giới hạn dãy số, ta được c = c − ln c ⇔ ln c = 0 ⇔ c = 1. Vậy dãy số (xn) có giới hạn là 1. Theo định lý Cessaro, ta có lim n→+∞ x1 + x2 + · · · + x2n 2n = 1 hay lim n→+∞ (x1 + x3 + · · · + x2n+1) + (x2 + x4 + · · · + x2n) 2n = 1 ⇔ lim n→+∞ nx1 − (n − 1) ln x1 − (n − 2) ln x3 − · · · − ln x2n−3 + n 2n = 1 ⇔ lim n→+∞ a 2 − Sn n + 1 2 = 1 hay lim n→+∞ Sn n = a − 1 2 . 3.2. Dãy con và sự hội tụ của dãy số Khi khảo sát sự hội tụ của dãy số ta thường sử dụng các định lý về tính đơn điệu và bị chặn, nếu dãy không đơn điệu thì xét dãy với chỉ số chẵn, chỉ số lẻ. Tuy nhiên có những dãy số phức tạp, tăng giảm bất thường, trong trường hợp như thể ta thường xây dựng các dãy số phụ đơn điệu, chứng minh các dãy số phụ có giới hạn, sau đó chứng minh dãy số ban đầu có cùng giới hạn, các dãy số phụ phải được xây dựng từ dãy số chính. 65
  • 68. Nhận xét. Mọi dãy con của dãy hội tụ đều hội tụ và ngược lại nếu lim x2n = lim x2n+1 = a thì lim xn = a. Một cách tổng quát ta có: Cho số nguyên m ≥ 2 nếu lim xmn+i = a với mọi i = 0, 1, 2, ..., m − 1 thì lim xn = a. Bài tập 3.2.1. Dãy số (xn) được xác định bởi công thức:    x0 = x1 = 1 5xn+2 = xn + 2xn+1. Chứng minh rằng dãy (xn) hội tụ. Lời giải. Xét dãy số (an) được xác định bởi: a0 = 1, an+1 = 2an 3 , dễ thấy (an) giảm dần về 0. Ta chứng tỏ rằng max{x2n, x2n+1} ≤ an, với mọi n. (1) Thật vậy, (1) đúng với n = 0 và n = 1. Giả sử (1) đúng với n và do (an) là dãy giảm nên 5x2n+2 = x2n + 2x2n+1 ≤ 3an ⇒ x2n+2 ≤ an+1. Và 5x2n+3 = x2n+1 + 2x2n+2 ≤ an + 2an+1 ≤ 3an ⇒ x2n+3 ≤ an+1. Như vậy (1) đúng với n + 1 hay (1) đúng với mọi n = 0, 1, 2, ... Dễ thấy xn > 0 với mọi n và từ (1) theo nguyên lý kẹp ta có lim x2n = lim x2n+1 = 0 ⇒ lim xn = 0. Nhận xét: Việc đưa vào dãy phụ (an) có tác dụng chặn cả hai dãy con (x2n) và (x2n+1) và làm chúng cùng hội tụ về một điểm. Có thể sử dụng phương pháp sai phân tìm được số hạng tổng quát xn = C1 1 − √ 6 5 n + C2 1 + √ 6 5 n . Thay các giá trị của x0, x1 để tìm C1, C2 từ đó tìm được lim xn = 0. 66
  • 69. Bài tập 3.2.2. Dãy (xn) được xác định bởi:    x0, x1, x2 ∈ (0; 1) 3xn+3 = x2 n + x2 n+2. Chứng minh rằng dãy (xn) hội tụ. Lời giải. Ta xét dãy số (an) xác định bởi: a0 = max{x0, x1, x2}, an+1 = 2a2 n 3 . Dễ thấy dãy số (an) giảm dần về 0. Ta chứng tỏ rằng max{x3n, x3n+1, x3n+2} ≤ an với mọi n. (1) Thật vậy (1) đúng với n = 0, 1, 2, ... Giả sử (1) đúng với n và do (an) là dãy giảm nên ta có 3x3n+3 = x2 3n + x2 3n+2 ≤ 2a2 n ⇒ x3n+3 ≤ an+1 3x3n+4 = x2 3n+1 + x2 3n+3 ≤ a2 n + a2 n+1 ≤ 2a2 n ⇒ x3n+4 ≤ an+1 3x3n+5 = x2 3n+2 + x2 3n+4 ≤ a2 n + a2 n+1 ≤ 2a2 n ⇒ x3n+5 ≤ an+1. Như vậy (1) đúng với n + 1, theo nguyên lý quy nạp, (1) được chứng minh. Dễ thấy xn > 0. Từ đó theo nguyên lý kẹp giữa ta có lim x3n+i = 0 (i = 0, 1, 2) do đó lim xn = 0. Từ các cách chọn dãy số phụ như trên ta có các dãy số sau đều hội tụ về 0 với x0, x1, x2, x3 đều thuộc (0; 1) 3xn+3 = x2 n + xn+1xn+2, 3xn+3 = x2 n + xnxn+1, 3xn+3 = x2 n + x2 n+2 2 + x2 n+1, 6xn+4 = xn+1xn+2 + x2 n + 2xnxn+1. 67
  • 70. Bài tập 3.2.3. Dãy (xn) được xác định bởi:    x0, x1, x2 > 0 xn+3 = √ xn + xn+2. Chứng minh rằng dãy (xn) hội tụ. Lời giải. Ta xây dựng hai dãy (an) và (bn) như sau:    a0 = max{x0, x1, x2, 2} an+1 = √ 2an    b0 = min{x0, x1, x2, 2} bn+1 = √ 2bn. Dãy (an) là dãy giảm dần về 2, dãy (bn) tăng dần về 2. Bằng quy nạp dễ dàng chứng minh được bn+1 ≤ min{x3n, x3n+1, x3n+2} ≤ max{x3n, x3n+1, x3n+2} ≤ an với mọi n. Từ đó dẫn đến lim x3n = lim x3n+1 = lim x3n+2 = 2 suy ra lim xn = 2. Bài tập 3.2.4. Cho dãy (xn) (n = 0, 1, 2, ...) được xác định như sau:    x0, x1, x2 là các số dương cho trước xn+2 = √ xn+1 + √ xn + √ xn−1 với mọi n ≥ 1. Chứng minh rằng dãy (xn) hội tụ và tìm giới hạn của dãy. Lời giải. Ta xây dựng hai dãy (an) và (bn) như sau:    a0 = max{x0, x1, x2, 9} an+1 = 3 √ an, n = 0, 1, 2, ...    b0 = min{x0, x1, x2, 9} bn+1 = 3 √ bn, n = 0, 1, 2, ... Dãy (an) là dãy giảm dần về 9, dãy (bn) tăng dần về 9 suy ra lim n→+∞ an = lim n→+∞ bn = 9. Ta chứng minh bn+1 ≤ min{x3n, x3n+1, x3n+2} ≤ max{x3n, x3n+1, x3n+2} ≤ an với mọi n. (1) 68
  • 71. Thật vậy với n = 0 thì (1) hiển nhiên đúng. Giả sử (1) đúng với n = k, khi đó với n = k + 1 ta có bn ≤ bn+1 = 3 bn ≤ x3k+3 = √ x3k+2 + √ x3k+1 + √ x3k ≤ ≤ 3 √ an = an+1 ≤ an bn ≤ bn+1 = 3 bn ≤ x3k+4 = √ x3k+3 + √ x3k+2 + √ x3k+1 ≤ ≤ 3 √ an = an+1 ≤ an bn ≤ bn+1 = 3 bn ≤ x3k+5 = √ x3k+4 + √ x3k+3 + √ x3k+2 ≤ ≤ 3 √ an = an+1 ≤ an. Vậy (1) cũng đúng với n = k + 1. Theo nguyên lý quy nạp thì (1) đúng với mọi số tụ nhiên n. Từ đó theo nguyên lý kẹp ta có lim n→+∞ x3n = lim n→+∞ x3n+1 = lim n→+∞ x3n+2 = lim n→+∞ an = lim n→+∞ bn = 9 ⇒ lim n→+∞ xn = 9. Dưới đây là một số bài toán tìm giới hạn dãy số dạng xn+1 = f(xn) (dãy số xác định như vậy gọi là cho dưới dạng lặp). Đây là dạng toán thường gặp nhất trong các bài toán về tìm giới hạn dãy số, dãy số hoàn toàn được xác định khi biết f và giá trị ban đầu x0. Do vậy sự hội tụ của dãy số phụ thuộc vào tính chất của f(x) và x0. Một đặc điểm quan trọng khác của dãy số dạng này là nếu a là giới hạn của dãy số thì a là nghiệm của phương trình x = f(x). Bài tập 3.2.5. Cho dãy số (xn) được xác định như sau: x1 = 0, xn+1 = 1 27 xn với mọi n ∈ N∗ . Chứng minh rằng dãy số (xn) có giới hạn và tìm giới hạn đó. Lời giải. Nhận xét rằng xn ≥ 0 với mọi n ∈ N∗ . Xét hàm số f(x) = 1 27 x nghịch biến trong khoảng [0; +∞). Khi đó xn+1 = f(xn) với mọi n ∈ N∗ và f(x) ≤ f(0) nên 0 ≤ xn ≤ 1. Ta có x1 = 0, x2 = 1, x3 = 1 27 nên 69
  • 72. x1 ≤ x3 và x4 = f(x3) ≤ f(x1) = x2 Bây giờ ta chứng minh bằng phương pháp quy nạp x2n−1 ≤ x2n+1 và x2n+2 ≤ x2n với n ∈ N∗ . Thật vậy giả sử có x2n−1 ≤ x2n+1 thì f(x2n−1) ≥ f(x2n+1) nên x2n ≥ x2n+2 và vì vậy f(x2n) ≤ f(x2n+2) suy ra x2n+1 ≤ x2n+3. Tương tự, giả sử có x2n ≥ x2n+2 thì f(x2n) ≤ f(x2n+2) suy ra x2n+1 ≤ x2n+3. Vì vậy f(x2n+1) ≥ f(x2n+3) suy ra x2n+2 ≥ x2n+4. Vậy dãy (x2n−1) là dãy tăng và dãy (x2n) là dãy giảm và đều thuộc [0; 1] nên có giới hạn hữu hạn: lim n→+∞ x2n = a; lim n→+∞ x2n−1 = b. Và a = lim n→+∞ x2n+2 = lim n→+∞ f(x2n+1) = lim n→+∞ f(f(x2n)) = f(f(a)) nên a = 1 27 ( 1 27 )a ⇒ a = 1 3 . Tương tự ta cũng tìm được b = 1 3 . Vậy a = b = 1 3 nên lim n→+∞ xn = 1 3 . Bài tập 3.2.6 (HSG QG 2008). Cho dãy số thực (xn) xác định như sau: x1 = 0, x2 = 2 và xn+2 = 2−xn + 1 2 với mọi n = 1, 2, 3, ... Chứng minh rằng dãy số (xn) có giới hạn hữu hạn và tìm giới hạn đó. Lời giải. Xét hàm số f(x) = 2−x + 1 2 xác định trên R. Với mỗi n ∈ N∗ , ta có xn+4 = f(xn+2) = f(f(xn)) hay xn+4 = g(xn), trong đó g là hàm số xác định trên R và g(x) = f(f(x)) với mọi x ∈ R. (1) Dễ thấy hàm số f giảm trên R, do đó hàm số g tăng trên R. Vì thế từ (1) suy ra với mỗi k ∈ {1; 2; 3; 4}, dãy (x4n+k), n ∈ N là dãy đơn điệu. Hơn nữa, 70
  • 73. từ cách xác định dãy (xn) dễ thấy 0 ≤ xn ≤ 2 với mọi n ∈ N∗ . Do đó với mỗi k ∈ {1; 2; 3; 4}, dãy (x4n+k) là dãy hội tụ. Với mỗi k ∈ {1; 2; 3; 4}, đặt lim n→+∞ x4n+k = ak ta có 0 ≤ ak ≤ 2. Hơn nữa do hàm số g liên tục trên R nên từ (1) suy ra g(ak) = ak. (2) Xét hàm số h(x) = g(x) − x trên [0; 2]. Ta có h (x) = 2−(f(x)+x) · (ln 2)2 − 1 < 0 với mọi x ∈ [0; 2] (do f(x)+x > 0 với mọi x ∈ [0; 2]). Suy ra, hàm số h giảm trên [0; 2]. Vì thế có nhiều nhất một điểm x ∈ [0; 2] sao cho h(x) = 0 hay g(x) = x. Mà g(1) = 1 nên từ (2) ta được ak = 1 với mọi k ∈ {1; 2; 3; 4}. Từ đây, vì dãy (xn) là hợp của bốn dãy con (x4n+k) nên dãy (xn) hội tụ và lim n→+∞ xn = 1. Bài tập 3.2.7. Cho dãy số thực (xn) xác định bởi:    x1 = 2007 xn+1 = √ 3 + xn x2 n − 1 với mọi n ≥ 1. 1/ Chứng minh rằng dãy (xn) bị chặn. 2/ Chứng minh rằng dãy (xn) có giới hạn và tìm giới hạn đó. Lời giải. Hiển nhiên xn > √ 3. xn+1 = √ 3 + xn x2 n − 1 = √ 3 + 1 + 1 x2 n − 1 < √ 3 + √ 2 với mọi n ≥ 1. Vậy xn ≤ 2007 với mọi n, suy ra dãy (xn) bị chặn. Hàm f(x) = √ 3 + x √ x2 − 1 = √ 3 + 1 + 1 x2 − 1 71
  • 74. nghịch biến trên [ √ 3; +∞) nên chứng minh được các dãy con (x2n) và (x2n+1) là đơn điệu. Theo 1/ các dãy đó bị chặn nên có lim x2n = a, lim x2n+1 = b. Từ xn+1 = √ 3 + xn x2 n − 1 qua giới hạn, ta có    a = √ 3 + b √ b2 − 1 b = √ 3 + a √ a2 − 1 ⇒ a + a √ a2 − 1 = b + b √ b2 − 1 . (*) Xét g(x) = x + x √ x2 − 1 có g (x) = 1 − 1 (x2 − 1) √ x2 − 1 > 0 với mọi x ≥ √ 3 nên g(x) đồng biến. Nên từ (∗) suy ra a = b hay lim x2n = lim x2n+1 ⇒ lim xn = a = b. Lúc đó a là nghiệm của phương trình x = √ 3 + x √ x2 − 1 ⇔ x = √ 3 + √ 15 2 ⇒ lim xn = √ 3 + √ 15 2 . Bài tập 3.2.8. Cho dãy số (xn) thỏa mãn:    x1 = √ a, a > 2 xn+1 = a − √ a + xn với mọi n ∈ N∗ . Chứng minh rằng dãy (xn) có giới hạn hữu hạn. Lời giải. Bằng quy nạp ta chứng minh được rằng 0 ≤ xn ≤ √ a với mọi n ∈ N∗ . Xét hàm f(x) = a − √ a + x, với mọi x ∈ [0; √ a], có xn+1 = f(xn) và f (x) = − 1 4 a − √ a + x √ a + x < 0, với mọi x ∈ [0; √ a]. Suy ra f(x) là hàm nghịch biến. Do đó dãy (xn) được tách thành hai dãy con (x2n) và (x2n+1), trong đó một 72
  • 75. dãy tăng và một dãy giảm, mặt khác lại có dãy (xn) bị chặn nên tồn tại lim x2n = α, lim x2n+1 = β, trong đó α, β là nghiệm của phương trình f(f(x)) = x ⇔ a − a + a − √ a + x = x. Xét hàm F(x) = a − a + a − √ a + x − x, với x ∈ [0; √ a]. Ta có F (x) = 1 a − a + a − √ a + x a + a − √ a + x a − √ a + x √ a + x −1. Với x ∈ [0; √ a], ta có a − √ a + x · √ a + x ≥ a − a + √ a · √ a > a − a + √ a + 1 4 · √ a = = a − √ a − 1 2 · √ a = √ a − 1 2 2 − 3 4 · √ 2 > √ 2 − 1 2 2 − 3 4 · √ 2 > > 0, 12 > 0, 3. Thay vai trò của x bởi a − √ a + x chứng minh tương tự ta có a − a + a − √ a + x · a + a − √ a + x > 0, 3. Suy ra F (x) < −0, 9 < 0 nên F(x) là hàm nghịch biến, lại có F(0) > 0, F( √ a) < 0 nên phương trình F(x) = 0 có nghiệm duy nhất. Do đó α = β. Suy ra lim x2n = lim x2n+1 = lim xn. Suy ra lim xn = T với T thỏa mãn f(f(T)) = T. 3.3. Dãy số xác định bởi phương trình Dãy số có mối quan hệ chặt chẽ với phương trình điều này thấy rõ qua hai nội dung cơ bản là phương trình sai phân tuyến tính được giải bằng phương 73
  • 76. trình đặc trưng, giới hạn của dãy số cũng thường được giải ra từ phương trình. Đây là một trong các nội dung quan trọng nhất của phần dãy số. Với dạng toán tìm giới hạn của dãy số có liên quan đến phương trình ta thường xét tính đơn điệu của hàm số, áp dụng định lý Lagrange và định lý về giới hạn kẹp giữa. Bài tập 3.3.1. Giả sử xn ∈ (0; 1) là nghiệm của phương trình 1 x + 1 x − 1 + · · · + 1 x − n = 0. Chứng minh rằng dãy (xn) hội tụ. Tìm giới hạn đó. Nhận xét. xn được xác định duy nhất vì hàm số fn(x) = 1 x + 1 x − 1 + · · · + 1 x − n liên tục và đơn điệu trên (0; 1). Tuy nhiên, ta không thể xác định được giá trị cụ thể của xn. Rất may mắn, để chứng minh tính hội tụ của xn, ta không cần đến điều đó. Chỉ cần chứng minh tính đơn điệu và bị chặn là đủ. Với tính bị chặn là hiển nhiên vì 0 < xn < 1. Với tính đơn điệu, ta chú ý một chút đến mối liên hệ giữa fn(x) và fn+1(x): fn+1(x) = fn(x) + 1 x − n − 1 . Đây chính là chìa khóa để chứng minh tính đơn điệu của xn. Lời giải. xn được xác định duy nhất vì hàm số fn(x) = 1 x + 1 x − 1 + · · · + 1 x − n liên tục và đơn điệu trên (0; 1). Để chứng minh dãy hội tụ, ta chứng minh dãy (xn) bị chặn và đơn điệu. Hiển nhiên dãy bị chặn vì 0 < xn < 1. Bây giờ, ta chứng minh dãy (xn) đơn điệu. Ta thấy 0 < xn < 1 nên fn+1(xn) = fn(xn) + 1 xn − n − 1 = 1 xn − n − 1 < 0. 74